Sei sulla pagina 1di 76

FUNDAMENTALS IN NURSING Situation 1. The nursing process provides a framework for a nurse's responsibility and accountability.

It requires critical thinking. 1. The patient states, My chest hurts and my left arm feels numb. What is the type and source of this data? A. Subjective data from a primary source B. Subjective data from a secondary source C. Objective data from a primary source D. Objective data from a secondary source Answer: A Rationale: Subjective data is apparent only to the person affected and cannot be measured, seen, felt, or heard by the nurse. It may be called covert data. It includes the patient's thoughts, beliefs, feelings, perceptions, and sensations. The patient is always considered the primary source. (Hogan et. al., Prentice Hall Reviews and Rationales Series for Nursing: Fundamentals of Nursing) 2. The nurse is measuring the patient's urine output and straining the urine to assess for stones. Which of the following should the nurse record as objective data? A. The patient stated, I feel like I have passed a stone. B. The patient's urine output was 550 mL C. The patient is complaining of abdominal pain D. The patient stated, I didn't see any stones in my urine. Answer: B Rationale: Measurable data is objective data. (Hogan et. al., Prentice Hall Reviews and Rationales Series for Nursing: Fundamentals of Nursing) 3. Which of the following demonstrates that the nurse is participating critical thinking? A. The nurse admits she does not know how to do a procedure and requests help B. The nurse makes her point with clever and persuasive remarks to win an argument C. The nurse accepts without question the values acquired in nursing school D. The nurse finds a quick and logical answer, even to complex questions Answer: A Rationale: Critical thinking is self-directed and supports what an individual knows and makes clear what she does not know. It is important for nurses to recognize when they lack the knowledge they need to provide safe care for a client. Nurses must utilize their resources to acquire the knowledge and support they need to fulfill a nursing responsibility safely. . (Hogan et. al., Prentice Hall Reviews and Rationales Series for Nursing: Fundamentals of Nursing) 4. What is the problem with the following outcome goal, Patient will state pain is less than or equal to 3 on a 0 to 10 pain scale? A. None, goal is written correctly B. It is not measurable C. No target time is given D. Patient behavior is missing Answer: C Rationale: Outcome goals should be SMART (specific, measurable, appropriate, realistic, and timely). There is no time estimate for goal attainment. Thus, option A is incorrect. (Hogan et. al., Prentice Hall Reviews and Rationales Series for Nursing: Fundamentals of Nursing) 5. When evaluating an adult patient's blood pressure reading. The nurse considers the patient's age. This is an example of which of the following? A. Comparing data against standards B. Clustering data C. Determining gaps in the data D. Differentiating cues and inferences Answer: A Rationale: Analysis of the client data (blood pressure reading) requires knowledge of the normal blood pressure range for an adult. The nurse compares client data against standards to identify significant cues. (Hogan et. al., Prentice Hall Reviews and Rationales Series for Nursing: Fundamentals of Nursing) Situation 2. Jason has a nursing diagnosis of ineffective airway clearance related to excessive secretions and is at risk for infection because of retained secretions. Part of Nurse Melais nursing care plan is to loosen and remove excessive secretions in the airway. 6. Nurse Melai listens to Jasons bilateral sounds and finds that congestion is in the upper lobes of the lungs. The appropriate position to drain the anterior and posterior apical segment of the lungs when the nurse does percussion would be: A. Patient lying on his back then flat on his abdomen on Trendelenburg position B. Patient seated upright in bed or on a chair then leaning forward in sitting position then flat on his back and on his abdomen C. Patient lying flat in his back and then flat on his abdomen D. Patient lying on his right then left side on Trendelenburg position Answer: B Rationale: Chest physiotherapy is a dependent nursing function that uses principles of percussion, vibration and postural drainage to drain thick tenacious bronchial secretions. Percussion is done by clapping cupped hand over the affected lobe of the lung. Positioning varies according to the area of

East West Nursing Review Center

affectation. Patients with posterior segment of the upper lobe affectation should be positioned in a sitting and leaning forward position and flat on bed (dorsal recumbent) with pillow under the buttocks for anterior segment of upper lobe affectation. 7. When documenting the outcome of Jasons treatment Nurse Melai should include the following in her recording, except: A. Color, amount, consistency of sputum B. Character of breath sounds and respiratory rate before and after procedure C. Amount of fluid intake of the patient before and after the procedure D. Significant changes in vital signs Answer: C Rationale: Though patients receiving Chest Physiotherapy are encouraged to increase oral fluid intake, this intervention is not too specific for documentation of pertinent data related to the procedure. 8. When assessing Jason for chest percussion or chest vibration and postural drainage, Nurse Melai would focus on the following, except: A. Amount of fluid taken during the last meal before treatment B. Respiratory rate, breath sounds and location of congestion C. Teaching the patients relatives to perform the procedure D. Doctors order regarding position restrictions and the patients tolerance for lying flat Answer: C Rationale: Options A, B and D are all assessable, C is an intervention that is not allowed. Chest Physiotherapy is a dependent nursing intervention that cannot be just delegated to the patients relatives. Even for home/community based care or long term care of patients; CPT is done during home visits by a home care nurse and not delegated to the relatives. 9. Nurse Melai prepares Jason for postural drainage and percussion. Which of the following is a special consideration when doing the procedure? A. Respiratory rate of 16-20 per minute B. Patient can tolerate sitting and lying positions C. Patient has no sign of infection D. Time of the last food and fluid intake of the patient Answer: D Rationale: The time of last food and fluid intake of the client is very important for the nurse to assess. The best time to perform chest physiotherapy is 1 hour before meals or 2-3 hours after meals to prevent food and fluid regurgitation or vomiting. 10. The purpose of chest percussion and vibration is to loosen secretions in the lungs. The difference between the procedures is: A. Percussion uses only one hand while vibration uses both hands B. Percussion delivers cushioned blows to the chest with cupped palms while vibration gently shakes secretion loose on the exhalation cycle C. In both percussion and vibration the hands are on top of each other and hand action is in tune with the clients breathing rhythm D. Percussion slaps the chest to loosen secretions while vibration shakes the secretions with inhalation of air Answer: B Rationale: Option B is the correct comparison. Situation 3. The vital signs are body temperature, pulse, respirations, and blood pressure. These signs, which should be looked at in total, are checked to monitor the functions of the body. They reflect changes in function that otherwise might not be observed. 11. For a patient with a previous blood pressure of 140/80 and pulse of 64, approximately how long should the nurse take to release the blood pressure cuff in order to obtain an accurate reading? A. 10 to 20 seconds C. 30 to 45 seconds B. 1 to 1.5 minutes D. 3 to 3.5 minutes Answer: C Rationale: If the cuff is inflated to about 30 mm Hg over the previous systolic pressure, which would be 170. To ensure that the diastolic has been determined, the cuff should be released slowly until the mid-60s mm Hg (and then completely) for someone with a previous reading of 80. The cuff should be deflated at a rate of 2 to 3 mm per second. Thus, a range of 90 mm Hg will require 30 to 45 seconds. (Kozier & Erb's Fundamentals of Nursing, 8th Edition) 12. A patient with pyrexia will most likely demonstrate: A. Dyspnea C. Increased pulse rate B. Precordial pain D. Elevated blood pressure Answer: C Rationale: The pulse increases to meet increased tissue demands for oxygen in the febrile state. (Mosby, 18th Edition) 13. Which of the following patients meet the criteria for selection of the apical site for assessment of the pulse rather than a radial pulse? A. A patient is in shock B. The pulse changes with body position changes

East West Nursing Review Center

C. A patient with an arrhythmia D. It is less than 24 hours since a patient's surgical operation Answer: C Rationale: The apical rate would confirm the rate and determine the actual cardiac rhythm for a client with an abnormal rhythm; a radial pulse would only reveal the heart rate and suggest an arrhythmia. (Kozier & Erb's Fundamentals of Nursing, 8th Edition) 14. The absence of which pulse may not be a significant finding when a patient is admitted to the hospital? A. Pedal C. Apical B. Femoral D. Radial Answer: A Rationale: Because the pedal pulse cannot be detected in 10% to 20% of the population, its absence is not necessarily a significant finding. However, the presence or absence of the pedal pulse should be documented upon admission so that changes can be identified during the hospital stay. (Kozier & Erb's Fundamentals of Nursing, 8th Edition) 15. All of the following can cause tachycardia, except: a. Sympathetic nervous system stimulation b. Parasympathetic nervous system stimulation c. Fever d. Exercise Answer: B Rationale: Parasympathetic nervous system stimulation of the heart decreases the heart rate as well as the force of contraction, rate of impulse conduction and blood flow through the coronary vessels. (Kozier & Erb's Fundamentals of Nursing, 8th Edition) Situation 4. Diagnostic and laboratory tests (commonly called lab tests) are tools that provide information about the client. Frequently, tests are used to help confirm a diagnosis, monitor an illness, and provide valuable information about the client's response to treatment. 16. A patient is admitted to the hospital with complaints of nausea, vomiting, diarrhea, and severe abdominal pain. Which of the following would immediately alert the nurse that the patient has gastrointestinal tract bleeding? A. Complete blood count C. Vital signs B. Guaiac test D. Abdominal girth Answer: B Rationale: To assess for GI tract bleeding when frank blood is absent, the nurse has two options: She can test for occult blood in vomitus, if present, or in stool - through Guaiac (Hemoccult) test. (Kozier & Erb's Fundamentals of Nursing, 8th Edition)

17. Before scheduling a patient for endoscopic retrograde cholangiopancreatography (ERCP), the nurse should assess the patient's: A. Urine output C. Serum glucose B. Bilirubin leve D. Blood pressure Answer: B Rationale: ERCP or endoscopic retrograde cholangiopancreatography involves the insertion of a cannula into the pancreatic and common bile ducts during an endoscopy. The test is not performed if the client's bilirubin is greater than 3 to 5 mg/dL because cannulization may cause edema, which would increase obstruction of bile flow. (Mosby, 18th Edition) 18. The laboratory tests that would indicate that the liver of a patient with cirrhosis is compromised and neomycin enemas might be helpful would be: A. Ammonia level C. Culture and sensitivity B. White blood count D. Alanine aminotransferase level Answer: A Rationale: Increased ammonia levels indicate that the liver is unable to detoxify protein byproducts. Neomycin reduces the amount of ammonia-forming bacteria in the intestines. (Mosby, 18th Edition) 19. The most important test used to determine whether a transplanted kidney is working is: A. Renal ultrasound C. White blood cell count B. Serum creatinine level D. Twenty-four hour output Answer: B Rationale: Serum creatinine concentration measures the kidney's ability to excrete metabolic wastes. Creatinine, a nitrogenous product of protein breakdown, is elevated in renal insufficiency. (Mosby, 18th Edition) 20. The best blood test for the nurse to use to evaluate fluid loss resulting from burns is the: A. Blood urea nitrogen C. Hematocrit B. Blood pH D. Sedimentation rate Answer: C

East West Nursing Review Center

Rationale: An increased hematocrit level indicates hemoconcentration secondary to fluid loss. Option A is incorrect because although blood urea nitrogen (BUN) may be used to indicate dehydration from burns, interpretation can be complicated by other conditions accompanying burns that also cause elevation of the BUN. (Mosby, 18th Edition) Situation 5. Nurse Flo is assigned to the triage area and while on duty, she assesses the condition of Mr. Floremonte who came in with asthma. He has difficulty of breathing and his respiratory rate is 40 breaths per minute. The nurse is asked to inject the patient epinephrine 0.3mg subcutaneously. 21. The indication for epinephrine injection for Mr. Floremonte is to: A. Reduce anaphylaxis B. Relieve hypersensitivity to allergen C. Relieve respiratory distress due to bronchial spasm D. Restore the patients cardiac rhythm Answer: C Rationale: Epinephrine is a sympathomimetic that is frequently used in emergencies to combat anaphylaxis, which is a life threatening allergic response. It is a potent inotropic agent that causes the blood vessels to constrict; thus blood pressure increases, and the bronchial tubes to dilate. Epinephrine increases cAMP (cyclic adenosine monophosphate), a molecule responsible for maintaining bronchodilation. Epinephrine is the drug of choice for the treatment of anaphylactic shock. 22. When preparing the epinephrine injection from an ampule, Nurse Flo initially: A. Taps the ampule at the top to allow fluid to flow to the base of the ampule B. Checks expiration date of the medication ampule C. Removes the needle cap of syringe and pulls plunger to expel air D. Breaks the neck of the ampule with a gauze around it Answer: B Rationale: Before preparing any medication for administration, it is a vital role of the nurse to check the integrity of the drug and its container, including the expiry date of the medication. Doing option A before option B would make option A useless. 23. Mr. Floremonte is obese. When administering subcutaneous injection to an obese patient, it is best for Nurse Flo to: A. Inject needle at a 15 degree angle over the stretched skin of the patient B. Pinch the skin and use an airlock technique C. Pull skin of the patient down to administer drug in a Z-track D. Spread the skin or pinch at the injection site and inject needle at a 45-90 degree angle Answer: D Rationale: Subcutaneous (SC) injections have systemic and sustained effects, absorbed mainly through the capillaries, usually slower in onset that Intramuscular (IM). SC route is used for small doses of non-irritating, water-soluble drugs. Locations for SC injections are chosen for adequate fatpad size and include the abdomen, upper hips, upper back, lateral upper arms and upper thighs. Insert the needle at an angle appropriate to body size 45 degrees (for those with little SC tissue) to 90 degrees. 24. When preparing for a subcutaneous injection, the proper size of syringe and needle would be: A. Syringe 3-5mL and needle 21-23 B. Tuberculin syringe 1mL with needle gauge 26-27 C. Syringe 2mL and needle gauge 22 D. Syringe 1-3mL and needle gauge 25-27 Answer: D Rationale: Since for SC injections, usually 0.5-1.5mL is injected, a 1-3mL syringe is recommended with a needle gauge 25-27 with 1/2 to 5/8 inches of needle length. 25. The rationale for giving medications through the subcutaneous route is: A. There are many alternative sites for SC injection B. Absorption time of the medicine is slower C. There are less pain receptors in this area D. The medication can be injected while the patient is in any position Answer: B Rationale: Since the subcutaneous layer as vascularized as the muscle layer, the medicine is not as rapidly absorbed as that in IM route. Absorption in the SC route occurs via the capillaries while in IM route, it can be through capillaries and main blood vessels. Situation 6. When positioning clients in bed, the nurse can do a number of things to ensure proper alignment and promote client comfort and safety. 26. When Rosenda, age 21, is in the right side-lying position after the insertion of a left hip prosthesis, Nurse Claudine ensures that the patient has an abduction pillow placed between the thighs and that the entire length of the upper leg is supported. The most important reason for this is to prevent: A. Strain on the operative site B. Thrombus formation in the leg C. Flexion contractures of the hip joint

East West Nursing Review Center

D. Skin surfaces from rubbing together Answer: A Rationale: The right side-lying position supports the operative site; the involved leg must be maintained in alignment, avoiding adduction to prevent dislocation of the prosthesis. (Mosby, 18th Edition) 27. The position that is indicated for Rosanna, age 23, after surgery for a perforated appendix with localized peritonitis is the: A. Sims' position C. Semi-Fowler's position B. Trendelenburg position D. Dorsal recumbent position Answer: C Rationale: The semi-Fowler's position aids in drainage and prevents spread of infection throughout the abdominal cavity. (Mosby, 18th Edition) 28. After surgery on the neck, Rosita, age 22, should be placed in a high-Fowler's position to: A. Avoid strain on the incision B. Promote drainage of the wound C. Provide stimulation for the patient D. Reduce edema at the operative site Answer: D Rationale: The high-Fowler's position promotes fluid drainage by gravity, minimizing edema. (Mosby, 18th Edition) 29. After a total hip replacement surgery, Nurse Claudine should avoid placing Rosario, age 25, in the: A. Supine position C. Orthopneic position B. Lateral position D. Semi-Fowler's position Answer: C Rationale: The orthopneic position involves hip flexion greater than 90 degrees. This puts stress on the operative site and could dislodge the prosthesis. (Mosby, 18th Edition) 30. Rosemary, age 20, is to have gastric gavage. When the gavage tube is being inserted, Nurse Claudine should place the patient in the: A. Supine position C. High-Fowler's position B. Mid-Fowler's position D. Trendelenburg position Answer: C Rationale: The high-Fowler's position promotes optimal entry into the esophagus aided by gravity. (Mosby, 18th Edition) Situation 7. Nurse Junelyn is assigned in the medical-surgical unit to provide basic nursing care to a group of clients with nasogastric tube. 31. Nurse Junelyn is to irrigate a nasogastric tube every two hours. Which solution should the nurse select to irrigate the tube? A. Normal saline C. Ringer's lactate B. Tap water D. Half-strength peroxide Answer: A Rationale: Normal saline is used to irrigate a nasogastric tube since it will not cause a loss of sodium when it is removed by suction. Saline is an isotonic solution which has the same osmotic pressure as that found across the semi-permeable membrane within the cell. (NSNA NCLEX-RN Review, 4th Edition) 32. Mr. Beans, a 37-year-old client with a nasogastric tube connected to a low continuous suction for abdominal decompression. Nurse Junelyn notes that gastric fluid in the suction tubing is not moving and the patient's abdomen is becoming distended. The nurse's best action is to: A. Pull out the nasogastric tube and insert a new one B. Irrigate the tube with 30 cc of water C. Tell the client to take a few deep breaths D. Turn the suction higher Answer: B Rationale: The most likely cause of the problem is that the nasogastric tube is plugged with gastric contents or has adhered to the gastric mucosa and is no longer draining. Irrigating the tube with 30 cc of water should clear any obstructions and free the tube from the gastric mucosa. (Davis's NCLEX-RN Success, 2nd Edition) 33. Mr. Borat, a 45-year-old male patient, has an order for a nasogastric tube. Before inserting the tube, Nurse Junelyn measures the amount of tube needed. To determine the amount of tube needed, the nurse should: A. Measure from the forehead to the ear and from the ear to the umbilicus B. Measure from the chin to the back of the throat and from the back of the throat to the umbilicus C. Measure from the mouth to the xiphoid process and add 2 inches D. Measure from the tip of the patient's nose to his earlobe and from the earlobe to the xiphoid process Answer: D Rationale: The best way to determine the amount of tube needed to reach the pylorus is to measure from the tip of the client's nose to his earlobe and from the earlobe to the xiphoid process.

East West Nursing Review Center

(NSNA NCLEX-RN Review, 4th Edition) 34. When assessing Mr. Peas, a postoperative patient, Nurse Junelyn notes a nasogastric tube to low constant suction, the absence of a bowel movement since surgery, and no bowel sounds. The most appropriate plan of care based on these findings is to: A. Increase the patient's mobility and ensure he is receiving adequate pain relief B. Increase coughing, turning, and deep breathing exercises C. Discontinue the nasogastric tube as the patient does not need it any more D. Assess for bladder pain and distention Answer: A Rationale: Paralytic ileus can be related to immobility and inadequate pain medication as well as bowel manipulation and the anesthetic used during surgery. (NSNA NCLEX-RN Review, 4th Edition) 35. Nurse Junelyn is caring for Mr. Turnips who has a nasogastric tube attached to low wall suction. The suction is not working. Which is the nurse least likely to note when assessing the patient? A. Patient vomits B. Patient has a distended abdomen C. There is no nasogastric output in the last two hours D. Large amounts of nasogastric output Answer: D Rationale: If the nasogastric suction is not working, the nurse would not expect to see large amounts of nasogastric output. (NSNA NCLEX-RN Review, 4th Edition) Situation 8. Nurse Bro is assigned to a group of clients with chest tubes. 36. The physician inserts a chest tube to Mariel, a 24-year-old patient who has been stabbed in the chest, and attaches it to a closed-drainage system. When caring for the patient, Nurse Bro should: A. Apply a thoracic binder to prevent tension on the tube B. Observe for fluid fluctuations in the water seal chamber C. Clamp the tubing securely to prevent a rapid decline in pressure D. Administer morphine sulfate, because the patient will be agitated Answer: B Rationale: Fluctuations occur with normal inspiration and expiration until the lung is fully expanded. If these fluctuations do not occur, the chest tube may be clogged or kinked; coughing should be encouraged. (Mosby, 18th Edition) 37. Paul Jake, age 25, arrives in the postanesthesia care unit after a segmental resection of the right lower lobe of the lung, with a chest tube drainage system in place. Nurse Bro caring for the client should: A. Add 3 to 5 ml of sterile saline to the water seal chamber B. Raise the drainage system to bed level to check its patency C. Mark the time and the fluid level on the side of the drainage chamber D. Secure the chest catheter to the wound dressing with a sterile safety pin Answer: C Rationale: The fluid level and time must be marked so that the amount of drainage in the chest tube drainage system can be evaluated. (Mosby, 18th Edition)

38. Tibo, age 27, has a chest tube to a Pleur-evac drainage system attached to wall suction. An order
to ambulate the patient has been received. To ambulate the patient safely, Nurse Bro should: A. Clamp the chest tube and carefully ambulate the patient a short distance B. Question the order to ambulate the patient C. Carefully ambulate the patient, keeping the Pleur-evac lower than the patient's chest D. Disconnect the Pleur-evac from the patient's chest tube, leave it attached to the bed, ambulate the patient, and then reconnect the chest tube when he is returned to bed Answer: C Rationale: The Pleur-evac must not be raised above chest level because it can cause back flow of the fluid into the pleural space precipitating collapse of the lung or mediastinal shift. The Pleurevac must remain upright and the chest tube should not have traction on it. Option A is incorrect because the chest tube should not be clamped for ambulation. Chest tubes should not be clamped unless an order to do so is present. (NSNA NCLEX-RN Review, 4th Edition) 39. Kathy, a 30-year-old client with a spontaneous pneumothorax, has had a chest tube for 3 days. On morning rounds, the physician clamped the chest tube to determine the patient's readiness to have the chest tube discontinued. Two hours after having the chest tube clamped, the patient began to have difficulty breathing. What action should Nurse Bro take first? A. Notify the physician B. Unclamp the chest tube C. Assess the patient for subcutaneous emphysema D. Place the patient on 2L nasal cannula oxygen Answer: B Rationale: The chest tube should be immediately unclamped. The client still has an air leak that is

East West Nursing Review Center

causing a build up of air in the pleural space, collapsing part of the lung, and causing breathing difficulty. The clamp must be removed from the chest tube immediately to allow this air to escape and the lung to re-expand. Option A is incorrect because the first priority is to remain with the client and address the breathing difficulty by unclamping the chest tube clamp. The physician should be notified as priority number 4 in this option sequence. (Davis's NCLEX RN Success, 2nd Edition) 40. Johan, a 29-year-old client who has had thoracic surgery, is admitted to the postanesthesia care unit. After the chest catheters are attached to a closed drainage system, Nurse Bro should: A. Check that the fluid in the water seal compartment rises with expiration B. Ensure the security of the connections from the patient to the drainage unit C. Ensure that there is vigorous bubbling in the wet suction control compartment D. Empty the drainage container, measure and record the amount, and send a sample for analysis every 24 hours Answer: B Rationale: The system must remain airtight (closed) to prevent collapse of the lung. (Mosby, 18th Edition) Situation 9. Gibo Posible is returned to the medical-surgical floor after tracheostomy insertion. His respirations are regular and unlabored. 41. When performing tracheostomy care, Nurse Edu must: A. Place the patient in the semi-Fowler's position B. Maintain sterile technique during the procedure C. Monitor the patient's temperature after the procedure D. Use Betadine to clean the inner cannula when it is removed Answer: B Rationale: The tracheostomy site is a portal of entry for microorganisms. Sterile technique must be used. (Mosby, 18th Edition) 42. Mr. Posible's tracheostomy is producing a small amount of thin, white secretions. The stoma is pink with no drainage noted. Nurse Edu should expect to provide tracheostomy care for the patient every: A. Four hours C. 24 hours B. Eight hours D. Hour Answer: B Rationale: Tracheostomy care should be provided once every eight hours. (NSNA NCLEX-RN Review, 4th Edition) 43. Nurse Edu is administering tracheostomy care to Mr. Posible. Which of the following should be included in the procedure? A. Soaking the outer cannula with saline solution B. Performing the procedure utilizing medical asepsis C. Soaking the inner cannula in half-strength hydrogen peroxide solution D. Cutting a sterile gauze pad to place between the neck and the tracheostomy tube Answer: C Rationale: The inner cannula is removed utilizing sterile gauze and is soaked in half-strength hydrogen peroxide solution. Clean the inner cannula with a small brush or pipe cleaners. Rinse in sterile saline or water and replace after the outer cannula has been suctioned. (NSNA NCLEX-RN Review, 4th Edition) 44. When suctioning Mr. Posible, with a tracheostomy, Nurse Edu must remember to: A. Use a new sterile catheter with each insertion B. Initiate suction as the catheter is being withdrawn C. Insert the catheter until the cough reflex is stimulated D. Remove the inner cannula before inserting the suction catheter Answer: B Rationale: During suctioning of a client, negative pressure (suction) should not be applied until the catheter is ready to be drawn out because, in addition to the removal of secretions, oxygen is being depleted. (Mosby, 18th Edition) 45. Mr. Posible had a cuffed tracheostomy tube placed 4 weeks ago. The patient is going to begin eating by mouth, with the tracheostomy tube in place. To prevent aspiration, Nurse Edu will: 1. Raise the head of the bed to high-Fowler's position 2. Deflate the cuff on the tracheostomy tube 3. Suction the client before eating 4. Assess gag and swallow ability 5. Replace the tracheostomy tie A. All except 2 C. All except 5 B. All except 3 D. All of the above Answer: C Rationale: Number 1 is correct because by raising the head of the bed, gravity helps the client

East West Nursing Review Center

swallow and helps prevent aspiration. Number 2 is correct because the cuff should initially be deflated to assess gag and swallowing ability. In addition, some clients find it more difficult to swallow with the cuff inflated. Thus, option A is incorrect. Number 3 is correct because the client's airway should be patent, and secretions removed before deflating the cuff. Thus, option B is incorrect Number 4 is correct because assessing the client's gag and swallowing ability is crucial before starting the client on PO food and fluids. Numbers 5 is incorrect because it is not necessary to replace the tracheostomy ties before the client eats, unless they are noted to be soiled or loose. Thus, making option C the correct answer and option D incorrect. (Davis's NCLEX RN Success, 2nd Edition) Situation 10. Noynoy Shananga, 58-year-old, is admitted to the medical-surgical unit of a tertiary hospital with complains of alternating diarrhea and constipation, weight loss, abdominal distention, and frank blood in the stool. Diagnostic tests reveal cancer of the colon and creation of a colostomy was performed. 46. During a colostomy irrigation, if Mr. Shananga complains of abdominal cramps, Nurse Mar Kit should: A. Clamp the tubing and allow the patient to rest B. Reassure the patient and continue the irrigation C. Pinch the tubing so that less fluid enters the colon D. Raise the irrigating container to complete the irrigation quickly Answer: A Rationale: Rapid instillation of fluid into the colon may cause abdominal cramps. By clamping the tubing, the nurse allows the cramps to subside so the irrigation can be continued. Option B is incorrect because emotional support will not interrupt the physical adaptation to abdominal (Mosby, 18th Edition) 47. Mr. Shananga is concerned about the odor of the stool in the ostomy drainage bag. Nurse Mar Kit teaches the patient to include which of the following foods in the diet to reduce odor? A. Cucumbers C. Yogurt B. Broccoli D. Eggs Answer: C Rationale: The client should be taught to include deodorizing foods in the diet, such as beet greens, parsley, buttermilk, and yogurt. Spinach also reduces odor, but is a gas-forming food as well. (Saunders, 2nd Edition) 48. When discussing the regaining of bowel control with Mr. Shananga who has had surgery for a colostomy, which is most important? A. High-protein diet C. Managing fluid intake B. Irrigation routine D. Soft, low-residue diet Answer: B Rationale: Colostomy irrigations done daily at the same time help establish a regular pattern of bowel evacuation. (Mosby, 18th Edition) 49. Mr. Shananga tells Nurse Mar Kit that his wife does not let him change his colostomy bag himself. Which response by the nurse indicates an understanding of the situation? A. Your wife's need to help you is a reality you should accept. B. Do you think your wife might benefit from counseling? C. You feel you need privacy when changing your colostomy? D. Have you discussed the situation with your doctor? Answer: C Rationale: This type of communication technique, making an observation, enables the nurse to acknowledge that something exists or has changed in some way. This acknowledgement made by the nurse should open communication with the client. (NSNA NCLEX-RN Review, 4th Edition) 50. An adhesive-backed ostomy opening should be how much larger than the stoma? A. inch B. inch C. 1/3 inch D. 1/8 inch Answer: D Rationale: In general, the opening should be 1/8 inch larger than the stoma itself. An opening that fits too tightly can injure the stoma. (Fundamentals of Nursing: Made Incredibly Easy!, 2007) Situation 11. Nurse Loren is taking care of Mr. Money Billiards, a 35-year-old businessman with acute renal failure, who is scheduled for hemodialysis. 51. Mr. Billiards is placed on hemodialysis three times a week. Which is an attainable short-term goal for this patient when he is placed on hemodialysis? A. Understanding the treatment and its implications B. Independence in the care of the AV shunt C. Self-monitoring during dialysis D. Recording dialysate composition and temperature Answer: A

East West Nursing Review Center

Rationale: Prior to the start of dialysis, the client should fully comprehend its meaning and the changes in lifestyle required. (NSNA NCLEX-RN Review, 4th Edition) 52. Mr. Billiards has an arteriovenous fistula. Which finding is expected when assessing the fistula? A. Ecchymotic area C. Pulselessness B. Enlarged veins D. Redness Answer: B Rationale: The leaking of arterial blood into an AV fistula causes the veins to enlarge so they are easier to access for hemodialysis. An AV fistula requires 4 to 6 weeks to mature before it can be used. Peritoneal dialysis or external shunts may be used while the fistula is maturing. (NSNA NCLEX-RN Review, 4th Edition) 53. When caring for Mr. Billiards who has had an arteriovenous shunt inserted for hemodialysis, Nurse Loren should: A. Cover the entire cannula with an elastic bandage B. Notify the physician if a bruit is heard in the cannula C. Use surgical aseptic technique when giving shunt care D. Take the blood pressure every 4 hours, using the arm that contains the shunt Answer: C Rationale: Insertion of an arteriovenous shunt represents a break in the first line of defense against infection, the skin. An infection of an arteriovenous shunt can be avoided by strict aseptic (sterile) technique. (Mosby, 18th Edition) 54. Nurse Loren is monitoring Mr. Billiards who is undergoing hemodialysis. The patient suddenly becomes cyanotic and complains of dyspnea and chest pain. His blood pressure is 70/40 mm Hg and his pulse is weak and rapid. The nurse calls the physician immediately because the signs and symptoms suggest which of the following complications of dialysis? A. Disequilibrium syndrome B. Air embolism C. Internal bleeding D. Hemorrhage at the shunt Answer: B Rationale: Air embolism is a potentially fatal complication characterized by sudden hypotension, dyspnea, chest pain, cyanosis, and weak, rapid pulse. (NSNA NCLEX-RN Review, 4th Edition) 55. Before discharge, Nurse Loren discusses care at home with Mr. Billiards and his wife. The nurse recognizes that further teaching is required when the wife says: A. I must touch the shunt several times a day to feel for the bruit. B. I have to take his blood pressure every day in the arm with the fistula. C. He will have to be very careful at night not to lie on the arm with the fistula. D. We really should check the fistula every day for signs of redness and swelling. Answer: B Rationale: Taking the blood pressure in the affected arm could injure the fistula. Option A is incorrect because the presence of a bruit indicates the circulation is not obstructed by a thrombus. (Mosby, 18th Edition) Situation 12. Nurse Jason is taking care of Mr. Freddie Ivler, 30 years old, with chronic renal failure, who is scheduled for peritoneal dialysis. 56. The purpose of peritoneal dialysis is to: A. Reestablish kidney function B. Clean the peritoneal membrane C. Provide fluid for intracellular spaces D. Remove toxins and metabolic wastes Answer: D Rationale: Peritoneal dialysis uses the peritoneum as a selectively permeable membrane for diffusion of toxins and wastes from the blood into the dialyzing solution. (Mosby, 18th Edition) 57. When caring for Mr. Ivler who is receiving peritoneal dialysis, Nurse Jason should: A. Position the patient from side to side if fluid is not draining adequately B. Notify the physician if there is a deficit of 200 ml in the drainage fluid C. Maintain the patient in a flat, supine position during the entire procedure D. Remove the cannula at the end of the procedure and apply a dry, sterile dressing Answer: A Rationale: If fluid is not draining adequately, the client should be positioned from side to side or with the head raised; or manual pressure should be applied to the lower abdomen to facilitate drainage by using external pressure and gravity. Mosby, 18th Edition) 58. When assessing Mr. Ivler during peritoneal dialysis, Nurse Jason observes that drainage of the dialysate from the peritoneal cavity has ceased before the required amount has drained out. The nurse should assist the patient to:

East West Nursing Review Center

A. Drink 8 oz of water C. Deep breathe and cough B. Turn from side to side D. Periodically rotate the catheter Answer: B Rationale: Turning from side to side will change position of the catheter, thereby freeing the drainage holes, which may be obstructed. (Mosby, 18th Edition) 59. Nurse Jason is completing the exchange by draining the dialysate and notices the dialysate is opaque. The nurse best interprets this finding as: A. The normal appearance of draining dialysate B. A sign of infection C. An indication of an impending lower back problem D. A sign of a vascular access occlusion Answer: B Rationale: Peritonitis is usually caused by Staphylococcus. The first indication of peritonitis is opaque or cloudy dialysate. (NSNA NCLEX-RN Review, 4th Edition) 60. Nurse Jason suspects a complication in Mr. Ivler who is receiving peritoneal dialysis. Which of the following observations would support this evaluation? A. Pain during the inflow of dialysate B. Occasional diarrhea C. Cloudy effluent D. Clear or light yellow effluent Answer: C Rationale: The major complication of peritoneal dialysis is peritonitis. Cloudy or opaque effluent is the earliest sign of peritonitis. Other signs of infection include fever, rebound abdominal tenderness, malaise, nausea, and vomiting. (NSNA NCLEX-RN Review, 4th Edition) Situation 13. Ms. Chica Go, a 40-year-old patient with esophageal cancer, is to receive total parenteral nutrition. A right subclavian catheter is inserted by the physician. 61. Nurse Archipela knows that the primary reason for using a central line is that: A. It prevents the development of phlebitis B. There is less chance of this infusion infiltrating C. It is more convenient so clients can use their hands D. The large amount of blood helps to dilute the concentrated solution Answer: D Rationale: Unless diluted, the highly concentrated solution can cause hyperosmolar diuresis. (Mosby, 18th Edition) 62. Nurse Archipela is preparing to change the total parenteral nutrition (TPN) solution bag and tubing. The nurse asks Ms. Go to do which of the following most essential action during the tubing changes? A. Take a deep breath, hold it, and bear down B. Exhale slowly and evenly C. Turn the head to the right D. Breathe normally Answer: A Rationale: The client should be asked to perform Valsalva maneuver during tubing changes. This helps to avoid air embolism during tubing changes. The nurse asks the client to take deep breath, hold it, and bear down. (Silvestri, Saunders Comprehensive Review for the NCLEX-RN Examination, 3rd Edition) 63. Nurse Archipela is changing the central line dressing of Ms. Go who is receiving total parenteral nutrition (TPN). The nurse notes that the catheter insertion site appears reddened. The nurse next assesses which of the following? A. Tightness of tubing connections B. Clients temperature C. Expiration date of the bag D. Time of last dressing change Answer: B Rationale: Redness at the catheter insertion site is a possible indication of infection. The nurse would next assess for other signs of infection. Of the options given, the temperature is the next item to assess. (Silvestri, Saunders Comprehensive Review for the NCLEX-RN Examination, 3rd Edition) 64. Ms. Go with total parenteral nutrition (TPN) infusing has disconnected the tubing from the central line catheter. Nurse Archipela assesses the client and suspects an air embolism. The nurse should immediately place the patient in which of the following positions? a. On the left side with the head higher than the feet b. On the left side with the head lower than the feet c. On the right side with the head higher than the feet d. On the right side with the head lower than the feet Answer: B

East West Nursing Review Center

Rationale: When air embolism is suspected, the client should be placed in a left side-lying position. The head should be lower than the feet. This position is used to try to minimize the effect of the air traveling as a bolus to the lungs by trapping it in the right side of the heart. (Silvestri, Saunders Comprehensive Review for the NCLEX-RN Examination, 3rd Edition) 65. Ms. Go is being weaned from total parenteral nutrition (TPN) and is expected to being taking solid food today. The ongoing solution rate has been 100 ml/hr. Nurse Archipela anticipates that which of the following orders regarding the TPN solution will accompany the diet order? a. Discontinue the TPN b. Continue current infusion rate orders for TPN c. Decrease TPN rate to 50 ml/hr d. Hang 1000 ml 0.9% normal saline Answer: C Rationale: When a client begins taking a diet after a period of receiving parenteral nutrition, the TPN is decreased gradually. Total parenteral nutrition that is discontinued abruptly (option A) can cause hypoglycemia. Clients often have anorexia after being without food for some time, and the digestive tract also is not used to producing the digestive enzymes that will be needed. Gradually decreasing the infusion rate allows the client to remain adequately nourished during the transition to a normal diet and prevents the occurrence of hypoglycemia. Even before clients are started on a solid diet, they are given clear liquids followed by full liquids to further ease the transition. (Silvestri, Saunders Comprehensive Review for the NCLEX-RN Examination, 3rd Edition) Situation 14. Using Maslows need theory, Airway, Breathing and Circulation are the physiological needs vital to life. The nurses knowledge and ability to identify and immediately intervene to meet these needs is important to save lives. 66. Which of these patients has a problem with transport of oxygen from the lungs to the tissues? A. Carol with a tumor in the brain B. Patria with anemia C. Jimson with fracture in the femur D. Kenny with diarrhea Answer: B Rationale: A person with anemia may actually have decreased oxygen carriers (RBC containing Hgb) in the blood. The RBC component of blood serves as the oxygen carrying media that diffuses into it from the pulmonary capillaries. 67. You noted from the lab exams in the chart of Tado, a 25-year-old patient, that he has reduced oxygen in the blood. This condition is called: A. Cyanosis C. Hypoxemia B. Hypoxia D. Anemia Answer: C Rationale: Hypoxemia is a state of reduced oxygen in the blood. 68. You will do nasopharyngeal suctioning on Hermes, 30 years old. Your guide for the insertion of the tubing for an adult would be: A. Tip of the nose to the base of the neck B. The distance from the tip of the nose to the middle of the cheek C. The distance from the tip of the nose to the earlobe D. Eight to ten inches Answer: C Rationale: The guide for insertion of the tubing for an adult for nasopharyngeal suctioning is: Measure the distance between the tip of the clients nose and the earlobe, or about 13cm (5inches). 69. While doing nasopharyngeal suctioning on Hermes, the nurse can avoid trauma to the area by: A. Apply suction for at least 20-30 seconds each time to ensure that all secretions are removed B. Using gloves to prevent introduction of pathogens to the respiratory system C. Applying no suction while inserting the catheter D. Rotating the catheter as it is inserted with gentle suction Answer: C Rationale: Without applying suction, insert the premeasured catheter, or the prescribed length into the naris, and advance it along the floor of the nasal cavity. 70. Cath, 21 years old, has difficulty of breathing when on her back and must sit upright in bed in order to breathe effectively and comfortably. The nurse documents this condition as: A. Apnea C. Dyspnea B. Orthopnea D. Tachypnea Answer: B Rationale: Orthopnea is an abnormal condition in which a person must sit of stand to breathe deeply or comfortably, occurring in many disorders of the cardiac and respiratory systems. Situation 15. Nurse Jhang witnesses a vehicular accident near the hospital where she works. She decides to get involved and help the victims of the accident. 71. Her priority nursing action would be to: A. Assess damage to property B. Assist in the police investigation since she is a witness C. Report the incident immediately to the local police authorities D. Assess the extent of injuries incurred by the victims of the accident

East West Nursing Review Center

Answer: A Rationale: Standard operating procedure in responding to disasters would be: 1st, survey the scene if it is safe, 2nd, call for help, 3rd, begin primary survey, 4th begin secondary survey. For multiple victims, do triage simultaneously with primary survey. 72. Priority attention should be given to which of these patients? A. Bryan who shows severe anxiety due to trauma of the accident B. Anton who has chest injury, is pale and with difficulty of breathing C. Chris who has laceration on the arms with mild bleeding D. Joseph whose left ankle swelled and has some abrasions Answer: B Rationale: The use of ABC principle and Maslows Hierarchy of needs would indicate that B is the best answer. Anxiety due to trauma is not a physiologic need which requires greater priority. Having lacerations on the arms with mild bleeding and having a swollen ankle and some abrasions may be a part of physiologic aspect, but they are not as critical as a patient with chest injury, who is pale and has dyspnea. Using the principle of ABC, airway deserves the greatest priority. 73. In the emergency room, Nurse Jhang is assigned to attend to the patient with lacerations on the arms. While assessing the extent of the wound the nurse observes that the wound is now starting to bleed profusely. The most immediate nursing action would be to: A. Apply antiseptic to prevent infection B. Clean the wound vigorously of contaminants C. Control and reduce the bleeding of the wound D. Bandage the wound and elevate the arm Answer: D Rationale: According to the Red Cross guidelines for first-aid, management for bleeding will involve the application of direct pressure only. The application of direct pressure, elevation and pressure points is the old protocol in bleeding management. Bandaging can cause pressure application and elevation of the affected part wound will reduce blood flow to the area. This is the most appropriate intervention. Option C is not an intervention but a nursing goal. 74. Nurse Jhang applies pressure dressing on the bleeding site. This intervention is done to: A. Reduce the need to change dressing frequently B. Allow the pus to surface faster C. Protect the wound from microorganisms in the air D. Promote homeostasis Answer: D Rationale: For active bleeding in emergency situations, it is vital that the nurse or the trained responder applies direct pressure over the bleeding site for the main purpose of preventing further bleeding and even at times promote homeostasis. The application of direct pressure over the bleeding site will not allow the pus to surface faster nor will it protect the wound from microorganisms in the air. The reduction of the need to change dressing is but just a secondary effect of the said intervention. 75. After the treatment, the patient is sent home and asked to come back for follow-up care. Your responsibilities when the patient is to be discharged include the following, except: A. Encouraging the patient to go to the out patient clinic for follow up care B. Accurate recording of treatment done and instructions given to the patient C. Instructing the patient to see you after discharge for further assistance D. Providing instructions regarding wound care Answer: C Rationale: Discharge includes obtaining order for discharge, planning continuity of care, providing patient teaching, performing a final assessment, caring for personal property, performing business functions, and documentation. Instructing the client to see the nurse privately after discharge for further assistance is not a responsibility. Situation 16. Regardless of the prescribed solution, the nurse prepares the solution for administration, performs a venipuncture, regulates the rate of administration, monitors the infusion, and discontinues the administration when fluid balance is restored. 76. The physician is going to order a hypotonic intravenous solution for a patient with cellular dehydration. Nurse Vince would expect which of the following fluids to be administered? A. 0.9% normal saline B. 5% dextrose in normal saline C. Lactated Ringer's D. 0.45% sodium chloride Answer: D Rationale: 0.45% normal saline is a hypotonic solution that draws fluid from the vascular compartment into the cells (Hogan, Prentice Hall Reviews and Rationales Series for Nursing: Fundamentals of Nursing)

77. IV orders for a 75-year-old patient include D5 NS with 20 mEq KCl at 75 cc/hr. At 6:00 A.M. a new
bag of 1000 ml was hung. At 8:00 A.M. Nurse Vince sees that only 200 ml remains. The first action by the nurse would be to: A. Check the site for infiltration and elevate the extremity if needed B. Order an infusion pump to provide better control of the rate C. Slow the IV to keep open and check the patient for lung crackles

East West Nursing Review Center

D. Assist the patient to the bathroom to void Answer: C Rationale: The first priority is to assess the effect on an elderly client from the rapid infusion of 800 ml of fluid. The older adult is at greater risk for signs and symptoms of volume overload (shortness of breath, crackles, and decreased oxygen saturation) particularly with a history of heart failure. (Davis's NCLEX RN Success, 2nd Edition) 78. Potassium chloride is to be added to an infant's intravenous fluids. Before adding this electrolyte, Nurse Vince should determine that: A. The infant has voided recently B. Moro reflex is present C. Respiratory rate is between 25 and 40 D. Mucous membrane is moist Answer: A Rationale: Before adding potassium chloride to any IV line, the nurse must check that the client's kidneys are functioning to avoid potassium overload and hyperkalemia. (Davis's NCLEX RN Success, 2nd Edition)

79. A patient admitted for decreased level of consciousness and dehydration is receiving an IV of D5
NS with 40 mEq KCl/L infusing at 100 cc/hr. The patient's potassium level is 5.9 mEq/L. Nurse Vince knows to: A. Recheck the potassium level 6 hours after the blood is drawn in the morning B. Stop the IV, maintain the site, notify the physician about the lab level C. Decrease the IV rate to 50 cc/hr and notify the physician when the physician is on rounds D. Assess the IV site and continue with the current order Answer: B Rationale: A normal potassium level is 3.5 to 5.5 mEq/L and the client has a high potassium level of 5.9 mEq/L with potassium continuing to infuse in the IV fluid. The nurse should stop the IV, maintain the IV site by flushing the IV with normal saline or heparin per agency protocol, and notify the physician immediately for a change in IV fluid order. (Davis's NCLEX RN Success, 2nd Edition) 80. When preparing an IV piggyback medication for a patient, Nurse Vince is aware that it is essential to: A. Use strict sterile technique B. Rotate the bag after adding the medication C. Use exactly 100 ml of fluid to mix the medication D. Change the needle just before adding the medication Answer: A Rationale: Because IV solutions enter the body's internal environment, all solutions and medications utilizing this route must be sterile to prevent the introduction of microbes. (Mosby, 18th Edition) Situation 17. Blood is collected, stored, and checked for safety and compatibility before it is administered as a transfusion. 81. Which nursing intervention is appropriate for Nurse Yuri to take when setting up supplies for Toni, age 45, who requires a blood transfusion? A. Add any needed IV medication in the blood bag within one half hour of planned infusion B. Obtain blood bag from laboratory and leave at room temperature for at least one hour prior to infusion C. Prime tubing of blood administration set with normal saline solution, completely filling filter D. Use a small-bore catheter to prevent rapid infusion of blood products that may lead to a reaction Answer: C Rationale: The tubing is primed with normal saline solution (0.9% NS solution). If the filter is not completely primed, debris will coagulate in the filter and the transfusion will be slowed. In addition, saline is prepared to infuse in case a transfusion reaction occurs. (NSNA NCLEX-RN Review, 4th Edition) 82. The physician orders 3 units of whole blood for Mr. Otol, age 51, who is in hypovolemic shock after a gastrointestinal hemorrhage. When administering blood, Nurse Yuri first verifies the type and crossmatch and then routinely: A. Warms the blood to body temperature to prevent chills B. Uses an infusion pump to increase the accuracy of the infusion C. Draws blood samples from the patient before and after each unit is transfused D. Runs the blood at a slower rate during the first 10 to 15 minutes of the transfusion Answer: D Rationale: A slow rate provides time to recognize a transfusion reaction that is developing before too much blood has been administered. (Mosby, 18th Edition) 83. Princess, age 30, requires a blood transfusion. Her blood type is AB. To prevent complications of blood incompatibilities, Nurse Yuri knows that this patient may receive:

East West Nursing Review Center

A. Type A or B blood only C. Type O blood only B. Type AB blood only D. Either type A, B, AB, or O blood Answer: D Rationale: Persons with type AB blood, because they are universal blood recipients, are able to receive either type A, B, AB, or O blood. People with any blood type other than AB (type A, B and O blood), are restricted as to the type of blood they can receive. (NSNA NCLEX-RN Review, 4th Edition) 84. Nurse Yuri is preparing to administer two units of packed red blood cells to Bianca, age 34. The nurse should: A. Prime the blood administration tubing with 3% saline solution B. Administer the prescribed intravenous drugs through the blood administration tubing to ensure proper distribution throughout the body C. Understand that blood transfusion reactions usually do not occur until the client has received 500 ml of the packed red blood cells D. Use a special blood filter and an 18-gauge needle to administer the packed red blood cells Answer: D Rationale: The large bore needle prevents lysis of the red blood cells. The special blood filter prevents emboli or contaminated matter from flowing into the bloodstream. (NSNA NCLEX-RN Review, 4th Edition) 85. Mariel, 50 years old, is receiving 5% dextrose in 0.45% saline. The physician has ordered the patient receive one unit of packed red blood cells. Prior to hanging the blood, Nurse Yuri will prime the blood tubing with which of the following solutions? A. 3% saline solution C. 0.9% sodium chloride B. Lactated Ringer's D. 5% dextrose in 0.45% sodium chloride Answer: C Rationale: 0.9% sodium chloride (normal saline) is the only solution that can be administered with blood or blood products. Other solutions may cause the blood cells to clump or cause clotting. Thus, options A, B and D are incorrect. (Hogan, Prentice Hall Reviews and Rationales Series for Nursing: Fundamentals of Nursing) Situation 18. When evaluating ABG results to determine acid-base balance, it is important to use a systematic approach. Nurses need to assess each measurement individually, and then look at the interrelationships to determine what type of acid-base imbalance may be present. 86. Nurse Charitie understands that metabolic acidosis develops in kidney failure as a result of: A. Inability of the renal tubules to secrete hydrogen ions and conserve bicarbonate B. Inability of the renal tubules to reabsorb water to dilute the acid contents of blood C. Depression of respiratory rate by metabolic wastes causing carbon dioxide retention D. Impaired glomerular filtration causing retention of sodium and metabolic waste products Answer: A Rationale: Bicarbonate buffering is limited, hydrogen ions accumulate, and acidosis results. (Mosby, 18th Edition) 87. A patient's blood gases reflect diabetic acidosis. Nurse Charitie should expect: A. Increased pH C. Increased PCO2 B. Decreased PO2 D. Decreased HCO3 Answer: D Rationale: The bicarbonate-carbonic acid buffer system helps maintain the pH of the body fluids; in metabolic acidosis there is a decrease in bicarbonate because of an increase of metabolic acids. (Mosby, 18th Edition)

88. Nurse Charitie is interpreting the results of a blood gas analysis performed on an adult patient. The
values include pH of 7.35, PCO2 of 60, HCO3 of 35, and O2 of 60. Which interpretation is most accurate? A. The patient is in metabolic acidosis B. The patient is in compensated metabolic alkalosis C. The patient is in respiratory alkalosis D. The patient is in compensated respiratory acidosis Answer: D Rationale: A pH of 7.35 is on the acid side of normal. All of the other values are abnormal so the client is compensated. The CO2 is sharply elevated and will lower the pH. The HCO3 is also elevated and is responsible for bringing the pH up to the normal range. An abnormal O 2 suggests that the problem is a respiratory one. (NSNA NCLEX-RN Review, 4th Edition) 89. An adult woman is admitted with metabolic acidosis. Which set of arterial blood gases should Nurse Charitie expect to find in a client with metabolic acidosis? A. pH - 7.28; PCO2 - 55; HCO3 - 26 B. pH - 7.50; PCO2 - 40; HCO3 - 31 C. pH - 7.48; PCO2 - 30; HCO3 - 22 D. pH - 7.30; PCO2 - 36; HCO3 - 18 Answer: D Rationale: The pH is below the normal range of 7.35 to 7.45. The PCO2 is within the normal range of 35 to 45 mm Hg and the HCO3 is below the normal limits of 21 to 28 mEq/L. These values indicate a metabolic problem because the PCO2 is within normal limits. And the problem is acidosis

East West Nursing Review Center

because the pH is below normal. This is uncompensated metabolic acidosis because both the HCO 3 level and the pH are below normal. (NSNA NCLEX-RN Review, 4th Edition)

90. The patient's arterial blood gases on room air are: pH of 7.33; PO2 of 77; PCO2 of 50; and HCO3 of
23. Nurse Charitie instructs the patient to: A. Try to breathe more slowly B. Use the bedside inspirometer hourly when awake C. Wear nasal cannula oxygen at 6 L/min D. Increase fluid intake to flush the kidneys Answer: B Rationale: The blood gases indicate an uncompensated (normal HCO3; abnormal pH and PCO2) respiratory (abnormal PCO2) acidosis (below normal pH). The client is hypoventilating. To increase ventilation, the client should increase deep breathing and activity levels to facilitate better O 2/CO2 exchange. Instructing the client to use the bedside inspirometer is an excellent tool for encouraging deep breathing and giving the client a specific measured respiratory goal 10 times per hour when awake. Walking should also be advised. (Davis's NCLEX RN Success, 2nd Edition) Situation 19. Suctioning of the airway is the mechanical aspiration of mucous secretions from the tracheobronchial tree by application of negative pressure. Appropriate nursing care is needed in the performance of this procedure. 91. A nurse is performing oropharyngeal suctioning on an unconscious patient. Which of the following actions is safe? A. Insert the catheter approximately 20 cm while applying suction B. Allow 20- to 30-second intervals between each suction, and limit suctioning to a total of 15 minutes C. Gently rotate the catheter while applying suction D. Apply suction for 5 seconds while inserting the catheter and continue for another 5 seconds before withdrawing Answer: C Rationale: Gentle rotation ensures that all surfaces are reached and prevents trauma to any one area caused by prolonged suctioning. (Hogan, Reviews and Rationales Series for Nursing: Fundamentals of Nursing) 92. An emergency tracheostomy is performed on a child with croup, and the child is receiving humidified air via a tracheostomy collar. When caring for this child, the nurse should suction the tracheostomy if the child: A. Tells the nurse of difficulty in breathing B. Becomes restless, diaphoretic, and cyanotic C. Has severe substernal retractions and stridor D. Becomes restless, pale, or the pulse increases Answer: D Rationale: These are some of the first signs of hypoxia; the airway must be kept patent to promote oxygenation. (Mosby, 18th Edition) 93. The physician orders oropharyngeal suctioning as needed for a patient in a coma. The nurse prepares to suction when assessment reveals: A. Drainage of mucus and saliva from the mouth B. The presence of a gurgling sound with each breath C. Development of cyanosis in the nail beds of the fingers D. The presence of a dry cough at increasingly frequent intervals Answer: B Rationale: The presence of secretions in the upper airway produces gurgling sounds that interfere with the free flow of air with each breath. (Mosby, 18th Edition) 94. A nurse is suctioning fluids from a patient via a tracheostomy tube. When suctioning, the nurse must limit the suctioning to a maximum of: A. 5 seconds C. 30 seconds B. 10 seconds D. 1 minute Answer: B Rationale: Hypoxemia can be caused by prolonged suctioning, which stimulates the pacemaker cells within the heart. A vasovagal response may occur, causing bradycardia. The nurse must preoxygenate the client before suctioning and limit the suctioning pass to 10 seconds. (Silvestri, Saunders Comprehensive Review for the NCLEX-RN Examination, 3rd Edition) 95. A patient with a pulmonary embolus is intubated and placed on mechanical ventilation. When suctioning the endotracheal tube, the nurse should: A. Apply suction while inserting the catheter B. Hyperoxygenate with 100% oxygen before and after suctioning C. Use short, jabbing movements of the catheter to loosen secretions D. Suction two to three times in quick succession to remove secretions Answer: B Rationale: Suctioning also removes oxygen, which can cause cardiac dysrhythmias; the nurse should try to prevent this by hyperoxygenating the client before and after suctioning. (Mosby, 18th Edition) Situation 20. Nursing is a science. It involves a wide spectrum of theoretical foundation applied in current health care situation. The nurse must use these theories in order to deliver

East West Nursing Review Center

quality health care. 96. Florence Nightingale was born in: A. Italy, May 12, 1820 C. England, May 12, 1820 B. Italy, May 12, 1840 D. England, May 12, 1840 Answer: A Rationale: Florence Nightingale, the matriarch of modern nursing, was born on May 12, 1820. Her parents, Edward and Frances Nightingale, named their daughter after her birthplace, Florence, Italy. Nightingales theory focused on the environment. She believed that disease was a reparative process; disease was natures effort to remedy a process of poisoning or decay, or a reaction against the conditions in which a person is placed. In addition, her nursing principles remain applicable today. The environmental aspects of her theory (ventilation, warmth, quiet, diet and cleanliness) remain integral components of current nursing care. (Tomey, Nursing Theorists and their Work, 5th Edition) 97. The theorist that describes nursing as a significant, therapeutic, interpersonal process, which functions cooperatively with other human processes that make health possible for individuals in communities is: A. Hildegard Peplau C. Patricia Benner B. Joyce Travelbee D. Ida Jean Orlando Answer: A Rationale: Hildegard Peplaus theory is on Interpersonal Relations. She describes nursing as a significant, therapeutic, interpersonal process. It functions cooperatively with other human processes that make health possible for individuals in communities. (Tomey, Nursing Theorists and their Work, 5th Edition) 98. The theorist whose major theme is the idea of transcultural nursing and caring is: A. Jean Watson C. Virginia Henderson B. Madeleine Leininger D. Ernestine Wiedenbach Answer: B Rationale: Madeleine Leininger is the founder of transcultural nursing and a leader in transcultural nursing and human care theory. (Tomey, Nursing Theorists and their Work, 5th Edition) 99. The theorist whose theory can be defined as the development of a science of humankind, incorporating the concepts of energy fields, openness, pattern and organization is: A. Dorothy Johnson C. Martha Rogers B. Imogene King D. Myra Levine Answer: C Rationale: Martha Rogers develop the Science of Unitary Human Beings. Rogerian nursing focuses on concern with people and the world in which they live, a natural fit for nursing care, as it encompasses people and their environments. The integrality of people and their environments, operating from a pandimensional universe of open systems, points to a new paradigm and initiate the identity of nursing as a science. (Tomey, Nursing Theorists and their Work, 5th Edition) 100. The theorist who is the proponent of the Self Care Deficit theory of nursing is: A. Betty Neuman C. Faye Glenn Abdellah B. Dorothea Orem D. Sister Callista Roy Answer: B Rationale: Dorothea Orem is the proponent of the Self Care Deficit theory of nursing. (Tomey, Nursing Theorists and their Work, 5th Edition)

COMMUNITY HEALTH NURSING Situation 1.Community Health Nursing is a unique blend of nursing and public health practice woven into a human service that properly developed and applied has a tremendous impact on human well being. 1. The primary goal of Community Health Nursing is to: A. Support and supplement the efforts of the medical professions in the promotion of health and prevention of illness B. Enhance the capacity of individuals, families and communities to cope with their heath needs C. Increase the productivity of the people D. Raise the level of health of the citizenry Answer: B Rationale: The primary goal of Community Health Nursing is to help communities and families to cope with the discontinuities of health and threats in such a way as to maximize their potential for high level wellness, as well as to promote reciprocally supportive relationship between people and their physical and social environment. (Community Health Nursing Services in the Philippines) 2. The context of Community Health Nursing is based on the nurse's evaluation about the: A. Existing health problems and needs of the people B. Current health status of the people

East West Nursing Review Center

C. Department of Heath (DOH) goals D. Devolution of health Answer: A Rationale: The context of Community Health Nursing is based on the nurse's evaluation about the existing health problems and needs of the people. One of the principles of CHN states that Community Health Nursing is based on the recognized needs of individuals, families, communities and groups. (Community Health Nursing Services in the Philippines) 3. Community Health Nursing is a specialized field of nursing that follows the basic principles in Community Development work. Which of the following statements best described Community Development as a process of empowering people in the community? A. Community development may allow women to discover and strengthen their innate capabilities to enjoy and utilize equal opportunities in all aspects of development work B. Sustainability aspects of development strategies can be taken into consideration C. Community development is solely confined to the meeting of the day to day survival of the people D. Community development is a learning process where both women and men participate to improve their lives Answer: D Rationale: Community Development is defined as an organized effort of people to improve the conditions of the community life and the capacity of the people for participation, self-direction and integrated efforts in community affairs in which development is accomplished by the people. Everyone has something to contribute to the life of the community. It is not solely for women (option A) and not a short term process (option C). (C.E Jimenez, CO-PAR) 4. If a particular health service fails, the most basic question is: A. Is this what the people demanded? B. What went wrong? C. Is this what the people need? D. Who is responsible for the failure? Answer: C Rationale: If a particular health service fails, it is important to reassess the felt needs of the people not their demands (option A). (Community Health Nursing Services in the Philippines) 5. Which of the following statement is correct? A. If people are not attending to the services offered by the health staff, the team must reassess the needs of the people B. In participatory approach, the nurse must devotedly adhere to what the people want C. In a peasant community where people are fighting for land ownership, the nurse must not participate as this is not a health concern D. Nurses must not join protest actions as nurses should always be neutral at all times Answer: A Rationale: If people are not attending to the services offered by the health staff, the team must reassess the needs of the people. This is base on the principle that Community Health Nursing is based on the recognized needs of individuals, families, communities and groups. (Community Health Nursing Services in the Philippines) Situation 2. Nurse Sid, a public health nurse, prepares a community health nursing care plan utilizing the nursing process, which is responsive to the health promotion needs of the community. 6. To obtain a 100% population count of the community, Nurse Sid should need to do a: A. Survey C. Census B. One to one interview D. Sampling of the population Answer: C Rationale: The assessment process involves the collection of relevant data. It employs various methods to collect data. A census is the procedure of systematically acquiring and recording information about the members of a given population. It is a regularly occurring and official count of a particular population. It obtains a 100% population count of the community. (Public Health Nursing in the Philippines) 7. Which of the following is a well-stated objective in a community health nursing care plan? A. To increase the number of mothers coming for prenatal check-up by 25% in 6 months time B. To increase the number of home visits by 50% C. To increase the coverage for immunization in 1 year time D. To increase the number of children receiving food assistance Answer: A Rationale: A well-stated objective should be SMART (Specific, Measurable, Attainable, Realistic and Time-Bound). 8. Identification of health risks in the community is a step in formulating a community health diagnosis. Which of the following methods Nurse Sid should do to best facilitate the identification of health risks threatening the community? A. Assess community resources and industries available B. Familiarize with the prevalent lifestyle of the people within the community C. Study health center records and reports

East West Nursing Review Center

D. Review of vital statistics available Answer: D Rationale: To facilitate identification of health risks threatening the community, the community health nurse reviews available vital statistics. Statistics refers to a systematic approach of obtaining, organizing and analyzing numerical facts so that conclusion may be drawn from them. Specifically, vital statistics refers to the systematic study of vital events such as births, illnesses, marriages, divorce, separation and deaths. The statistics of disease (morbidity) and death (mortality) indicate the state of health of a community and the success or failure of health work. (Public Health Nursing in the Philippines) 9. The community health diagnosis is an important input to the formulation of a community health nursing care plan. In order to assure a successful implementation of the care plan, the diagnosis must be carried out in a manner where: A. An outside consultant determines what data to collect B. The community is directly involved in data collection and analysis C. The nurse delegates the collection of data to the barangay health workers D. All the members of the rural health unit participates in data collection Answer: B Rationale: In community health nursing, implementation involves various nursing interventions which have been determined by the goals/objectives that have been previously set. The public health nurse carry out nursing procedures which are consistent with the nursing care plan, are adapted to present situations which promote a safe and therapeutic environment. Public health nurses involve the patient and his/her family in the care provided in order to motivate them to assume responsibility for his or their care and to be able to teach and maintain a desired level of function. Explaining and answering questions to clarify doubts, to maximize the patients confidence and ability to care for himself/themselves. Thus, the role of the community health nurses shift from direct care giver to that of a teacher. (Public Health Nursing in the Philippines) 10. Which criterion in priority setting of health problems is used only in community health care? A. Magnitude of the health problem B. Preventive potential of the health problem C. Nature of the problem presented D. Modifiability of the problem Answer: A Rationale: Magnitude of the health problem refers to the percentage of the population affected by a health problem. (Public Health Nursing in the Philippines) Situation 3.The application of the nursing process is rational method of planning and providing nursing care. As basic tool in professional nursing practice, its utilization ensures competent and safe practice. It is a scientific tool that is utilized also in Community Health Nursing. 6. Which of the following is Community Health Nursing Assessment? A. Auditing Nursing Record C. Prioritizing needs B. Monitoring health services D. Intensive fact-finding Answer: D Rationale: Assessment is the systematic and continuous collection, organization, validation, and documentation of data (information). All phases of the nursing process depend on the accurate and complete collection of data. Data collection (intensive fact-finding) is the process of gathering information about a client's health status. It must be both systematic and continuous to prevent omission of significant data and reflect a client's changing health status. (Community Health Nursing Services in the Philippines) 7. When the nurse invites other members of the nursing team to develop evaluation parameters, this process is called: A. Interpreting data C. Planning nursing action B. Tabulating data D. Putting plan of action Answer: C Rationale: In the planning phase, the nurse performs prioritization, evaluation of parameters, goal setting, plan of action construction, and development of an operational plan. (Community Health Nursing Services in the Philippines) 8. When the nurse performs appraisals, this process is called: A. Assessment C. Evaluation B. Planning D. Diagnosing Answer: C Rationale: In the evaluation phase, the nurse perform auditing and appraisal. The elements of evaluation includes: structural (inputs), process (methods), and outcome (outputs). (Community Health Nursing Services in the Philippines) 9. Once the nurse initiates contact with a client, asking questions to gather data, this process is called: A. Implementation C. Planning B. Evaluation D. Assessment Answer: D Rationale: Data collection is done during the Assessment phase. It is the process of gathering information about a client's health status. It must be both systematic and continuous to prevent

East West Nursing Review Center

omission of significant data and reflect a client's changing health status. There are 4 phases of data gathering: data collection, collation, presentation or tabulation, and analysis. (Community Health Nursing Services in the Philippines) 10. When the RHU nurse provides health teaching to individuals or families, this process is called: A. Intervention C. Assessment B. Planning D. Evaluation Answer: A Rationale: During the implementation or intervention phase, the nurse carries out interventions (such as providing health teachings) and utilizes resources. (Community Health Nursing Services in the Philippines) Situation 4. There are several factors in the ecosystem which affect the optimum level of functioning (OLOF) of individuals, families and communities. The nurse must be knowledgeable on this. 11. The modern concept of health refers to: A. How individuals maintain a maximum level of wellness B. How individuals can be called disease-free C. How individuals avoid diseases D. How individuals can avail of their immune system Answer: A Rationale: The modern concept of health refers to the optimum level of functioning (OLOF) or the maximum level of wellness of individual, families and communities. (Community Health Nursing Services in the Philippines) In the Health Care Delivery System, ideally, rehabilitation services begin: A. Upon admission of the client in the health care system B. Upon discharge of the client from the health care system C. After the client's physical condition stabilizes D. Soon after the client had requested for rehabilitation services Answer: A Rationale: In the Health Care Delivery System, ideally, rehabilitation services begin upon admission of the client in the health care system. (Community Health Nursing Services in the Philippines) 13. The factor in the ecosystem affecting the individuals health that is involved in the provision of essential health services whether community-based, accessible, sustainable and affordable is the: A. Socio-economic influences C. Behavioral B. Health Care Delivery System D. Political Answer: B Rationale: Health Care Delivery System is the factor in the ecosystem affecting the individuals health that is involved in the provision of essential health services whether community-based, accessible, sustainable and affordable. Although promotive and preventive health measures are emphasized in community health, the availability and accessibility of curative and rehabilitative services also affect people's health. (Community Health Nursing Services in the Philippines) 14. The factor in the ecosystem affecting the individual's health that is involved in the menace of pollution, basically man-made, is: A. Behavioral C. Environmental influences B. Socio-economic D. Health Care Delivery System Answer: C Rationale: Environmental influences are the factor in the ecosystem that is involved in the menace of pollution, which has been growing over the years and has greatly affected the health of the people. The disease today is largely man-made. Examples of these are communicable diseases due to poor sanitation, poor garbage collection, smoking, air pollution and utilization of chemicals such as pesticides. (Community Health Nursing Services in the Philippines) Which is not an example of behavioral influences in OLOF on health status? A. Cigarette smoking and alcohol drinking B. Exposure to toxic substances in the workplace C. Sedentary lifestyle of an office worker D. A grandmother with an adult-onset diabetes Answer: B Rationale: Behavioral influences refer to the factor in the ecosystem affecting the individuals health through certain habits that a person has. These may be in the form of smoking, intake of alcoholic drinks, substance abuse or lack of exercise. The people's lifestyle, health care and child rearing practices are shaped, to a large extent, by their culture and ethnic heritage. (Community Health Nursing Services in the Philippines) Situation 5. Public health is dedicated to the common attainment of the highest level of physical, mental and social well-being and longevity consistent with available knowledge and resources at a given time and place. 16. According to Dr. C.E. Winslow, which of the following is the goal of Public Health? A. For promotion of health and prevention and diseases B. For people to be organized in their health efforts 15. 12.

East West Nursing Review Center

C. For people to have access to basic health services D. For people to attain their birthrights and longevity Answer: D Rationale: Dr. C.E. Winslow defines public health as the science and art of preventing disease, prolonging life, promoting health and efficiency through organized community effort for the sanitation of the environment, control of communicable diseases, the education of individuals in personal hygiene, the organization of medical and nursing services for the early diagnosis and preventive treatment of disease, and the development of the social machinery to ensure everyone a standard of living adequate for the maintenance of health, so organizing these benefits as to enable every citizen to realize his birthright of health and longevity. (Public Health Nursing in the Philippines) 17. According to Dr. Margaret Shetland, the philosophy of public health nursing is based on which of the following? A. The worth and dignity of man B. The mandate of the state to protect the birthrights of its citizens C. Health and longevity as birthrights D. Public health nursing as a specialized field of nursing Answer: A Rationale: According to Dr. Margaret Shetland, the philosophy of community health nursing is based on the worth and dignity of man. (Community Health Nursing Services in the Philippines) 18. Which of the following is the most prominent feature of public health nursing? A. It involves providing home care to sick people who are not confined in the hospital B. Public health nursing focuses on preventive, not curative services C. The public health nurse functions as part of a team providing a public health nursing service D. Services are provided free of charge to people within the catchment area Answer: B Rationale: The public health nurses in this country are using their nursing skills in the application of public health functions and social assistance within the context of public health programs designed to promote health and prevent diseases. Public health nursing focuses on preventive, not curative services. (Public Health Nursing in the Philippines) 19. The public health nurse is the supervisor of rural health midwives. Which of the following is a supervisory function of the pubic health nurse? A. Referring cases or patients to the midwife B. Providing technical guidance to the midwife C. Proving nursing care to cases referred by the midwife D. Formulating and implementing training programs for midwives Answer: B Rationale: Generally, the public health nurse is the supervisor of the midwives and other auxiliary health workers in the catchment area. This is in accordance with agencys policies and in a manner that improves performance and promotes job satisfaction. During the visit, the public health nurse identifies together with the supervisee any issue or problem encountered and addresses them accordingly. If it is a technical matter like a breach in the procedure or established protocol, coaching is immediately instituted. Option D is incorrect because if problems or issues identified needs further capacity enhancement or training for the supervisee, then the nurse arranges, not formulate or implement, for the conduct of this training. (Public Health Nursing in the Philippines) Qualifications to be a public health nurse includes which of the following: 1. Good physical and mental health 2. BSN graduate 3. Registered nurse 4. Masters degree in Nursing 5. 3 years experience as a Clinical Instructor A. 1, 2 and 3 C. 1, 4 and 5 B. 1, 3 and 4 D. 2, 3 and 4 Answer: A Rationale: The Standards of Public Health Nursing in the Philippines developed by the National League of Philippine Government Nurses in 2005 described the qualification and functions of a Public Health Nurse.The PHN has the professional, personal and other qualifications that are appropriate to his/her job responsibilities. They are as follows: is a graduate of BSN (2) and a Registered Nurse (3); has the following personal qualities and professional competencies such as good physical and mental health (1), interest and willingness to work in the community, with leadership potential, resourcefulness and creativity, honesty and integrity, active membership to professional nursing organizations. (Public Health Nursing in the Philippines) Situation 6. Primary Health Care as an approach to delivery of health care services 21. Which one is the goal of Primary Health Care in the Philippines? A. Reorientation and reorganization of the national health care system with the establishment of functional support mechanism B. Essential health care made universally accessible, acceptable, available, and affordable to all C. Health for all Filipinos and health in the hands of the people by the year 2020 D. To strengthen the health care system, let the people manage their own health care Answer: C Rationale: The goal of Primary Health Care in the Philippines is Health for all Filipinos and health in the 20.

East West Nursing Review Center

hands of the people by the year 2020. (Public Health Nursing in the Philippines) 22. The mission of Primary Health Care refers to which of the following? A. To strengthen the health care system, let the people manage their own health care B. Guarantee equitable, sustainable and quality health for all Filipinos, especially the poor, and shall lead the quest for excellence in health C. Health for all by the year 2020 D. Health for all Filipinos and health in the hands of the people by the year 2020 Answer: A Rationale: The mission of Primary Health Care in the Philippines is To strengthen the health care system, let the people manage their own health care. 23. The law that provided mandate for the implementation of Primary Health Care in the Philippines is: A. R.A. 7160 C. R.A. 8423 B. LOI 949 D. R.A. 9255 Answer: B Rationale: Primary Health Care was declared during the First International Conference on Primary Health Care held in Alma Ata, USSR on September 6-12, 1978 by WHO. The goal was Health for All by the year 2000. This was adopted in the Philippines through Letter of Instruction (LOI) 949 signed by President Marcos on October 19, 1979 and has an underlying theme of Health in the hands of the people by 2020. Which of the following is not a corner stone of Primary Health Care? A. Support mechanisms made available B. Active community participation C. Intra and intersectoral solicitation linkages D. Use of appropriate technology Answer: C Rationale: The four corner stones or pillars in PHC are as follows: active community participation (option B); intra and inter-sectoral linkages (option C); use of appropriate technology (option D); and support mechanism made available (option A). 25. Which does not describe Primary Health Care? A. It emphasizes partnership between health care providers and the people B. It is a total approach to community development C. It stresses the importance of linkages D. It aims to provide free health services to the people Answer: D Rationale: Option D is incorrect because PHC aims to provide affordable, not free, health services to the people. (Public Health Nursing in the Philippines) Situation 7.Clinic Visit is done at the health center or health station so that the community health nurse can provide the necessary health care services to the people in the community. On the other hand, home visit is a family-nurse contact which allows the health worker to assess the home and family situations in order to provide necessary nursing care. 26. During clinic visit, which among the following activities is done during pre-consultation conference? A. One-on-one counseling C. First come, first served queuing B. Record preparation D. Pre-clinic lecture Answer: D Rationale: Pre-clinic lecture is usually done during pre-consultation conference that is conducted prior to the admission of patients, which is one way of providing health education. (Public Health Nursing in the Philippines) 27. Certain DOH programs utilize an acceptable decision to which the nurse has to follow. What should the public health nurse do to a program-based case? A. Manage the case C. Provide first aid treatment B. Refer to the physician D. Refer the case to the next level of care Answer: A Rationale: In triaging, a public health nurse manages a program-based case. Certain programs of the DOH like the IMCI utilize an acceptable decision to which the nurse has to follow in the management of a simple case. All non-program based cases are refer to the physician (option B). All emergency cases are provided with first aid treatment and refer the case to the next level of care (option C and D). (Public Health Nursing in the Philippines) 28. Clinic visit is being executed by a health team. Who acts as a leader in planning the clinical activities? A. Barangay Health Worker C. Public Health Nurse B. Physician-in-Charge D. Rural Health Midwife Answer: B Rationale: The Physician-in-Charge is the leader in planning the clinical activities. (Public Health Nursing in the Philippines) 29. The following best describes a home visit, except: A. May or may not be recorded 24.

East West Nursing Review Center

B. Extension of the services of the health center C. A professional contact made by the nurse D. Should have an objective Answer: A Rationale: Home visit is a professional contact made by the nurse and an extension of the services of the health center. When preparing for a home visit, it should have a purpose or objective. Home visit should always be recorded or documented (making option C incorrect). (Public Health Nursing in the Philippines) 30. Planning for a home visit is an essential tool in achieving best results in health care. The following are principles in a home visit, except: A. Planning should be flexible and practical B. Home visit should have a purpose C. Plans are based in available information including those from other agencies that may have rendered services to the family D. Planning of continuing care must be developed by the nurse Answer: D Rationale: The following are principles of home visits: A home visit must have a purpose or objective (option) Situation 8. The following procedures are very important to the public health nurse in rendering effective nursing care to clients in varied settings. 36. The nurse should understand which rationale when performing the bag technique? A. It should not overshadow the concerns for the client B. It should render effective nursing care to clients or other family members C. It should minimize or prevent the spread of infection D. It should save time and effort when performing nursing procedures Answer: B Rationale: The rationale when performing the bag technique is that it should render effective nursing care to clients or other family members. (Community Health Nursing Services in the Philippines) 37. Tourniquet test or Rumpel-Lead's test is used as screening for dengue. If the nurse counted 10 petechial spots in the imaginary one square inch just below the BP cuff or in the antecubital fossa, this should be interpreted as: A. Positive C. Alarming B. Negative D. Warning Answer: B Rationale: Tourniquet test or Rumpel-Lead's test is used as screening for dengue. An imaginary one square inch just below the BP cuff or in the antecubital fossa is made to check for the presence of petechial rashes. If it manifested 20 or more petechial rashes, it indicates a positive dengue result. If it is less than 20 it means that the test is negative. (Community Health Nursing Services in the Philippines) 38. The nurse utilizes three different agents when wiping the thermometer after use. Arrange them according to sequence: A. 3x soap, 3x water, 3x alcohol C. 3x soap, 3x water, 1x alcohol B. 1x soap, 1x water, 1x alcohol D. 1x soap, 1x water, 3x alcohol Answer: C Rationale: When wiping the thermometer after use, clean the thermometer in a downward spiral motion from the stem to the bulb, holding it over the waste paper bag using the following technique: 1st - 3 cotton balls moistened with soap. Discard. 2Nd - 3 cotton balls moistened with water. 3Rd - 1 cotton ball moistened with alcohol, then wrap around the bulb of the thermometer and lay it inside the kidney basin. (Community Health Nursing Services in the Philippines) 39. An orange result of a Benedict's test should be interpreted by the nurse as to which extent of glucosuria? A. (-) B. + C. ++ D. +++ Answer: D Rationale: An orange result of a Benedict's test indicates a+++ result. 40. I the result of a Heat and Acetic acid test is clear, the nurse knows that: A. Albuminuria is abnormal B. Protein is traced in the urine C. This is a positive result of protein leak D. There is no albumin in the urine Answer: D Rationale: If the result of a Heat and Acetic acid test is clear, there is no albumin in the urine. If it is cloudy, the nurse should repeat the procedure. If on the second test, the result is still cloudy then it is interpreted as positive protein leak or positive proteinuria. (Community Health Nursing Services in the Philippines) Situation9. COPAR (Community Organizing Participatory Action Research) recognizes people's participation as a tool for community development. 41. Arrange the activities of community organizing as a continuous sustained process for community development. A. Organization, Education, Mobilization B. Organization, Mobilization, Education C. Education, Organization, Mobilization

East West Nursing Review Center

D. Education, Mobilization, Organization Answer: C Rationale: Community organizing as a continuous sustained process for community development involves the process of Education, Organization and Mobilization. (Community Health Nursing Services in the Philippines) 42. The following statements pertain to Community Organizing, except: A. Its goal is community development B. A never-ending process once started C. Can apply for increasing awareness D. A process for increasing organization and initiating responsible action Answer: B Rationale: Community Organizing is a process by which people, health services and agencies of the community are brought together to learn about the common problems, identify these problems as their own, plan the kind of action needed to solve these problems and act on this basis. Its goal is community development (option A). It can be applied to increase the awareness of the community (option C). It is also a process for increasing organization and initiating responsible action (option D). Option B is incorrect because it sets up action pattern to solve problems, not a never-ending process once started. (Community Health Nursing Services in the Philippines) 43. Which among the following does not pertain to Participatory Action Research? A. Problem identification involves the community or group experiencing the problem B. Method of data gathering is determined by local culture and innovativeness C. Use of research results is within the full control of the people D. It is done by an outsider utilizing technical quantitative techniques for publication use Answer: D Rationale Option D is incorrect because it is not done by an outsider. (Community Health Nursing Services in the Philippines) 44. Manageable units of the community to facilitate service delivery and people's participation is called the: A. Core group C. Spot map B. Small group D. Organizing group Answer: A Rationale: Core group is the manageable units of the community to facilitate service delivery and people's participation. (Community Health Nursing Services in the Philippines) 45. The basic reasons why community organizers need to phase out from the community is to enable the: A. People to exercise self-reliance B. Nurse to open community organization work in other depressed communities C. People's organization to expand their coverage D. People to test their unity and strength Answer: A Rationale: The reason why community organizers need to phase out from the community is to enable the people to exercise self-reliance. People are given a chance to study their problems, offer solutions and give a chance to plan an action. (Community Health Nursing Services in the Philippines) Situation 10. Community Organization is a process wherein people in the community, health care providers and agencies in the community are brought together. 46. Which of the following is the primary principle involved in community organization? A. Technique in asking questions B. Assembly of community leaders C. Defined functions in each group D. Planning group needs Answer: D Rationale: The primary principle involved in community organization is planning group needs to represent all people concerned and the discussion must include people with technical knowledge of health problems. (Community Health Nursing Services in the Philippines) 47. Which one of the following is not an objective of community organization? A. Learn about common problems B. Plan the kind of action needed to solve problems C. Never to act on issues presented D. Identify problems as their own Answer: C Rationale: Community Organizing is a process by which people, health services and agencies of the community are brought together to learn about the common problems (option A), identify these problems as their own (option D), plan the kind of action needed to solve these problems (option B) and act on this basis. (Community Health Nursing Services in the Philippines) 48. Which basic method can determine the extent to which the basic needs are met for the health workers to bring about the adjustment between need and resources? A. Fact finding C. Determination of needs B. Program formation D. Education and Interpretation Answer: A

East West Nursing Review Center

Rationale: Fact finding is the basic method that serves to identify needs, determine the extent to which the basic needs are met and make known gaps and overlaps in existing services. This will also help health workers to bring about the adjustment between need and resources. (Community Health Nursing Services in the Philippines) 49. As a nurse, you have gained entry in the community. Which initial step is done? A. List names of persons to contact B. Gather information from persons and records C. Arrange the first meeting D. Prepare the agenda Answer: B Rationale: The initial step when the nurse gained entry in the community is gathering initial information about the community from other members of the RHU or from records and reports. (Community Health Nursing Services in the Philippines) 50. Which of the following should a nurse do if the purpose in community organizing is to integrate with the people? a. Live with the people b. Assign community officers c. Be punctual in reporting at the Rural Health Unit (RHU) d. Reside in the home of the Municipal Health Officer Answer: ARationale: When integrating with the people, it is crucial for the nurse to reside in his/her area of assignment. This is the first act of integrating with the people. Living with them will give the nurse an in depth participation in community health problems and needs. (Community Health Nursing Services in the Philippines) PEDIATRIC NURSING Situation 1: Nurse Melai is taking care of a 12-year-old female hospitalized with a suspected diagnosis of acute bacterial meningitis. 1. In performing the lumbar puncture, which safety measure would Nurse Melai implement while CSF specimen is obtained? Place the child in a sitting position with the head extended Place the child in a side-lying position with the back closest to the edge of the bed Do not wear gloves because the physician is collecting the specimen Place one arm behind the childs neck and the other behind the knees Answer: B Rationale: This allows the physician to assess the site with ease. (Chernecky) 2. Nurse Melai reviews the results of the CSF analysis and determines that which of the following results would confirm bacterial meningitis? a. Cloudy CSF, protein, and glucose b. Cloudy CSF, protein, and glucose c. Clear CSF, protein, and glucose d. Clear CSF, pressure, and protein Answer: B Rationale: In case of bacterial meningitis, findings usually include an elevated pressure, turbid or cloudy CSF, elevated leukocytes, elevated protein, and decreased glucose levels. (Saunders, 3 rd Edition) 3. Based on the mode of transmission of this infection, which of the following would be included in the plan of care? a. No precautions are required as long as antibiotics have been started b. Maintain enteric precautions c. Maintain respiratory isolation precautions for at least 24 hours after the initiation of antibiotics d. Maintain neutropenic precautions Answer: C Rationale: For a child with meningitis, prescribed antibiotic is administered as soon as it is ordered. The child is placed in respiratory isolation for at least 24 hours while culture results are obtained and the antibiotics having an effect. (Saunders, 3rd Edition) 4. In planning a roommate for the child, Nurse Melai should realize that a child of this developmental level will: a. Prefer another girl her own age b. Most likely seek out opportunities to socialize with teenagers c. Enjoy being with either a girl or boy, as long as they are the same age d. Feel helpful if given the opportunity to look after a slightly younger child Answer: A Rationale: Younger teenage girls in particular prefer the company of other young girls. They have a definite preference for same sex, same-age companions. (Davis NCLEX-RN Success, 2nd Edition)

a. b. c. d.

East West Nursing Review Center

5. As the child recovers from meningitis, for which long-term complication should Nurse Melai watch carefully? Encephalitis c. Learning disabilities Hydrocephalus d. Mental retardation Answer: B Rationale: As healing of the pathways of the CSF occurs following an episode of meningitis, scar tissue naturally forms. This may lead to non-communicating hydrocephalus. (Davis NCLEX-RN Success, 2nd Edition) Situation 2: A child is admitted to the hospital with a history of vomiting and diarrhea for two days. The admitting diagnosis is gastroenteritis and isotonic dehydration. 6. The nurse understands that isotonic dehydration: a. Occurs when water and electrolytes are lost in about the same proportions as they exist in the body b. Occurs when the loss of electrolytes is greater than the loss of water c. Occurs when the loss of water is greater than the loss of electrolytes d. Causes the serum sodium level to rise above 150 mEq/L Answer: A Rationale: Isotonic dehydration occurs when water and electrolytes are lost in about the same proportions as they exist in the body. In this type of dehydration, the serum sodium levels remain normal (135 to 145 mEq/L). (Saunders Comprehensive Review for the NCLEX-RN Examination, 3rd Edition) 7. What type of room assignment should the nurse make? a. A room near the nurse's station so that he can be checked frequently and heard if he vomits b. A single room with a sink nears the doorway for isolation use c. A double room with another toddler who also has vomiting and diarrhea d. A bed in the pediatric intensive care unit, in case dehydration develops Answer: B Rationale: The child should be placed on enteric isolation until the lab reports no contagious organisms in the stool. If the stool is infected, isolation is continued after the antibiotics are completed until three consecutive daily stool specimens are negative. (NSNA NCLEX-RN Review, 4th Edition) 8. Which priority nursing intervention should be included when caring for a client diagnosed with gastroenteritis? a. Encouraging optimal nutritional intake b. Alleviating abdominal pain and cramping c. Administering an oral anti-emetic every 2 hours d. Monitoring intake and output, and electrolyte levels Answer: D Rationale: With gastroenteritis, the client typically experiences vomiting and diarrhea, which put the client at risk for fluid volume deficit and electrolyte imbalance. Close monitoring of intake and output along with serum electrolyte levels is important to prevent any imbalances and ensure prompt treatment if an imbalance occurs. (Lippincott's Review Series: Medical-Surgical Nursing, 4th Edition) 9. The nurse is assessing the child for dehydration. The nurse determines that the child is moderately dehydrated if which symptom is noted on assessment? Flat fontanels Moist mucous membranes Pale skin color Oliguria Answer: D Rationale: In moderate dehydration, oliguria would be present. (Saunders Comprehensive Review for the NCLEX-RN Examination, 3rd Edition)

a. b. c. d.

10. The nurse prepares to take the child's temperature and avoids which method of measurement? a. Tympanic c. Rectal b. Axillary d. Electronic Answer: C Rationale: Rectal temperature measurements should be avoided if diarrhea is present. Use of rectal temperature stimulates peristalsis and cause more diarrhea. (Saunders Comprehensive Review for the NCLEX-RN Examination, 3rd Edition) Situation 3: A nurse is working in the Holy Child ward. One of her patients is an infant diagnosed with Cystic fibrosis. 11. The foul-smelling, frothy characteristics stool in cystic results from the presence of large amounts of: a. Undigested fat b. Sodium and chloride c. Semidigested carbohydrates d. Lipase, trypsin and amylase Answer: A Rationale: Because of lack of the pancreatic enzyme lipase, fats remain unabsorbed and are excreted in excessive amounts in the stool.

East West Nursing Review Center

(Mosbys, 18th Edition) 12. The nurse, when planning care, recalls that chest percussion and postural drainage for a toddler with cystic fibrosis are best done: a. After suctioning b. Before aerosol therapy c. One hour before meals d. Immediately after meals Answer: C Rationale: This regimen will give the child an opportunity to rest before eating. (Mosbys, 18th Edition) 13. Medications that will probably be used in the therapeutic regimen for a child with cystic fibrosis include: a. A steroid and an antimetabolite b. Pancreatic enzymes and antibiotics c. Aerosol mists, decongestants, and fat-soluble vitamins d. Antibiotics, a multivitamin preparation, and cough drops Answer: B Rationale: Pancreatic enzymes are given as replacement because of the lack of their production by the pancreas. Antibiotics are prescribed to control respiratory tract infection. (Mosbys, 18th Edition) 14. At a previous visit, the parents of an infant with cystic fibrosis received instruction in administration of pancrelipase (Pancrease). At a follow up visit, which finding in the infant suggests that the parents are not administering the pancreatic enzymes as instructed? a. Fatty stools c. Bloody stools b. Bloody urine d. Glucose in urine Answer: A Rationale: Pancreatic enzymes normally aid in food digestion in the intestine. In a child with cystic fibrosis, however, these natural enzymes cannot reach the intestine because mucus blocks the pancreatic duct. Without these enzymes, undigested fats and proteins produce fatty stools. Treatment with pancreatic enzymes should result in stools of normal consistency. Noncompliance with the treatment produces fatty stools. 15. In young children with cystic fibrosis, frequent stool and tenacious mucus often produce: a. Anal fissures c. Rectal prolapse b. Intussusception d. Meconium ileus Answer: C Rationale: Rectal prolapse is a common GI complication and results from wasting of perirectal supporting tissues, secondary to malnutrition. (Mosbys, 18th Edition) Situation 4: The following cases pertain to different Gastrointestinal Disorders in children. 16. A toddler is brought to the emergency department with sudden onset of abdominal pain, vomiting and stools that look like red currant jelly. To confirm intussusception, the suspected cause of these findings, the nurse would expect the physician to order: a. Barium enema b. Suprapubic aspiration c. NG Tube insertion d. Indwelling urinary catheter insertion Answer: A Rationale: A barium enema commonly is used to confirm and correct intussusception. Performing a suprapubic aspiration or NG tube insertion or an indwelling catheter insertion wouldnt help diagnose or treat this disorder. 17. The nurse documents that the infant with GERD should be maintained in which position following feeding and at night in order to manage reflux? 30-degree angle when supine 60-degree angle when supine Head-elevated prone position 20-degree angle when supine Answer: C Rationale: The infant should be placed in the flat prone position or the head-elevated prone position following feedings and at night. (Saunders Comprehensive Review for the NCLEX-RN Examination, 3rd Edition) 18. A 5-year-old child has lactose intolerance. The nurse tells the mother that it is necessary to provide which dietary supplement in the childs diet? Zinc c. Calcium Protein d. Fats Answer: C Rationale: Lactose intolerance is the inability to tolerate lactose, the sugar found in dairy products. Removing milk from the diet can provide adequate relief from symptoms. Additional dietary changes may be required to provide adequate source of calcium, and in the infant, protein and calories. (Saunders Comprehensive Review for the NCLEX-RN Examination, 3rd Edition)

East West Nursing Review Center

19. A nurse reviews the record of 3-week-old infant and notes that the physician has documented a diagnosis of suspected Hirschsprungs disease. The nurse reviews the assessment finding documented in the record, knowing that which symptom most likely led the mother to seek healthcare for the infant? Diarrhea Projectile vomiting Regurgitation of feedings Foul-smelling ribbon-like stools Answer: D Rationale: Chronic constipation beginning in the first month of life and resulting in pellet-like of ribbon stools that are foul-smelling is a clinical manifestation of this disorder. Delayed passage or absence of meconium stool in the neonatal period is the cardinal sign. Bowel obstruction, especially in the neonatal period, abdominal pain and distention, and failure to thrive are also clinical manifestations. (Saunders Comprehensive Review for the NCLEX-RN Examination, 3rd Edition) 20. The nurse monitors the infant with suspected diagnosis of imperforate anus, knowing that which of the following is a clinical manifestation associated with this disorder? a. Sausage-shaped mass palpated in the upper right abdominal quadrant b. Bile-stained fecal emesis c. Failure to pass meconium stool in the first 24 hours after birth d. The passage of currant jelly-like stools Answer: C Rationale: During the newborn assessment, this defect should be identified easily on sight. However, a rectal thermometer or tube may be necessary to determine patency if meconium is not passed in the first 24 hours after birth. Other assessment findings include absence or stenosis of the anal rectal canal, presence of an anal membrane, and an external fistula to the perineum. (Saunders Comprehensive Review for the NCLEX-RN Examination, 3rd Edition) Situation 5: The following questions will assess your knowledge on basic life support measures for infant and children. 21. The nurse performs cardiopulmonary resuscitation (CPR) for 1 minute on an infant without calling for assistance. In reassessing the infant after 1 minute of CPR, the nurse finds he still isnt breathing and has no pulse. The nurse should then: a. Resume CPR beginning with breaths b. Declare her efforts futile c. Resume CPR beginning with chest compressions d. Call for assistance Answer: D Rationale: After 1 minute of CPR, the nurse should call for assistance and then resume efforts. 22. A 10-year-old child begins to choke and cough on a piece of candy. Which priority nursing action should the nurse implement? a. Provide abdominal thrusts b. Give 5 back slaps, followed by 5 abdominal thrusts c. Look in the mouth and perform a blind finger sweep d. Allow a child to expel the candy by himself or herself Answer: D Rationale: If a child is choking, allow the victim to continue to cough if the cough is forceful. (Saunders, 3rd Edition) 23. A nurse is preparing to attempt to relieve an airway obstruction on a 3-year-old conscious child. The nurse performs this maneuver by placing the hands between the: a. Umbilicus and the groin b. Groin and the abdomen c. Umbilicus and the xiphoid process d. Lower abdomen and the chest Answer: C Rationale: To perform the Heimlich maneuver on a child, the rescuer places the thumb side of one fist against the victims abdomen in the midline slightly above the umbilicus and well below the tip of the xiphoid process. (Saunders, 3rd Edition) 24. A nurse is performing CPR on an infant. When performing chest compressions, the nurse understands that the compression rate is at least: a. 60 times per minute b. 80 times per minute c. 100 times per minute d. 160 times per minute Answer: C Rationale: In an infant, the rate of chest compression is at least 100 times per minute. (Saunders, 3 rd Edition) 25. Which among the following is the most appropriate location to assess the pulse of an infant under 1 year of age? a. Brachial c. Popliteal

East West Nursing Review Center

b. Carotid d. Radial Answer: A Rationale: To assess a pulse in an infant, check the pulse at the brachial artery. (Saunders, 3rd Edition) Situation 6: The first hours after birth represent a critical adjustment period for the newborn. The nurse must provide direct care to the newborn immediately after birth. 26. Which of the following would the nurse identify as goal of newborn care in the initial postpartum period? a. To facilitate development of a close parent-newborn relationship b. To assist parents in developing healthy attitudes about childbearing practices c. To identify actual or potential problems requiring immediate or emergency attention d. To provide the parents of the newborn with information about well-baby programs Answer: C Rationale: In the initial postpartum period, one of the goals of newborn care is to identify actual and potential problems that might require immediate attention. Other goals include establishing and maintaining an airway and supporting respirations, maintaining warmth and preventing hypothermia, and ensuring safety and preventing injury or infection. 27. Which of the following actions would be least effective in maintaining a neutral environment for the newborn? a. Placing an infant under a radiant warmer after bathing b. Covering the scale with a warmed blanket prior to weighing c. Placing crib close to the nursery window for family viewing d. Covering the infant's head with a stockinette Answer: C Rationale: Heat loss by radiation occurs when the infant's crib is placed too near cold walls or windows. Thus, placing the newborns crib close to the viewing window would be least effective. 28. When preparing to administer the vitamin K to a neonate, the nurse would select which of the following sites as appropriate for the injection? a. Deltoid muscle b. Anterior femoris muscle c. Vastus lateralis muscle d. Gluteus maximus muscle Answer: C Rationale: The middle third of the vastus lateralis is the preferred injection site for vitamin K administration because it is free of blood vessels and nerves, and is large enough to absorb the medication. 29. The first period of reactivity in the newborn begins at birth. The following are characteristics of this period EXCEPT: a. This lasts for 30 minutes b. Respirations are increased to 80/minute c. Flaring of nares and grunting are common d. Bowel sounds are present Answer: D Rationale: Bowel sounds are usually absent at this time. 30. A nurse providing care to a newborn would use knowledge of which of the following concepts underlying adaptation of the newborn's immune system? a. Iron stores from the mother are sufficient to carry the newborn through the 5th month of extrauterine life b. Unconjugated bilirubin can leave the vascular system and permeate the other extravascular tissues c. The newborn is unable to limit invading organisms at their point of entry d. Most newborns void in the first 24 hours after birth and 5 to 20 times thereafter Answer: C Rationale: The newborns cannot limit the invading organism at the port of entry. Situation 7: Santino, a 3-year-old child is scheduled for a tonsillectomy. 31. A nurse is reviewing the laboratory results of Santino. The nurse determines that which of the following laboratory values is most significant to review? a. Prothrombin time c. Blood urea nitrogen b. Sedimentation rate d. Creatinine Answer: A Rationale: Because the tonsillar area is so vascular, postoperative bleeding is a concern. PT, PTT, platelet count, hg, hct, WBC, and urinalysis are performed preoperatively. PT results would identify a potential for bleeding. (Saunders, 3rd Edition) 32. On the day of surgery, Santino will most likely be fearful of: a. Intrusive procedure b. Perceived abandonment c. Premature death

East West Nursing Review Center

d. Unfamiliar caregivers Answer: A Rationale: One of the greatest fears of preschoolers is fear of mutilation. Other options are not developmentally appropriate responses for a preschooler. 33. A nurse panning care for Santino knows that which of the following would present the highest risk of aspiration during surgery? a. Difficulty in swallowing b. The presence of loose teeth c. Bleeding during surgery d. Exudate in the throat area Answer: B Rationale: In the preoperative period, the child should be observed for the presence of loose teeth to decrease the risk of aspiration during surgery. (Saunders, 3rd Edition) 34. After a tonsillectomy, the nurse documents on the plan of care to place Santino in which most appropriate position? a. Supine c. Side-lying b. Trendelenburg's d. High Fowler's Answer: C Rationale: The child should be placed in a prone or side-lying position following tonsillectomy to facilitate drainage. (Saunders, 3rd Edition) 35. After a tonsillectomy, the nurse suspects hemorrhage postoperatively when the child: a. Snores noisily b. Becomes pale c. Complains of thirst d. Swallows frequently Answer: D Rationale: The seeping of blood from the operative site increases secretions, which the child adapts to by swallowing frequently. (Mosbys, 18th Edition) Situation 8: A 2-week-old is diagnosed as having hypertrophic pyloric stenosis, and is scheduled for corrective surgery. 36. When palpating the infant's abdomen, the nurse would expect to find: a. An impacted and distended colon b. Marked tenderness around the umbilicus c. An olive-sized mass in the right upper quadrant d. Rhythmic peristaltic waves in the lower abdomen Answer: C Rationale: The olive-like mass is caused by the thickened muscle of the pyloric sphincter. 37. The nurse should carefully observe the infant for: a. Quality of cry b. Character of stool c. Signs of dehydration d. Coughing after feeding Answer: C Rationale: Hypertrophy of the pyloric sphincter, at the distal end of the stomach, causes partial and then complete obstruction. Non-projectile vomiting progresses to projectile vomiting which rapidly leads to dehydration. 38. After the corrective surgery, the nurse caring for an infant, who had been formula-fed, notices that the post-operative orders are similar to those for the other infants having undergone such surgery and include: a. Thickened formula 24 hours after surgery b. Withholding all feedings for the first 24 hours c. Regular formula feeding 24 hours after the surgery d. Additional glucose feeding as desired after the first 24 hours Answer: C Rationale: Initial feedings of glucose and electrolytes in water or breast milk are given 4 to 6 hours after surgery. When clear fluids are retained, usually within 24 hours, formula feedings are begun. 39. When the corrective surgery for hypertrophic pyloric stenosis is completed, the infant was returned in stable condition to the pediatric unit with an intravenous infusion and a nasogastric tube in place.

East West Nursing Review Center

The priority nursing action should be to: Apply adequate restraints Administer a mild sedative Assess the IV site for infiltration Attach the nasogastric tube to wall suction Answer: A Rationale: Protecting the IV and nasogastric tube from becoming dislodged is a priority. B, C and D. These are not priority actions. 40. To reduce vomiting, the nurse should teach the mother that immediately after the feeding the infant, she should: Rock the baby for 20 minutes Place the baby in an infant seat. Place the baby flat on the right side d. Keep the baby awake with sensory stimulation Answer: B Rationale: An elevated position allows gravity to aid in preventing vomiting. Situation 9: Nurses must know the developmental milestones of children in order to perform age-appropriate assessments. 41. Which behavior should the nurse expect a 3-year-old child to be capable of doing? Going up the stairs on alternate feet Pedaling a bicycle Dressing without supervision Tying shoelaces Answer: A Rationale: 3-year-olds should be able to coordinate the brain and gross motor activity necessary to go up stairs using alternate feet. (Davis NCLEX-RN Success, 2nd Edition) 42. The best way to perform a DDST on a 9-month-old is to: a. Take the infant from the mother and ask her to wait in the childs room b. Take the infant from the mother and ask her to come with them to the testing area c. Briefly talk first with the mother, then take the infant to the testing area alone d. Ask the infants mother to carry the child to testing area Answer: D Rationale: The parent should accompany the infant who is to have the DDST and that the examiner should do everything possible to establish rapport with the parent and the infant. (Davis NCLEX-RN Success, 2nd Edition) 43. At 7 months of age, an infant exhibits the following skills. The nurse should know that the most recently acquired skill is the ability to: Roll over Sit up Bear some weight on legs Pick up objects with palmar grasp Answer: B Rationale: At 7 months of age, an infant may sit with some support or sit alone. Either behavior is commonly acquired at this age. (Davis NCLEX-RN Success, 2nd Edition) 44. In assessing the development of a 5-year-old, the nurse would not expect the child to be able to: Name primary colors Count to 100 Know the days of the week Give telephone number and address Answer: B Rationale: 5-year-olds may count up to 20 or 25, but seldom beyond this. (Davis NCLEX-RN Success, 2nd Edition) 45. Which of the following activities should a 2-year-old child be able to do? a. Build a tower of 8 cubes b. Point out a picture c. Wash and dry his hands d. Remove a garment Answer: D Rationale: According to Denver II Developmental Screening Test, most 2 years old are able to remove one garment. Situation 10: Nurse Jean is working in the Orthopedic ward, wherein she encounters various cases of fractures in children. 46. A 9-year-old child has a fractured tibia, and a full leg cast has been applied. Nurse Jean should immediately notify the physician if assessment demonstrates: a. A pedal pulse of 90 b. An increased urinary output c. An inability to move the toes d. A plaster cast that is still damp after 4 hours

East West Nursing Review Center

Answer: C Rationale: Cold toes, loss of sensation on toes, and pain, which indicate poor circulation, should be reported to the physician immediately. (Mosbys, 18TH Edition) 47. To hasten drying of the cast, Nurse Jean should include the following in the care plan: a. Using a blow dryer b. Exposing the casted extremity c. Covering the cast with a light sheet d. Opening the window slightly to circulate air Answer: B Rationale: This is the safest way. (Mosbys, 18TH Edition) 48. An 11-year-old child has just had the application of a cast for a fractured wrist. The wrist and elbow are immobilized. When providing home care instructions before discharge, Nurse Jean should include the fact that the: a. Child can resume usual activities b. Casted arm should be elevated while upright and resting c. Physician should be notified if swelling of the fingers occurs d. Shoulder on the affected side should remain immobilized in a splint Answer: C Rationale: Swelling may cause the cast to become too tight resulting in neurovascular damage; permanent damage can occur in 6-8 hours. (Mosbys, 18TH Edition) 49. After orthopedic surgery, a 15-year-old complains of pain and is given 15 mg of codeine sulfate ad ordered every3 hours PRN. Two hours after having been given this medication, the adolescent complain of severe pain. Nurse Jean should: a. Report that the adolescent has an apparent idiosyncrasy to codeine b. Tell the adolescent that additional medication cannot be given for 1 more hour c. Request that the physician evaluate the adolescents need for additional mediation d. Administer another dose of codeine within 30 minutes, because it is a relatively safe drug Answer: C Rationale: (Mosbys, 18TH Edition) 50. Nurse Jean takes care of an 8-year-old child with a diagnosis of a basilar skull fracture. Nurse Jean reviews the physicians orders and contacts the physician to question which order? a. Clear liquid intake b. Maintain a patent intravenous line c. Daily weight d. Suction as needed Answer: D Rationale: This is contraindicated; there is a high risk of secondary infection and a probability of the catheter entering the brain through the fracture. A. Child is maintained on NPO or restricted to clear liquids until it is determined that vomiting will not occur. B. This is necessary for fluids and medications. C. Fluid balance is monitored closely by daily weight, and I&O measurement to detect early signs of water retention, excessive dehydration, and states of hyper/hypotonicity. (Saunders, 3rd Edition) MATERNITY NURSING Situation 1: Cora, 9 months pregnant, is admitted to the hospital with bleeding caused by possible placenta previa. The laboratory technician takes blood samples and IV fluids are begun. 31. A client with placenta previa is likely to present with: a. Hard, tender uterus b. Painless, bright-red vaginal bleeding after the 20th week of gestation c. A sluggish fetus with weak heart sounds on auscultation d. Bleeding during the first trimester Answer: B Rationale: With placenta previa, the client has painless, bright-red vaginal bleeding after the 20th week of gestation that starts without warning and stops spontaneously. Palpation reveals a soft, nontender uterus, and auscultation reveals an active fetus with good heart sounds. Also, bleeding commonly occurs during the third trimester. (Straight As in Maternal-Neonatal Nursing, 2nd Edition) 32. Nursing care for Cora includes: Withholding foods and fluids Encouraging ambulation and supervision Inspecting the hemorrhage Avoiding all extraneous stimuli Answer: C Rationale: To prevent further maternal and fetal complication, clients must be continuously observed

East West Nursing Review Center

for blood loss by the monitoring of external bleeding and the counting and weighing of pads. 33. The nurse following the physicians order, begins administering oxygen by mask. The clients apprehension is increasing and she asked the nurse what is happening. The nurse tells her not to worry, that she is going to be all right and everything is under control. The nurse statements are: a. Correct, since only the physician should explain why treatment are being done b. Proper, since the clients anxieties would be increased if she knew the dangers c. Adequate, since all preparations are routine and need no explanation d. Questionable, since the client has the right to know what treatment is being given and why Answer: D Rationale: The client has the right to a complete and accurate explanation of treatment. 34. If a vaginal examination is to be performed on Cora, the nurse should be prepared for an immediate: Induction of labor c. Forceps delivery Cesarean delivery d. X-ray examination Answer: B Rationale: Vaginal exam might precipitate severe bleeding, which could be life threatening to the mother and infant and necessitate immediate CS delivery. 35. The care of a client with placenta previa includes: Vital signs at least once per shift A tap-water enema before delivery Observation and recording of the bleeding Limited ambulation until the bleeding stops Answer: C Rationale: Observation and documentation of bleeding are independent nursing functions and necessary for implementing safe care, because hemorrhage and shock can be life-threatening. Situation 2: Helen, age 20, is 37 weeks pregnant. She is admitted to the hospital with preeclampsia, moderate vaginal bleeding, and sudden abdominal pain. The results of the ultrasound indicate that abruptio placenta is present. 36. Based on these findings, the nurse would prepare the client for: a. Complete bed rest for the remainder of the pregnancy b. Delivery of the fetus c. Strict monitoring of intake and output d. The need for weekly monitoring of coagulation studies until the time of delivery Answer: B Rationale: The goal of management in abruptio placenta is to control the hemorrhage and deliver the fetus a soon as possible. Delivery is the treatment of choice if the fetus is at term gestation, or if bleeding is moderate to severe, and the mother or fetus is in jeopardy. (Saunders, 3rd Edition) 37. On Helens admission to the unit the nurse should observe for: a. Decrease in size of uterus, cessation of contractions, visible or concealed hemorrhage b. Firm and tender uterus, concealed or external hemorrhage, shock c. Increase in size uterus, visible bleeding, no associated pain d. Shock, decrease in size uterus, absence of external bleeding Answer: B Rationale: Signs of mild to moderate placental separation include uterine discomfort and tenderness because of concealed bleeding. Visible bleeding maybe scant, moderate or heavy 38. The nurse realizes that the abdominal pain associated with abruption placenta is caused by: Hemorrhagic shock Inflammatory reactions Blood in the uterine muscle Concealed hemorrhage Answer: D Rationale: The blood can not escape from behind the placenta. Thus the abdomen becomes board like and painful because of the entrapment. 39. Helen is given a unit of blood. The realizes that this is necessary, since the bleeding following severe abruptio placenta is usually caused by: Hypofibrinogenemia c. Thrombocytopenia Hyperglobulinemia d. Polycythemia Answer: A Rationale: Clotting defects are common in moderate and severe abruption placentae because of the loss of fibrinogen from severe internal bleeding. 40. A nurse is monitoring Helen for Disseminated Intravascular Coagulopathy. Which assessment finding is least likely to be associated with DIC? a. Swelling of the calf on one leg b. Prolonged clotting times

East West Nursing Review Center

Decreased platelet count Petechiae, oozing from injection sites, and hematuria Answer: A Rationale: These signs are most likely associated with thrombophlebitis. Situation3: Anne delivered a child two days ago. She is breastfeeding her child. 41. To show Anne how to help her have good attachment of the infant during breast feeding. Which of the following statements should NOT be included? a. Place the infant in your most convenient position b. Touch the infants lips with her nipples c. Wait until the infants mouth is widely open d. Move the infant quickly onto her breast, aiming the infants lips well below the nipple Answer: A Rationale: The infant should be positioned in such a way that the chin of the child is touching the breast of the mother, for better latch-on technique. 42. Which of the following signs of good attachment should the nurse teach Anne? a. The chin should touch the breast, the mouth is wide open while the lower lip is turned inward, and more areola is visible above than below b. The chin should touch the breast, the mouth is wide open while the lower lip turned outward, and more areola visible above than below c. The chin should touch the breast, the mouth is wide open while the lower lip turned outward, and more areola visible below than above d. The chin should touch the breast, the mouth is wide open while the lower lip turned inward, and more areola is visible below than above Answer: B Rationale: Signs of good latch-on technique include: chin touching the breast, the mouth is wide open; the lower lip turned outward; and 43. Which of the following instructions can the nurse give to Anne? Wear a good, well-supporting bra Apply warm compresses to breast if too full Apply cold compresses to breast if too full Do not apply any soap to your nipples Answer: A Rationale: A well-fitting supportive bra with wide straps can be recommended for a nursing mother. The nursing mother's bra should have front flaps over each breast for easy access during nursing periods. 44. Anne is visited by the home health nurse two weeks after delivery. The woman is febrile with flu-like symptoms. On assessment, the nurse notes a warm, reddened painful area of the right breast. The best initial action of the nurse is to: a. Contact the physician for an order of antibiotics b. Advise the mother to stop breastfeeding and pumping c. Assess the mother's feeding technique and knowledge of breast care d. Obtain a sample of the breast milk for culture Answer: A Rationale: These symptoms are signs of infectious mastitis, usually caused by Staphylococcus aureus. A 10-day course of antibiotics is needed. 45. Anne has received treatment for a warm, reddened, painful area in the breast as well as cracked and fissured nipples. The client expresses the desire to continue breastfeeding. The following are instructions that the nurse should include to prevent recurrence of this condition EXCEPT: a. Change the breast pads frequently b. Expose the nipples to air for part of each day c. Wash hands before handling the breast and breastfeeding d. Make sure that the baby grasps the nipple only Answer: D Rationale: The baby should grasp both the nipple and areola. Situation 4: Rita, 27 years old, is a gravida 1 in the active phase of labor. Fetal position is LOA, and cervix is 4 cm dilated. 46. Rita wants to walk about in the labor room. Which of the following criteria will help the nurse determine whether she should walk? a. Whether membranes are intact b. Frequency of contraction c. Fetal position d. Fetal station Answer: D Rationale: If the fetus is engaged (0 station, or +1 or more), cord prolapse will be prevented whether her membranes have ruptured or not. 47. When planning comfort measures to help Rita in active labor to tolerate her pain, the nurse must consider which of the following? a. Early labor contractions are usually regular, coordinated, and very painful

c. d.

East West Nursing Review Center

If women are properly prepared, they will require no pain medication to manage their pain Pain medication given during the latent phase of labor is not likely to impair contractions The acceleration phase of labor can be a time of true discomfort and high anxiety Answer: D Rationale: During the acceleration/active phase of labor, contractions grow strong, last longer and begin to cause true discomfort and high anxiety as she realizes that labor is truly progressing and her life is about to change forever.

b. c. d.

48. When her membranes rupture, the nurse should expect to see: a. A large amount of bloody fluid b. A moderate amount of clear to straw-colored fluid c. A small amount of greenish fluid d. A small segment of the umbilical cord Answer: B Rationale: With the baby in a vertex LOA presentation and no other indicators of fetal distress, amniotic fluid has a clear to straw-colored appearance. 49. When her membranes rupture, the nurse's first action should be to: a. Notify the physician because delivery is imminent b. Measure the amount of fluid c. Count the fetal heart rate d. Perform a vaginal exam Answer: C Rationale: Immediately after rupture of membranes fetal heart tones should be checked, and then checked again after the next contraction or after 5-10 minutes. 50. During the third stage of labor, the nurse may have which of the following responsibilities? a. Administer intramuscular Oxytocin to facilitate uterine contractility b. Monitor for blood loss greater than 100 cc, which would indicate gross hemorrhage c. Note if the placenta makes a Schultz presentation, which is a sign of gross complication d. Push down on the relaxed uterus to aid in the removal of the placenta Answer: A Rationale: Oxytocin causes uterine contraction, thus preventing hemorrhage. Situation 5: A pregnant client has delayed her first prenatal visit. She visits the prenatal clinic only after she starts to experience edema of the feet and hands. The nurse takes a history and physical assessment to begin Mrs. Barton's care. 51. The client's response to one of the nurse's questions is, This is my third pregnancy. I miscarried twice, the first time I was 8 weeks pregnant, and the last time I was 26 weeks pregnant. The nurse correctly records Mrs. Barton's pregnancy status as: G2, P0, A1 c. G3, P0, A2 G2, P1, A1 d. G3, P1, A1 Answer: D Rationale: G3 (total of 3 pregnancies: present pregnancy plus the two miscarriage); P1 (second miscarriage at 26 weeks AOG); A1 (first miscarriage at 8 weeks AOG). Gravida pertains to the number of pregnancies regardless of the duration. Para pertains to the number of pregnancies that lasted more than 20 weeks, regardless of the outcome. Abortion pertains to the number of terminated pregnancies, not reaching the age of 20 weeks (age of viability). 52. During the examination, while client is lying in a lithotomy position, the client complains of dizziness and nausea. What would be an appropriate nursing action to relieve the client's discomfort? a. Administering an antiemetic ordered by the physician b. Offering small sips of ginger ale c. Assisting to a side-lying position temporarily d. Discontinuing the examination Answer: C Rationale: Lying supine for a prolonged period of time, with the legs in a lithotomy position. The weight of the growing uterus presses the vena cava against the vertebrae, obstructing blood flow to the lower extremities. This causes a decrease ion blood return to the heart, and consequently decreased cardiac output and hypotension, manifesting as nausea and dizziness, lightheadedness, faintness and palpitations. Assisting the client in a side-lying position relieves pressure on the vena cava, thus improving blood circulation. 53. Diplopia was noted during the assessment of Mrs. Barton. This condition is described as: Elevated pigmentation of the skin Double vision Facial edema Gingivitis Answer: B Rationale: Diplopia is described as having double vision. Women with PIH commonly report spots

East West Nursing Review Center

before their eyes, or having double vision. 54. The physician asks the nurse to make sure his pregnant patient's next appointment is scheduled correctly. The patient is in her 33rd week. Her next appointment should be in: 1 month c. 2 weeks 3 weeks d. 1 week Answer: C Rationale: First visit may be made as soon as the woman suspects she is pregnant. Subsequent visits are as follows: monthly until the 8th month; every 2 weeks during the 8 th month, and weekly during the 9th month. More frequent visits are scheduled if problems arise. The client is 8 months pregnant (33 weeks), so the next visit will be scheduled after 2 weeks. 55. Mrs. Barton reports that the last day of her last menstrual period was May 11, 2009. Her menstruation lasted for 5 days. Her expected date of delivery will be: February 14, 2010 c. August 4, 2010 February 18, 2010 d. August 18, 2010 Answer: A Rationale: A. February 14, 2010 Her first day of LMP is May 7, 2009. Use Naegel's rule: month-3; day+7; year+1 05 07 2009 -3 +7 +1 -----------------------------02 14 2010 Situation 6: Nene is a 26-year-old woman you admit to a birthing room. Shes been having contractions 45 seconds long and 3 minutes apart for the last 6 hours. She tells you she wants to have her baby natural without analgesia or anesthesia. Her husband is in the army assigned overseas, so he is not with her. Although her sister lives only two blocks from the birthing center, Nene doesnt want her called. She asks if she can talk to her mother on the telephone. As you finish assessing contractions, she screams with pain and shouts, Ginagawa ko na ang lahat ng makakaya ko! Kailan ba matatapos ang paghihirap kong ito? 56. Nene did not recognize for over an hour that she was in labor. A sign of true labor is: a. Sudden increased energy from epinephrine release b. Nagging but constant pain in the lower back c. Urinary urgency from increased bladder pressure d. Show or release of the cervical mucus plug Answer: D Rationale: Signs of true labor involve uterine and cervical changes. As the cervix softens and ripens, the mucus plug that filled the cervical canal during pregnancy (operculum) is expelled. The exposed cervical capillaries seep blood as a result of pressure exerted by the fetus. The blood, mixed with mucus, takes on a pink tinge and is referred to as show or bloody show. 57. Nene asks which fetal position and presentation are ideal. Your best answer would be: a. Right occipitoanterior full flexion b. Left transverse anterior in moderate flexion c. Right occipitoposterior with no flexion d. Left sacroanterior with flexion Answer: A Rationale: A fetus is born fastest from ROA/LOA position. Full flexion (chin touches the sternum, arms are flexed and folded on the chest, thighs are flexed onto the abdomen, and the calves are pressed against the posterior aspect of the thighs) allows the smallest diameter of the skull to enter to the pelvis. 58. Nene is having long and hard uterine contraction. What length of contraction would you report as abnormal? a. Any length over 30 seconds b. A contraction over 70 seconds in length c. A contraction that peaks at 20 seconds d. A contraction shorter than 60 seconds Answer: B Rationale: Uterine contractions lasting longer than 70 seconds should be reported, because contractions of this length begin to compromise fetal well-being by interfering with adequate uterine artery filling. (Pillitteri, Maternal and Child Health Nursing, 5th Edition) 59. You assess Nenes uterine contractions. In relation to the contraction, when does a late deceleration begin? a. 45 seconds after the contraction is over b. 30 seconds after the start of a contraction c. After every tenth or more contraction d. After a typical contraction ends Answer: B

East West Nursing Review Center

Rationale: Late decelerations are those that are delayed until 30 to 45 seconds after the onset of a contraction, and continue beyond the end of the contraction. This is an ominous pattern in labor, because it suggests uteroplacental insufficiency. (Pillitteri, Maternal and Child Health Nursing, 5th Edition) 60. Immediately after the membranes rupture, which of the following should the nurse check? a. Check the presence of infection b. Assess for prolapsed umbilical cord c. Check for maternal heart rate d. Assess the color of the amniotic fluid Answer: B Rationale: When membranes rupture, amniotic fluid is allowed to escape. The nurse should check for prolapse of the umbilical cord because there is a possibility that a loop of cord will escape with the fluid, which can cu off the oxygen supply to the fetus. Situation 7: Sexually Transmitted Diseases are important to identify during pregnancy because of its potential effect on the pregnancy, fetus, or newborn. The following questions pertain to STDs. 61. Frankie, a promiscuous woman in Manila, submits herself to the clinic for check-up. She complains of vaginal irritation, redness and a thick cream cheese-like vaginal discharge. As a nurse, you will suspect that Frankie is having a vaginal infection caused by: a. Gardnerella vaginalis b. Candida albicans c. Treponema pallidum d. Herpes simplex virus type 2 Answer: B Rationale: Vaginal infection with Candida albicans (Candidiasis) is characterized by red and irritated vagina, pruritus, and thick vaginal discharge that resemble cream cheese. 62. Cecil comes to the health center. Her doctor examines Cecils vaginal secretions and finds out that she has Trichomoniasis infection. Trichomoniasis is diagnosed through which of the following methods? a. Vaginal secretions are examined in a wet slide that has been treated with potassium hydroxide b. Vaginal speculum is used to obtain secretions from the cervix c. A litmus paper is used to test if the vaginal secretions are infected with Trichomoniasis d. Vaginal secretions are examined on a wet slide treated with zephiram solution Answer: A Rationale: Trichomoniais is diagnosed by examination of vaginal secretions on a wet slide that has been treated with potassium hydroxide. (Pillitteri, Maternal and Child Health Nursing, 5th Edition)

63. The drug of choice for a client with Trichomoniasis is: a. Flagyl c. Monistat b. Cotrimazole d. Zovira Answer: A Rationale: The drug of choice for Trichomoniasis a single-dose oral metronidazole (Flagyl). 64. Jarisch-Herxheimer reaction maybe experienced by the client with syphilis after therapy with benzathine penicillin G. The characteristic manifestation of Jarisch-Herxheimer reaction are: a. Rashes, itchiness, hives and pruritus b. Confusion, drowsiness, and numbness of extremities c. Sudden episode of hypotension, fever, tachycardia, and muscle aches d. Episodes of nausea and vomiting, with bradypnea and bradycardia Answer: C Rationale: After therapy with benzathine penicillin G, a woman may experience sudden episode of hypotension, fever, tachycardia and muscle aches, which is known as the Jarisch-Herxheimer reaction. This occurs when large quantities of toxins are released into the body as spirochetes die, due to antibiotic treatment. Typically the death of these spirochetes and the associated release of endotoxins occur faster than the body can remove the toxins via the natural detoxification process performed by the kidneys and liver. The reaction lasts about 24 hours and then fades. (Pillitteri, Maternal and Child Health Nursing, 5th Edition) 65. Cryocautery may be used in removing the lesions of a client with venereal warts. The healing period after cryocautery may be completed in 4-6 weeks but may cause some discomforts to the woman. What measures can alleviate these discomforts? a. Kegels exercise b. Cool air c. Topical steroids d. Sitz bath and lidocaine cream Answer: D Rationale: With cryocautery, edema at the site is evident immediately. Lesions become gangrenous, and sloughing occurs in 7 days. Healing will be complete in 4 to 6 weeks with only slight depigmentation at the site. Sitz baths hand lidocaine cream may be soothing during the healing period. (Pillitteri, Maternal and Child Health Nursing, 5th Edition)

East West Nursing Review Center

Situation 8: Nurse Gwen is an Independent Nurse Practitioner following-up referred clients in their respective homes. She handles Leah's case, a postpartal mother. 66. Leah is developing constipation from being on bed rest. What measures would you suggest she take to help prevent this? a. Eat more frequent small meals instead of three large one daily b. Walk for at least half an hour daily to stimulate peristalsis c. Drink more milk, increased calcium intake prevents constipation d. Drink eight full glasses of fluid such as water daily Answer: D Rationale: Increasing oral fluid intake relieves constipation. 67. Which of the following actions would alert Nurse Gwen that Leah is entering a postpartal taking-hold phase? a. She urges the baby to stay awake so that she can breast-feed him or her b. She tells you she was in a lot of pain all during labor c. She says that she has not selected a name for the baby as yet d. She sleeps as if exhausted from the effort of labor Answer: A Rationale: Taking-hold phase usually happens by the third postpartum day. 68. At 6-week postpartum visit, what should Leah's fundic height be? a. Inverted and palpable at the cervix b. Six finger breadths below umbilicus c. No longer palpable on her abdomen d. One centimeter above the symphysis pubis Answer: C Rationale: Involution of the uterus occurs at a rate of 1 finger breadth (1 cm) per postpartum day, until by the end of the second week postpartum it is already a pelvic organ and cannot be palpated through the abdominal wall. B. This is during the 6th postpartum day. 69. Leah wants to loose the weight she gained in pregnancy, so she is reluctant to increase her caloric intake for breast-feeding. By how much should a lactating mother increase her caloric intake during the first 6 months after birth? a. 350 kcal/day c. 200 kcal/day b. 500 kcal/day d. 1000 kcal/day Answer: B Rationale: A lactating mom should have an additional intake of 500 kcal/day. 70. When preparing recommendations for Leah, which of the following contraceptive methods would be avoided? a. Diaphragm c. Oral contraceptives b. Female condom d. Rhythm method Answer: A Rationale: A. The diaphragm must be fitted individually to ensure effectiveness. Because of the changes to the reproductive structures during pregnancy and following delivery, the diaphragm must be refitted, usually at the 6 weeks examination following childbirth or after a weight loss of 15 lbs or more. In addition, for maximum effectiveness, spermicidal jelly should be placed in the dome and around the rim. However, spermicidal jelly should not be inserted into the vagina until involution is completed at approximately 6 weeks.

Situation 9: Nette, a nurse palpates the abdomen of Mrs. Medina, a primigravida. She is unsure of the date of her last menstrual period. Leopolds Maneuver is done. The obstetrician said that she appears to be 20 weeks pregnant. 71. Nette explains this because the fundus is: a. At the level of the umbilicus, and the fetal heart can be heard with a fetoscope b. 18 cm, and the baby is just about to move c. Is just over the symphysis, and fetal heart cannot be heard d. 28 cm, and fetal heart can be heard with a Doppler Answer: A Rationale: According to Bartholomew's rule, fundus located at the umbilicus is approximately 20 weeks AOG. Fetal heart can be heard using a fetoscope by 18 to 20 weeks AOG. 72. In doing Leopolds Maneuver palpation which among the following is NOT considered a good preparation: a. The woman should lie in a supine position with her knees flexed slightly b. The hands of the nurse should be cold so that abdominal muscles would contract and tighten c. Be certain that your hands are warm (by washing them in warm water first if necessary) d. The woman empties her bladder before palpation

East West Nursing Review Center

Answer: B Rationale: The hands of the nurse should be warm so that abdominal muscles will not contract and tighten on palpation. 73. In her pregnancy, she experienced fatigue and drowsiness. This probably occurs because: a. Of high blood pressure b. She is expressing pressure c. The fetus utilizes her glucose stores and leaves her with a low blood glucose d. Of the rapid growth of the fetus Answer: C 74. The nurse assesses the woman at 20 weeks gestation and expects the woman to report: a. Spotting related to fetal implantation b. Symptoms of diabetes as human placental lactogen is released c. Feeling fetal kicks d. Nausea and vomiting related HCG production Answer: C Rationale: Quickening is felt by 16 to 20 weeks AOG. 75. Mrs. Medina comes to you for check-up on June 2, her EDC is June 11. What do you expect during assessment? a. Fundic height of 2 fingers below xyphoid process, engaged b. Cervix close, uneffaced, FH-midway between the umbilicus and symphysis pubis c. Cervix open, fundic height 2 fingers below xyphoid process, floating d. Fundic height at least at the level of the xyphoid process, engaged Answer: A Rational: Around this time, the fetus has already engaged (dropped into the maternal true pelvis; otherwise known as lightening) as indicated by the fundus located 2 finger breadths below the xiphoid process. Situation 10: A professional nurse should know how to respond in these varied health situations. 76. RhoGAM is given to Rh-negative women to prevent maternal sensitization from occurring. The nurse is aware that in addition to pregnancy, Rh-negative women would also receive this medication after which of the following? a. Unsuccessful artificial insemination procedure b. Blood transfusion after hemorrhage c. Therapeutic or spontaneous abortion d. Head injury from a car accident Answer: C Rationale: Therapeutic or spontaneous abortion causes mixing of the fetal blood (RH+) and maternal blood (Rh-). This could trigger the production of maternal antibodies against the circulating Rh+ blood. The circulating antibodies in the maternal blood will destroy future pregnancies with a Rh+ blood. Rhogam is given 72 hours post delivery to prevent production of maternal antibodies. 77. The breathing technique that the mother should be instructed to use as the fetus' head is crowning is: a. Blowing c. Shallow b. Slow chest d. Accelerated-decelerated Answer: A Rationale: Blowing forcefully through the mouth controls the strong urge to push and allows for a more controlled birth of the head. 78. When providing prenatal education to a pregnant woman with asthma, which of the following would be important for the nurse to do? a. Demonstrate how to assess her blood glucose levels b. Teach correct administration of subcutaneous bronchodilators c. Ensure she seeks treatment for any acute exacerbation d. Explain that she should avoid steroids during her pregnancy Answer: D Rationale: Steroids cause cleft lip/palate in newborns. 79. Which of the following conditions would cause an insulin-dependent diabetic client the most difficulty during her pregnancy? a. Rh incompatibility b. Placenta Previa c. Hyperemesis Gravidarum d. Abruptio Placenta Answer: C Rationale: Both conditions predispose the mother to accumulating high levels of ketone bodies in the blood.

80. Which of the following would the nurse use as the basis for the teaching plan when caring for a pregnant teenager concerned about gaining too much weight during pregnancy?

East West Nursing Review Center

a. 10 pounds per trimester b. 1 pound per week for 40 weeks c. pound per week for 40 weeks d. A total gain of 25 to 30 pounds Answer: D Rationale: To ensure adequate fetal growth and development during pregnancy, a total weight gain 25 to 30 lbs (other books: 25 to 35 lbs) pounds is recommended: 1 lb per month in the first trimester; and 1 lb per week in the last two trimesters. Situation 11: Angel, 25 years old, is a 2-day postpartum client. She delivered a 6 lb 4 oz baby boy. 81. Angel verbalizes her labor and delivery experience, does not appear confident about holding the baby or changing diapers. The nurse identifies the client is in which phase of the postpartum period? a. Letting go c. Holding out b. Taking in d. Taking hold Answer: B Rationale: The taking-in phase is the first postpartum phase. During this phase, the mother feels overwhelmed by the responsibilities of the newborn care and is still fatigued from delivery. 82. When checking Angel's fundus, the nurse observes that the fundus is above the umbilicus and displaced to the right. The nurse evaluates that the client probably has: a. A slow rate of involution b. A full, overdistended bladder c. Retained placental fragments d. Overstretched uterine ligaments Answer: B Rationale: A distended bladder will displace the fundus upward and laterally. 83. During the postpartum period, Angel tells the nurse she is having leg cramps. The nurse should suggest that she should increase her intake of: a. Eggs and bacon c. Juices and water b. Liver and onions d. Cheese and broccoli Answer: D Rationale: The leg cramps may be related to low calcium intake. Cheese and broccoli both have high calcium content. 84. When performing discharge teaching for Angel, the nurse should inform her that: a. The episiotomy sutures will be removed at the first postpartum checkup b. She may not have any bowel movements for up to a week after the birth c. She has to schedule a postpartum checkup as soon as her menses returns d. The perineal tightening exercises started during pregnancy should be continued indefinitely Answer: D Rationale: Kegels exercise can be resumed immediately and should be done for the rest of her lie. 85. Which of the following statements would indicate to the nurse that Angel has begun to integrate her new baby into the family structure? a. All the baby does is cry. He's not like my other child. b. I wish he had curly hair like my husband c. My parents wanted a granddaughter d. When he yawns, he looks like just his brother Answer: D Rationale: Family identification of the newborn is an important part of attachment. The first step in identification is done in terms of likeness to family members. Situation12: Awareness of the complications that may accompany pregnancy is essential in order to render apt nursing management. 86. In which of the following clients would the nurse suspect anemia? a. Client in her first trimester with a hemoglobin level of 12 g/dL b. Client in her second trimester with a hemoglobin level of 11 g/dL c. Client in her third trimester with a hemoglobin level of 8 g/dL d. Client in her first trimester with a hemoglobin level of 10.5 g/dL Answer: C Rationale: Anemia during pregnancy is described as a hemoglobin level of 10 g/dL or less during the second and third trimesters. Thus, the nurse would suspect anemia in the client in her third trimester with hemoglobin of 8 g/dL. Hemoglobin levels of 12 g/dL, 11 g/dL, and 10.5 g/dL are above the cut-off range for the diagnosis of anemia. (Lippincotts Review Series: Maternal-Newborn Nursing, 4th Edition) 87. Which of the following would the nurse identify as a classic sign of PIH? a. Edema of the feet and ankles b. Edema of the hands and face c. Weight gain of 1 lb/week d. Early morning headache Answer: B Rationale: This is the classic sign of PIH.

East West Nursing Review Center

88. Which of the following may happen if the uterus becomes overstimulated by oxytocin during the induction of labor? a. Weak contractions prolonged to more than 70 seconds b. Tetanic contractions prolonged to more than 90 seconds c. Increased pain with bright red vaginal bleeding d. Increased restlessness and anxiety Answer: B Rationale: Hyperstimulation of the uterus such as with oxytocin during the induction of labor may result in tetanic contractions prolonged to more than 90 seconds.

89. Which of the following best describes preterm labor? a. Labor that begins after 20 weeks gestation and before 37 weeks gestation b. Labor that begins after 15 weeks gestation and before 37 weeks gestation c. Labor that begins after 24 weeks gestation and before 28 weeks gestation d. Labor that begins after 28 weeks gestation and before 40 weeks gestation Answer: A Rationale: Preterm labor is best described as labor that begins after 20 weeks gestation and before 37 weeks gestation. (Lippincotts Review Series: Maternal-Newborn Nursing, 4th Edition) 90. The nurse evaluates that the danger of a seizure in a woman with eclampsia subsides: After labor begins After delivery occurs 24 hours postpartum 48 hours postpartum Answer: D Rationale: The danger of a seizure in a woman with eclampsia subsides when postpartum diuresis has occurred, usually 48 hours after delivery. The risk for seizures may remain for up to two weeks after delivery. (Mosbys Comprehensive Review of Nursing for NCLEX-RN Examination, 18th Edition) Situation 13: You are the nurse taking care of a client moving into the active phase of labor. 91. The nurse should include which of the following as priority care? a. Offer support by reviewing the short-pant form of breathing b. Administer a narcotic analgesia c. Allow the mother to walk around the unit d. Watch for the rupture of the membrane Answer: A Rationale: By helping the client use the short pant form of breathing, the nurse can help the client manage her contractions and reduce the need for narcotics and other form of pain relief which can affect the fetal outcome.

92. A client is progressing through the 1st stage of labor. Which finding signals the 2nd stage of labor?
Passage of the mucus plug Bearing down reflex Change in uterine shape Gush of dark blood Answer: B Rationale: 2nd stage of labor is heralded by bearing down reflex with each contraction, increased bloody show, severe rectal pressure and rupture of membrane if this hasnt already occurred. 93. As the nurse assigned to a laboring woman, you are observing the FHR. Which of the following findings would you consider abnormal for a client in active labor? a. A rate of 160 with no significant changes through a contraction b. A rate of 130 with accelerations to 150 with fetal movement c. A rate that varies between 120 and 130 d. A rate of 170 with a drop to 140 during a contraction. Answer: D Rationale: A rate of 170 is suggestive of fetal tachycardia. A drop to 140 during contraction represents some periodic change, which is not normal finding. 94. The client is now 8 cm dilated. To support her during this phase of labor, you should: a. Leave her alone most of the time b. Offer her a back rub during contraction c. Offer her sips of oral fluids d. Provide her with warm blanket Answer: B Rationale: The counterpressure of a back rub during contraction may relive discomfort.

a. b. c. d.

East West Nursing Review Center

95. During the 4th stage of labor, the client should be carefully assessed for:
Uterine atony Complete cervical dilatation Placental expulsion Umbilical cord prolapse Answer: A Rationale: Uterine atony should be carefully assessed during the 4th stage of labor. 2nd stage Begins with complete cervical dilatation and ends with birth 3rd stage Begins immediately after birth and ends with expulsion of the placenta Situation 14: As the fetus grows and hormone shift during pregnancy, physiologic adaptations occur in every body system to accommodate the fetus. 96. The nurse suggests breast pumping to relieve clients breast engorgement. Which instruction should the nurse provide? a. Pump each breast 5 to 10 minutes every 3 to 4 hours round the clock b. Pump each breast for at least 10 minutes every 3 to 4 hours, pump at night only if awake c. Pump each breast for no more than 10 minutes every 2 hours round the clock d. Pump each breast for 10 minutes every 2 hours, skip one pumping at night Answer: B Rationale: To relieve engorgement, she should try to pump 10 min for 24 hrs. Pumping every 3 to 4 hours allows milk to build up. Waking up at night just to pump interferes with mothers rest. 97. Weng complains of constipation. The nurse should explain that constipation frequently occurs during pregnancy because of: a. Pressure of the growing uterus on the anus b. Increased intake of milk as recommended during pregnancy c. The slowing of peristalsis in the GIT d. Changes in the metabolic rate Answer: C Rationale: The growing uterus exerts pressure on the mesentary slowing peristalsis; more water is reabsorbed, and constipation results. 98. Jane is concerned about the mask of pregnancy, the dark nipples and the dark line from her navel to her pubis. The nurse explains that these adaptations are caused by the hyperactivity of the: Adrenal gland c. Ovaries Thyroid gland d. Pituitary gland Answer: D Rationale: Pigmentation is caused by the anterior pituitary hormone, melanotropin, which increases during pregnancy. 99. Jane complains of morning sickness. The nurse realizes that a predisposing factor that causes morning sickness during the first trimester of pregnancy is the adaptation to increased level of: a. Estrogen c. Luteinizing hormone b. Progesterone d. Chorionic gonadotropin Answer: D Rationale: Chorionic gonadotropin, secreted in large amount by the placenta during gestation, and the metabolic changes associated with pregnancy can precipitate N/V in early pregnancy. 100. During pregnancy, what happens to the heart from displacement of the diaphragm? It moves: Upward and to the left Upward and to the right Downward and to the left Downward and to the right Answer: A Rationale: During pregnancy, the heart is displaced upward and to the left from pressure of the pregnant uterus on the diaphragm. (Straight As in Maternal-Neonatal Nursing, 2nd Edition) Situation 15: The process of data gathering and analysis is ongoing. The nurse, therefore, should focus on trimester-specific issues. 101. Fetal heart rate can be auscultated with a fetoscope as early as which of the following? a. 5 weeks gestation b. 10 weeks gestation c. 15 weeks gestation d. 20 weeks gestation Answer: D Rationale: The FHR can be auscultated with a fetoscope at about 20 weeks gestation. FHR can be heard using a Doppler at 10-12 weeks gestation. FHR cannot be heard any earlier than 10 weeks gestation. (Lippincotts Review Series: Maternal-Newborn Nursing, 4th Edition) 102. An ultrasound is typically performed during the third trimester for which of the following reasons? a. To evaluate the fetus for possible congenital anomalies b. To determine the fetal position and estimate fetal size c. To confirm the suspicion of possible multiple gestation

a. b. c. d.

East West Nursing Review Center

d. To enhance prenatal testing and evaluation of pelvic mass Answer: B Rationale: Ultrasound is typically performed during the third trimester to determine feta position and estimate fetal size. 103. Quickening in primigravidas usually can be detected during which of the following weeks of gestation? a. 10 to 14 weeks b. 15 to 17 weeks c. 18 to 20 weeks d. 20 to 22 weeks Answer: C Rationale: Quickening is typically described as a light fluttering feeling, can usually be detected between 18 and 20 weeks gestation in primigravidas. However, in multigravidas, fetal movement can be detected as early as 16 weeks gestation. (Lippincotts Review Series: Maternal-Newborn Nursing, 4th Edition)
104. At 12 weeks gestation, what fetal development would you expect to find? Eyelids are open Insulin is present in the pancreas Vernix caseosa is copious Subcutabneous fat increases Answer: B Rationale: At 12 weeks gestation, the pancreas secretes insulin. 105. During prenatal development, fetal weight gain is greatest in the: First trimester Third trimester Second trimester Implantation period Answer: B Rationale: This is the period in which the fetus stores deposits of fat. Situation 16: The nurse bears responsibilities during the postpartum stage, wherein the mother undergoes physical and psychological adjustment to the process of childbearing. 106. While bottle feeding her neonate, a post partum client asks the nurse when she can expect her menstrual period to return. How should the nurse respond? a. In 1-2 weeks c. In 7-9 weeks b. In 3-4 weeks d. In 10-12 weeks Answer: C Rationale: In non-lactating clients, menstruation typically resumes 7-9 weeks after delivery. The average time before return of ovulation is about 10 weeks after delivery. 107. When assessing lochia serosa, which of the following would the nurse expect? a. Creamy yellow color b. Characteristic odor c. Serosanguineous appearance d. White to colorless Answer: C Rationale: Lochia serosa appears as pink and brownish, serosanguineous discharge with a strong odor occurring from 3 to 10 days after delivery. Lochia alba is typically almost colorless to creamy yellowish discharge occurring from 10 to 21 days after delivery. Lochia rubra appears as dark red vaginal discharge with a characteristic odor, occurring in the first 2 days after delivery. (Lippincotts Review Series: Maternal-Newborn Nursing, 4th Edition)

108. Which behavior would the postpartum client demonstrate during the taking-in phase? Passive dependent role Increased energy Receptiveness to self-care education Increased responsibility for neonate Answer: A Rationale: During the taking-in phase, the client typically demonstrates a passi2ve-dependent role, directing energy toward herself, instead of the neonate. 109. Which statement about nutrition in the postpartum period is true? a. The client should maintain a high-carbohydrate diet b. The client should expect a decrease in thirst c. The client should eat low-fiber food d. The client should increase protein and caloric intake Answer: D Rationale: The client should increase protein and caloric intake not necessarily increased carbohydrates. The nurse working on the postpartum unit should encourage clients to ambulate early to:

110.

East West Nursing Review Center

Promote respirations Increase the tone of the bladder Maintain tone of abdominal muscles Increase peripheral vasomotor activity Answer: D Rationale: There is extensive activation of the blood clotting factor after delivery. This, together with immobility, trauma, or sepsis, encourages thromboembolization, which can be limited through activity. Situation 17: Nutrition is an important aspect during pregnancy. It has a direct bearing on her health and on the fetal growth and development. Nurses can help guide pregnant women in planning a good diet. 111. The nurse in the prenatal clinic should provide nutritional counseling to all newly pregnant women because: a. Most weight gain during pregnancy is fluid retention b. Dietary allowances should not increase during pregnancy c. Pregnant women must adhere to a specific pregnancy diet d. Different sources of essential nutrients are favored by different cultural groups Answer: D Rationale: The nurse should become informed about the cultural eating patterns of clients so that foods containing essential nutrients, which are part of these dietary patterns, will be included. 112. Nutritional planning for a newly pregnant woman of average height weighing 145 pounds should include: A decrease of 200 calories per day An increase of 300 calories per day An increase of 500 calories per day A maintenance of her present caloric intake Answer: B Rationale: This is the recommended caloric increase for adult women to meet the increased metabolic demands of pregnancy. 113. A primigravida woman in her 10th week of gestation is concerned because she has read that nutrition during pregnancy is important for the growth and development of the fetus. She wants to know something about the food she should eat. The nurse should: a. Instruct her to continue eating a normal diet b. Assess what she eat by taking a diet history c. Give her a list of food to help her better plan her meals d. Emphasize the importance of limiting salt and highly seasoned food Answer: B Rationale: By taking a diet history, the nurse can assess the woman's level of nutritional knowledge and gain clues for appropriate methods of counseling. 114. A client is concerned about gaining weight during pregnancy. The nurse explains that the largest part of weight gain during pregnancy is because of: The fetus Fluid retention Metabolic alterations Increased blood volume Answer: A Rationale: The average weight gain is 25 to 35 lbs. Of this, the fetus accounts for 7 to 8 lbs, or approximately 30% of weight gain. 115. A patient who is 20 weeks pregnant tells the nurse that her prenatal vitamin makes her sick. Which is the best suggestion? a. Take the vitamin with breakfast b. Stop taking the vitamin for a few days and then resume c. Take the vitamin at bedtime d. Take the vitamin with milk between meals Answer: C Rationale: When prenatal vitamins cause a bit of nausea, it is suggested that patients take the medication at bedtime so they may experience less nausea when they sleep. Situation 18: A nurse is taking care of a client who is scheduled for cesarean delivery. 116. Which assessment finding would indicate a need to contact the physician? a. Fetal heart rate of 180 bpm b. WBC count of 12,000 cells/mm3 c. Maternal pulse rate of 85 bpm d. Hemoglobin of 11.0 g/dL Answer: A Rationale: A normal fetal heart rate is 120 to 160 bpm. A count of 180 bpm could indicate fetal distress and would warrant physician notification. 117. A client in labor is transported to the delivery room and is prepared for a cesarean delivery. The

East West Nursing Review Center

nurse places the client in the: a. Trendelenburgs position with the legs in stirrups b. Semi-fowler position with a pillow under the knees c. Prone position with the legs separated and elevated d. Supine position with a wedge under the right hip Answer: D Rationale: Vena cava and descending aorta compression by the pregnant uterus impedes blood return from the lower trunk and extremities. This leads to decreasing cardiac return, cardiac output, and blood flow to the uterus and subsequently the fetus. The best position to prevent this would be side-lying with the uterus displaced off the abdominal vessels. 118. A nurse prepares to auscultate the fetal heart rate by using a Doppler ultrasound device. The nurse most accurately determines that the fetal heart sound are heard by: a. Noting if the heart rate is greater than 140 bpm b. Placing the diaphragm of the Doppler on the mothers abdomen c. Performing Leopolds maneuver first to determine the location of the fetal heart d. Palpating the maternal radial pulse while listening to the fetal heart rate Answer: D Rationale: The nurse simultaneously should palpate the maternal radial or carotid pulse and auscultate the fetal heart rate to differentiate the two. If the fetal and maternal heart rates are similar, the nurse may mistake the maternal hear rate for the FHR. 119. After delivery, the nurse is assessing the mother for signs and symptoms of superficial venous thrombosis. Which of the following signs and symptoms would the nurse note if superficial venous thrombosis were present? Paleness of the calf area Enlarged, hardened veins Coolness of the calf area Palpable dorsalis pedis pulses Answer: B Rationale: Thrombosis of superficial veins usually is accompanied by signs and symptoms of inflammation. These include swelling of the involved extremity and redness, tenderness and warmth. (Saunders, 3rd Edition) 120. A nurse has provided discharge instructions to a client who delivered healthy newborn infant by cesarean delivery. Which statement if made by the client indicates a need for further instructions? a. I will notify the physician if I develop a fever. b. I will lift nothing heavier than the newborn infant for at least 2 weeks. c. I will begin abdominal exercises immediately. d. I will turn on my side and push up with my arms to get out of bed. Answer: C Rationale: Abdominal exercise should not start immediately following abdominal surgery, and the client should wait at least 3 to 4 weeks post-operatively to allow healing of the incision. (Saunders, 3 rd Edition) Situation 19: The following questions pertain to care of womens reproductive health. 121. For women aged 19-39 years, recommended health screening diagnostic testing includes which of the following? a. Pap smear b. Mammography c. Cholesterol and lipid profile d. Bone mineral density testing Answer: A Rationale: A Pap smear is recommended for women aged 19-39 years, as well as for women aged 40 and older. 122. The nurse teaches the female patient who is premenopausal to perform breast self-examination (BSE): a. Any time during the month b. With the onset of menstruation c. On day 5 to day 7, counting the first day of menses as day 1 d. On day 2 to day 4, counting the first day of menses as day 1 Answer: C Rationale: BSE is best performed after menses, when less fluid is retained. 123. The nurse is caring for a 38-year-old unmarried woman. She is 38 weeks pregnant and has two school-age children at home. She has gained 4 pounds in the last week and she states her rings are tight and wont come off. Her significant other, who is not the father of this baby or the other children, attends prenatal clinic with her to provide support and share information. Based on the philosophy of family-centered maternal-newborn nursing, the following principles apply to the care of this family EXCEPT: a. Pregnancy and childbirth are usually normal, healthy events in the family b. Pregnancy and childbirth affect the entire family

East West Nursing Review Center

c. Families are able to make decisions about care if given the proper information d. Personal and cultural attitudes influence the meaning of pregnancy and birth in the family Answer: A Rationale: The statement pregnancy and childbirth are usually normal, healthy events within the family does not apply because the mother has a weight gain of 4 pounds, which is excessive for this time of pregnancy, and states that her rings are tight and wont come off. In this non-traditional family, the applicable philosophy of family-centered maternal-newborn nursing includes the statements in B, C and D. (Lippincotts Review Series: Maternal-Newborn Nursing, 4th Edition) 124. Stage 3 of breast development, according to Tanner, occurs when: a. Breast budding begins b. The breast develops into a single contour c. The areola (a darker tissue ring around the nipple) develops d. The areola and nipple form a secondary mound on top of breast tissue Answer: C Rationale: Stage 3 further enlargement of breast tissue and the areola (a darker tissue around the nipple) A. Stage 2 Breast budding (the first sign of puberty in a female) B. In stage 5 female demonstrates continued development of a larger breast with a single contour. D. In stage 4 the nipple and areola form a secondary mound on top of breast tissue Note: Stage 1 describes the prepubertal breast (Brunner and Suddarth's Medical-Surgical Nursing, 11th Edition)

125. In counseling the couple about the various Family Planning Methods, the nurse identifies that the most important factor in choosing a contraceptive method is: a. Financial expense b. Compliance with cultural expectations c. Non-contraceptive benefits d. Correct and consistent use Answer: D Rationale: To achieve the maximum effectiveness from a contraceptive, it must be used correctly and consistently. Situation 20: All maternal body systems are altered by pregnancy. The following questions pertain to the signs and symptoms of pregnancy. 126. Cervical softening and uterine souffl are classified as which of the following? a. Diagnostic signs c. Probable signs b. Presumptive signs d. Positive signs Answer: C Rationale: Cervical softening (Goodell sign) and uterine souffl are two probable signs of pregnancy. Probable signs are objective findings that strongly suggest pregnancy. Which of the following would the nurse identify as a presumptive sign of pregnancy? a. Hegars sign b. Nausea and vomiting c. Skin pigmentation changes d. Positive serum pregnancy test Answer: B Rationale: Presumptive signs are subjective signs. Of the signs listed, only nausea and vomiting are presumptive signs.

127.

128. The nurse documents positive ballottement in the client's prenatal record. The nurse understands that this indicates which of the following? a. Contractions palpable on the abdomen b. Passive movement of the unengaged fetus c. Fetal kicking felt by the client d. Enlargement and softening of the uterus Answer: B Rationale: Ballottement indicates passive movement of the unengaged fetus. A. Ballottement is not a contraction. C. This represents quickening. D. This is known as the Piskacek's sign. 129. The client has completed an at-home pregnancy test with positive results. Which of the following indicates the client understands the meaning of the test results? a. I understand that this means I have ovulated in the past 24 hours. b. I understand that this means I am not pregnant. c. I understand that this means I might be pregnant. d. I understand that this means I am pregnant. Answer: C Rationale: A positive at-home pregnancy test indicates the presence of growing trophoblastic tissue and not necessarily a uterine pregnancy.

East West Nursing Review Center

130. The client has come to the clinic because she suspects that she is pregnant. Which of the following would be the most definitive way to confirm the diagnosis? a. Client's report of amenorrhea for 3 months b. Positive Hegar's sign c. Pigmentation changes of the breast d. Palpation of fetal movement by the care provider Answer: D Rationale: Palpation of fetal movement by the care provider is a positive sign of pregnancy. MEDICAL-SURGICAL NURSING Situation 1: You are the nurse taking care of a 65-year-old client diagnosed with left-sided heart failure. 1. When assessing a patient with left-sided heart failure, you would expect to detect: a. Distended neck veins b. Edema of the lower extremities c. Dyspnea on exertion d. Hepatomegaly Answer: C Rationale: Diminished left ventricular function allows blood to pool in the ventricle and atrium and eventually back up into the pulmonary veins and capillaries. As the pulmonary circulation becomes engorged, rising capillary pressure pushes sodium and water into the interstitial space, causing pulmonary edema leading to dyspnea. (Lippincotts Fluids and Electrolytes Made Incredibly Easy, 4th Edition) 2. The patient receives daily doses of furosemide (Lasix) and digoxin (Lanoxin) for treatment of heart failure. The patient is more likely to develop a toxic reaction to digoxin if he has concurrent: a. Hyponatremia b. Hyperkalemia c. Hypernatremia d. Hypokalemia Answer: D Rationale: Hypokalemia, which can occur with diuretic therapy, may lead to digoxin toxicity. (Lippincotts Fluids and Electrolytes Made Incredibly Easy, 4th Edition) 3. His serum potassium level is 3.1 mEq/L. Which associated electrocardiogram changes would you expect? a. Peaked T wave b. Depressed ST segment c. Narrow QRS complex d. Absent P waves Answer: B Rationale: Hypokalemia causes flattened/inverted T wave, depressed ST segment, and a prominent U wave. (Lippincotts Fluids and Electrolytes Made Incredibly Easy, 4th Edition) 4. As part of the patients treatment for hypokalemia, the doctor prescribes IV potassium supplementation. At which rate should it be administered? a. 5 mEq/hour c. 15 mEq/hour b. 10 mEq/hour d. 20 mEq/hour Answer: B Rationale: When supplemental potassium is given by IV infusion, it should be administered at a rate of 10mEq/hour. (Lippincotts Fluids and Electrolytes Made Incredibly Easy, 4th Edition)

5. The patient calls you to her room because hes short of breath. You assess him and find that his heart failure is worsening. Which type of fluid volume excess is the patient experiencing because of his heart failure? Intravascular c. Intracellular Extracellular d. Interstitial Answer: B Rationale: Because the heart doesnt pump effectively in a patient with heart failure, fluid imbalances develop. The most common fluid imbalance associated with heart failure is extracellular fluid volume excess. This results from the hearts failure to pump blood forward, consequent vascular pooling, and the sodium and water reabsorption triggered by the RAAS. (Lippincotts Fluids and Electrolytes Made Incredibly Easy, 4th Edition) Situation 2: A client, who suffered major burn injures, is rushed to hospital. The patient is a 155-lb male and is estimated at having 50% of his total body surface area burned.

East West Nursing Review Center

6. During the fluid accumulation phase of a major burn injury, fluid shifts from the: a. Intravascular space to the interstitial space b. Interstitial space to intravascular space c. Intracellular space to interstitial space d. Intravascular space to intracellular space Answer: A Rationale: During the fluid accumulation phase, which occurs within the first 24 to 36 hours after a burn injury, fluid shifts from the intravascular space to the interstitial space (third-space shift). (Lippincotts Fluids and Electrolytes Made Incredibly Easy, 4th Edition) 7. You insert an IV line and begin fluid resuscitation. The doctor wants you to use the Parkland formula. What amount of Lactated Ringers solution should you administer over the first 8 hours? a. 700 mL c. 1,400 mL b. 7,000 mL d. 6,000 mL Answer: B Rationale: The formula is 4 mL x TBSA x weight in kg. So, 4 mL x 50% x 70 kg = 14, 000 mL of Lactated Ringers solution in the first 24 hours. Therefore, you would give 7,000 mL (or half) in the first 8 hours. (Lippincotts Fluids and Electrolytes Made Incredibly Easy, 4th Edition) 8. 48 hours after the burn injury, what physiologic changes can be expected? a. Edema development b. Increased blood volume c. Decreased hemoglobin level d. Profuse urination Answer: D Rationale: During remobilization phase, which starts about 48 hours after the initial injury, fluid shifts back to the vascular compartment. Edema to the burn site decreases and blood flow to kidneys increases, which increases diuresis. (Lippincotts Fluids and Electrolytes Made Incredibly Easy, 4th Edition) 9. During the fluid remobilization phase, the nurse would expect to see signs of which electrolyte imbalance? a. Hypokalemia c. Hypernatremia b. Hyperkalemia d. Hypovolemia Answer: A Rationale: Hypokalemia occurs in the fluid remobilization phase as potassium shifts from the extracellular fluid back into the cells. (Lippincotts Fluids and Electrolytes Made Incredibly Easy, 4th Edition) 10. Burn wound sepsis develops and mafenide acetate 10% (Sulfamylon) is ordered BID. While applying Sulfamylon to the wound, it is important for the nurse to prepare the client for expected responses to the topical application which include a. Severe burning pain for a few minutes following application b. Possible severe metabolic alkalosis with continued use c. Black discoloration of everything that comes in contact with this drug d. Chilling due to evaporation of solution from the moistened dressings Answer: A Rationale: Mafenide acetate 10% (Sulfamylon) causes burning on application. An analgesic may be required before the ointment is applied. Situation 3: A 66-year-old woman, who survived a cardiac arrest, was admitted to the intensive care unit. She experienced a prolonged episode of hypotension and is now in acute renal failure. 11. Which type of renal failure did the patient experience? a. Intrarenal c. Postrenal b. Prerenal d. Renal Answer: B Rationale: The patients renal failure was due to hypotension, which is a prerenal cause. Prerenal conditions are those conditions outside of the kidneys that diminish blood flow to the kidneys. (Lippincotts Fluids and Electrolytes Made Incredibly Easy, 4th Edition) 12. Laboratory results associated with acute renal failure include: a. Increased BUN level and decreased serum creatinine level b. Decreased BUN level and increased urine output c. Increased BUN and serum creatinine levels d. Increased BUN level and increased urine output Answer: C Rationale: The patient with ARF has increased BUN and serum creatinine levels, and decreased urine output. (Lippincotts Fluids and Electrolytes Made Incredibly Easy, 4th Edition) 13. Which acid-base imbalance is this patient most likely to experience? Respiratory acidosis Respiratory alkalosis Metabolic acidosis Metabolic alkalosis Answer: C

East West Nursing Review Center

Rationale: As the kidneys lose their ability to excrete hydrogen ions, theres a buildup of hydrogen, which leads to metabolic acidosis. (Lippincotts Fluids and Electrolytes Made Incredibly Easy, 4th Edition) 14. Which of the following is the optimal diet for a patient with renal failure? a. High-calorie, low-protein, low-sodium, low-potassium b. High-calorie, high-protein, high-sodium, high-potassium c. Low-calorie, high-protein, low-sodium, low-potassium d. High-calorie, low-protein, low-sodium, high-potassium Answer: A Rationale: A high-calorie, low-protein, low-sodium, low-potassium is the optimal diet for a patient with renal failure. The patient should follow a high-calorie diet to meet daily requirements and to prevent breakdown of body protein. A low protein diet will reduce end products of protein metabolism that the kidneys are unable to excrete. The diet should also restrict phosphorus sodium, and potassium. (Lippincotts Fluids and Electrolytes Made Incredibly Easy, 4th Edition) 15. While caring for the client with acute renal failure, the nurse should expect that hypertonic glucose, insulin infusions, and sodium bicarbonate will be used to treat which complication? Hyperkalemia c. Hyperlipidemia Hypocalcemia d. Hyponatremia Answer: A Rationale: Hyperkalemia is a common complication of ARF. Glucose administration and regular insulin infusion (with sodium bicarbonate, if necessary) can temporary prevent cardiac arrest by moving potassium into the cells and temporarily reducing potassium levels. Less critical levels of hyperkalemia may be treated with oral sodium polysterene sulfonate. (Straight As in Pathophysiology) Situation 4: Juan is admitted to the hospital with chief complaint of seizures. Client reported reduced urine output and weight gain. He is diagnosed with Syndrome of Inappropriate Diuretic Hormone (SIADH). 16. Which of the following laboratory findings would the nurse expect to find? a. BUN level of 45 mg/dL b. Serum osmolality level of 250 mOsm/kg c. Serum sodium level of 145 mEq/L d. Urine specific gravity of 1.001 Answer: B Rationale: Serum osmolality level will be decreased in SIADH secretion due to fluid retention. (Gingrich, Medical-Surgical Nursing, 2nd Edition)

17. Which nursing diagnosis is most appropriate for Juan who has hyponatremia? a. Risk for injury related to seizure activity b. Impaired skin integrity related to peripheral edema c. Fluid volume excess related to increased thyrotropin secretion d. Impaired gas exchange related to pulmonary edema Answer: A Rationale: Patients with hyponatremia are at high risk for seizures. Nursing interventions should be aimed at safety and protection, including using padded siderails, administering supplemental oxygen, and keeping an oral airway readily available. 18. Juan is thirsty and frequently asks the nurse for water. The most appropriate response would be to: a. Keep adequate water at his bedside b. Give him extra fluids with his medications c. Explain that his fluid intake must be restricted to 27 to 34 oz (800 to 1,000 ml) per day d. Prepare an IV infusion of hypotonic saline Answer: C Rationale: Along with meticulous intake and output, fluid restriction is an important nursing intervention in syndrome of inappropriate antidiuretic hormone (SIADH) to prevent further dilutional hyponatremia. Ice chips may be offered for severe thirst. (Gingrich, Medical-Surgical Nursing, 2nd Edition) 19. Juans treatment plan should include which of the following to combat fluid imbalances with SIADH secretion? a. Hypotonic saline solution b. Fluid restriction c. Colloids d. 5% dextrose solution Answer: B Rationale: SIADH secretion is characterized by excessive amounts of antidiuretic hormone secreted from the posterior pituitary. Key features of antidiuretic hormone excess include water retention, hyponatremia, and low osmolality level. Treatment includes fluid restriction, and administration of hypertonic saline solution. (Gingrich, Medical-Surgical Nursing, 2nd Edition)

East West Nursing Review Center

20. Which sign suggests that a patient with SIADH has developed complications? a. Tetanic complications b. Neck vein distention c. Weight loss d. Polyuria Answer: B Rationale: SIADH causes fluid retention, which can lead to vascular fluid overload signaled by neck vein distention. (Gingrich, Medical-Surgical Nursing, 2nd Edition)

Situation 5: The nurse is discussing the prevention of osteoporosis with a group of clients. 21. Which among the following factors do NOT keep the bones strong? a. An adequate calcium intake b. Maintenance of a low weight c. Sufficient estrogen levels d. Weight-bearing exercises Answer: B Rationale: Low weight is not a requirement for bone strength. It is indicated for persons with osteoarthritis. (NSNA NCLEX-RN Review, 4th Edition) 22. The nurse suggests that the clients diet include adequate amounts of: a. Vitamin A c. Vitamin E b. Vitamin D d. Vitamin K Answer: B Rationale: Vitamin D is essential for normal formation of bones and teeth, and for the absorption of calcium and phosphorus form the GI tract. Sources: saltwater fish, sardines, organ meats, egg yolk (NSNA NCLEX-RN Review, 4th Edition) 23. A woman reports all of the following. Which should the nurse recommend she stop doing to help reduce the risk of osteoporosis? a. Smoking b. Overeating c. Biting her nails d. Skipping breakfast Answer: A Rationale: Smoking causes a decrease in bone density. (NSNA NCLEX-RN Review, 4th Edition) 24. To prevent or treat osteoporosis, adequate calcium intake: a. Is essential throughout the lifespan b. Is only necessary after menopause c. Can only be obtained by supplements d. Is important only until bone density peaks Answer: A Rationale: Adequate calcium intake is essential during childhood, adolescence, and early adulthood to maximize bone density. Later in life, continued calcium intake can minimize bone loss. Dietary intake of calcium-rich foods and calcium-fortified foods is important for all age groups. (Gingrich, MedicalSurgical Nursing, 2nd Edition) 25. Which two hormones regulate calcium and phosphate levels in the blood and stimulate (or inhibit) bone cell activity? Parathyroid hormone and calcitonin Vitamin D and erythropoietin Serotonin and acetylcholine Thyroid hormone and cortisol Answer: A Rationale: Parathyroid hormone is released in response to a negative feedback mechanism that is triggered by an abnormally low serum calcium level. The secretion of parathyroid hormone results in the release of calcium from the bones (stimulation of bone cell activity), conservation of calcium by the kidney, and increased intestinal absorption of calcium. Calcitonin is released in response to increased blood calcium levels and acts to inhibit the release of calcium from bones (inhibition of bone cell activity) and decrease renal reabsorption of calcium. (Chernecky, NCLEX-RN Review Guide) Situation 6: A nurse working in the orthopedic ward is assigned to handle clients with various cases of fracture. 26. Kulas, age 18, is admitted to the hospital with possible fractured tibia. The x-ray shows that the bone is in alignment but a fracture line extends around the ankle. This type of fracture is called a: a. Comminuted fracture b. Colles fracture

East West Nursing Review Center

c. Transverse fracture d. Greenstick fracture Answer: C Rationale: A fracture straight across a bone is a transverse, or linear fracture. (Gingrich, Medical-Surgical Nursing, 2nd Edition) 27. Bebong has a short-leg cast applied. 6 hours later, he complains that pain has increased, especially when his leg is elevated, and that the pain is not relieved by prescription analgesics. The nurse should: a. Instruct him to keep his leg elevated and apply ice continuously b. Call the doctor immediately c. Tell him this is normal and the pain should lessen in 24 to 48 hours d. Give ibuprofen (Motrin) with his pain medication to enhance its effectiveness Answer: B Rationale: A significant increase in pain that is unrelieved by analgesics and worsened by elevation of the extremity indicates compartment syndrome. This is a serious complication caused by bleeding and swelling that must be treated within 6 hours to prevent irreversible ischemia of the leg. The doctor must be notified immediately, and actions must be taken to relieve the pressure. (Gingrich, MedicalSurgical Nursing, 2nd Edition) 28. To decrease tissue pressure and maintain arterial perfusion to the lower arm, the nurse would prepare to assist the doctor in: a. Giving vasodilators IV b. Splitting or removing the cast c. Taking the patient to the operating room d. Elevating the arm on a pole Answer: B Rationale: The primary treatment for compartment syndrome is to relieve the source of the pressure. The cast would be removed or split to relieve the external pressure. (Gingrich, Medical-Surgical Nursing, 2nd Edition) 29. The client in balanced suspension traction needs to be repositioned toward the head of the bed. During repositioning, the nurse should: a. Place sight additional tension on the traction cords b. Release the weights and replace immediately after positioning c. Lift the traction and the client during repositioning d. Maintain the same degree of traction tension Answer: D Rationale: Traction is used to reduce the fracture and must be maintained at all times, including during repositioning. (Straight As Medical-Surgical Nursing, 2nd Edition) 30. Which complication would the nurse suspect when assessing a client with a fractured femur and pelvis who becomes restless, exhibits dyspnea and has petechiae over the chest area and crackles on auscultation? a. Compartment syndrome b. Deep vein thrombosis c. Fat embolism d. Osteomyelitis Answer: C Rationale: Restlessness, dyspnea, chest petechiae, and crackles strongly suggest a fat embolism. The presenting features of fat embolism typically include mental status changes, tachypnea, dyspnea, crackles, wheezes, and large amounts of thick, white sputum. Situation 7: Mang Edgardo, found lying unconscious in an enclosed parking space, is rushed to the emergency room. Carbon monoxide poisoning is suspected. 31. The nurse expects the physician to prescribe which of the following to confirm the diagnosis? a. Carboxyhemoglobin b. Complete blood cell count c. Pulse oximetry d. CT scan of the head Answer: A Rationale: The diagnosis of carbon monoxide poisoning is confirmed by the measurement of carboxyhemoglobin levels in the client's blood. (Silvestri, Saunders Comprehensive Review for the NCLEX-RN Examination, 4th Edition) 32. Mang Edgardo suffering from carbon monoxide poisoning: a. Appears intoxicated b. Presents with severe hypertension c. Appears hyperactive d. Will always present with a cherry red skin coloring Answer: A Rationale: A person suffering from carbon monoxide poisoning appears intoxicated (from cerebral hypoxia). Other signs and symptoms include headache, muscular weakness, palpitation, dizziness, and mental confusion.

East West Nursing Review Center

33. A nurse is setting up oxygen for Mang Edgardo. He is to receive oxygen at 10 L per non-rebreather mask. It is important for the nurse to do the following EXCEPT: a. Adjust the flow rate to keep the reservoir bag inflated greater than 2/3 full during inspiration b. Monitor the patient carefully for risk of aspiration c. Make sure the valves and rubber flaps are patent, functional, and not stuck d. Remind the client and his wife of the smoking policy Answer: B Rationale: This is appropriate for a client with a face mask because the face mask limits the clients ability to clear the mouth if vomiting occurs. (NSNA NCLEX-RN Review, 4th Edition) 34. A nurse is monitoring the results of serial arterial blood gases of Mang Edgardo who is asking for the oxygen mask to be removed. The nurse determines that the oxygen may be safely removed once the carboxyhemoglobin level decreases to less than: a. 5% c. 15% b. 10% d. 25% Answer: A Rationale: Oxygen may be removed safely from the client with carbon monoxide poisoning once carboxyhemoglobin levels are less than 5%. (Silvestri, Saunders Comprehensive Review for the NCLEXRN Examination, 4th Edition) 35. A nurse evaluates Mang Edgardo following treatment for carbon monoxide poisoning. The nurse would document that the treatment was effective when the: a. Client is awake and talking b. Carboxyhemoglobin levels are less than 5% c. Heart monitor shows sinus tachycardia d. Client is sleeping soundly Answer: B Rationale: Normal caboxyhemoglobin levels are less than 5% for an adult (0.05 to 2.5 % for a nonsmoker and 5 to 10% for a heavy smoker). (Silvestri, Saunders Comprehensive Review for the NCLEX-RN Examination, 4th Edition) Situation 8: A-75-year-old man with a 10-year history of Parkinsons disease is admitted to the hospital because his condition is deteriorating. 36. The symptom of Parkinsons disease that would be most obvious during the admission assessment is: a. Confusion c. Pallor b. Intention tremor d. Pill rolling Answer: D Rationale: Rhythmic flexion and contraction of the muscles cause a characteristic tremor called a pillrolling tremor. This is characteristic of Parkinsons disease. (NSNA NCLEX-RN Review, 4th Edition)

37. Amantadine hydrochloride (Symmetrel) is prescribed for a client with Parkinsons disease. The client asks how the drug works. In formulating a response, the nurse recalls that the drug: a. Allows accumulation of dopamine b. Corrects mineral deficiencies c. Elevates the clients mood d. Replaces enzymes Answer: A Rationale: Amantadine hydrochloride (Symmetrel) is a synthetic antiviral agent with an unknown mechanism of action that allows dopamine to accumulate in extracellular or synaptic sites. Parkinsons disease is characterized by dopamine deficiency. (NSNA NCLEX-RN Review, 4th Edition) 38. In planning care for the client with severe Parkinsons disease, which of the following is of the highest priority? a. Positioning b. Encouraging independence c. Increasing activity d. Preventing aspiration Answer: D Rationale: Persons with advanced Parkinsons disease usually have difficulty swallowing and are in danger of choking. Aspiration pneumonia must be prevented and is the highest priority. (NSNA NCLEX-RN Review, 4th Edition) 39. Which activity is most likely to be effective in alleviating fatigue? a. Getting him to bed on time b. Avoiding high carbohydrate food c. Collaborating with him when scheduling activities d. Providing for morning and afternoon naps while he is in the hospital Answer: C Rationale: Scheduling activities in collaboration with the client will allow him to proceed at his own pace and maximize his strength. (NSNA NCLEX-RN Review, 4th Edition)

East West Nursing Review Center

40. When does the nurse encourage a client with Parkinsons disease to schedule the most demanding physical activities to minimize the effects of hypokinesia? a. Early in the morning, when the clients energy level is high b. To coincide with the peak action of drug therapy c. Immediately after a rest period d. When family members will be available Answer: B Rationale: Demanding physical activity should be performed during the peak action of dug therapy. Clients should be encouraged to maintain independence in self-care activities to the greatest extent possible.

Situation 9: Brian, 55 years old, is admitted to the oncology ward. He has a history of weight loss, persistent cough that has increased, and blood- tinged sputum for 2 weeks. He has smoked up to two packs of cigarettes a day for 20 years. He is being admitted for further evaluation and treatment. 41. An early sign or symptom or lung cancer seen in the clients history is: Persistent cough c. Weight loss Hemoptysis d. Dyspnea Answer: A Rationale: Unfortunately, the only early symptom of lung cancer is a persistent cough. Many smokers, the largest group of people to have lung cancer, have a chronic cough. Many people do not notice the cough until they begin to cough up blood. 42. An MRI evaluation is scheduled. You prepare Brian for this study by telling him that: a. He will have to take laxatives before the study b. A dye will be injected into his veins just before the test c. No physical preparation is needed before the test d. A nuclear medication is administered by the radiology department 24 hours before the test Answer: C Rationale: A magnetic resonance imaging scan does not require any preparation. The MI is not a radiographic examination but instead uses magnetically stimulated images. 43. Brian is scheduled for a bronchoscopy so that the lesion can be evaluated for biopsy. Preoperative teaching will include an explanation that following the biopsy he will: a. Be unable to talk for several days b. Have nothing by mouth until his gag reflex returns c. Be unable to swallow for 12 hours d. Experience no soreness of the throat Answer: B Rationale: When a client has bronchoscopy, local anesthesia is used in the back of the throat to deaden the gag reflex. Nothing can be taken by mouth until the gag reflex returns so that the client does not choke. It usually takes several hours for this reflex to return. 44. Which of the following would not be a common method of obtaining a specimen to diagnose lung cancer? Thorancentesis Needle biopsy Mediastinoscopy Wedge resection Answer: D Rationale: Specimens can be obtained without surgery. Chest tubes are required if a wedge resection is done. This procedure would be performed if a small tumor had already been diagnosed and needed to be resected. 45. To assist Brian and his family to cope with his diagnosis, the nurse should: a. Explain procedures and their purposes before they are carried out b. Tell him the physician will have to tell him about the tests c. Limit the number of visitors for a few days d. Provide extensive teaching regarding his illness Answer: A Rationale: It is important for the client and family to be well informed of the tests and procedures to be done. Fear of the unknown is one the most anxiety-producing problems for ill client. Situation 10: Pain is considered the fifth vital sign. Clients have the right to appropriate assessment and management of pain. 46. The pain associated with migraine headaches is believed to be caused by: a. Dilation of the cranial arteries

East West Nursing Review Center

A temporary decrease in intracranial pressure Irritation and inflammation of the openings of the sinuses Sustained contraction of muscles around the scalp and face Answer: A Rationale: Vascular disturbance involving branches of the carotid artery is believed to cause migraine attacks. Vasoconstriction of blood vessels apparently occurs first. The extracranial arteries then dilate, causing the headache. 47. When the nurse administers meperidine hydrochloride, its effectiveness as an analgesic is related to its ability to: a. Reduce the perception of pain b. Decrease the sensitivity of pain receptors c. Interfere with pain impulses traveling along sensory nerve fibers d. Block the conduction of pain impulses along the central nervous systems Answer: A Rationale: Opioid analgesics relieve pain by reducing or altering the perception of pain. 48. The client asks the nurse why she has migraine headaches. What is the nurses best response? a. Migraine headaches are believed to be caused by dilation of the cranial arteries b. Migraine headaches are believed to be caused by a temporary decrease in intracranial pressure c. Migraine headaches are believed to be caused by irrigation and inflammation of the opening of the sinuses d. Migraine headaches are believed to be caused by sustained contraction of muscles around the scalp and face Answer: A Rationale: Migraine headaches are believed to be caused by a vascular disturbance involving branches of the carotid artery, where vasoconstriction of blood vessels apparently occurs first. The extracranial and intracranial arteries then dilate, causing the headache. 49. A client, who crashed her motorcycle, suffered a tibial fracture that required casting. Approximately 5 hours later, the client begins to complain of increasing pain distal to the left tibial fracture despite the morphine injection administered 30 minutes previously. The nurses next action should be to assess for which of the following? Presence of a distal pulse Pain with a pain rating scale Vital sign changes Potential for drug tolerance Answer: A Rationale: The nurse should assess the clients ability to move her toes and for the presence of distal pulses, including a neurovascular assessment of the area below the cast. Increasing pain unrelieved by usual analgesics and occurring 4 to 12 hours after the onset of casting or trauma may be the first sign of compartment syndrome, which can lead to permanent damage to nerve and muscles. 50. The nurse teaches the client with chronic cancer pain about optimal pain control. Which of the following recommendations is most effective for pain control? a. Get used to some pain and use a little less medication than needed to keep from being addicted b. Take prescribed analgesics on an around-the-clock schedule to prevent recurrent pain c. Take analgesics only when pain returns d. Take enough analgesics around the clock so that you can sleep 12 to 16 hours a day to block the pain Answer: B Rationale: The regular administration of analgesics provides a consistent breakthrough pain. Therefore, taking the prescribed analgesics on a regular schedule is the best way to manage chronic cancer-related pain. Situation 11: Care of clients undergoing surgery extends until after the surgical procedure is done. Nurses must know how to handle post-op clients. 51. The nurse is caring for a client who has just had a splenectomy. When planning care in the immediate postoperative period the nurse should avoid using which position? Left side-lying c. Semi-Fowlers Right side-lying d. Supine Answer: D Rationale: The supine position allows abdominal organs to rise, permitting less space for lung expansion. With a high abdominal incision, such as used for splenectomy, incisional pain and irritation at the surgical site predispose the client to respiratory complications.

b. c. d.

East West Nursing Review Center

52. The nurse is assessing a client who has had kidney transplant. Which of the following assessment findings would indicate to the nurse that the client might be developing acute rejection of the kidney? a. Oliguia or anuria b. Temperature range of 37.2 at 37.70C c. Decreased blood pressure d. Stabilization of urine and blood chemistry values Answer: A Rationale: Acute rejection of a kidney transplant can be differentiated from chronic rejection. Oliguria or anuria are signs of acute rejection. Signs of rejection include signs of organ failure. 53. A client who is scheduled for a bowel resection tomorrow has just completed preoperative teaching by nurses. Which of the following statements to the nurse indicates the client needs further instruction on postoperative care? a. I know Ill have pain after surgery, but I can call the nurses for medicine. b. They will be taking my pulse and blood pressure many times after the operation. c. The intravenous needle will be removed in the recovery room. d. Ill show you how I can deep breathe and cough. Answer: C Rationale: Intravenous fluids are necessary post-op to maintain fluid and electrolyte balance and as a route for medications. The intravenous infusion will be left in until fluids can be taken by mouth. 54. An adult client is scheduled for a colonoscopy under anesthesia. Which statement by the client indicates he understands the prescribed preparation regimen? a. All I need to do is give myself a packaged enema the morning of the procedure. b. I will eat only jello and drink clear liquids for two days before the test. c. I will take the dye tablets with water the night before the test. d. All I have to do is not eat anything after midnight the night before the test. Answer: B Rationale: The client is given laxatives for 2 days before the procedure and must have only clear liquids during this time. He will be NPO the night before the procedure. 55. Your client has just had major surgery. He refuses to reposition or deep breathe because of fear of pain. He also refuses to take pain medication for fear of addiction. Your best response to this client is to: a. Praise him for wanting to be drug-free b. Inform him that he has to take the pain medication c. Educate him about the benefits of pain relief and risk addiction d. Inform him that the physician is ordering him to take the pain medication Answer: C Rationale: This client is showing lack of understanding related to pain relief and helping him understand the minimal risk of addiction will most likely decrease his fears and increase his compliance level. This decreases the risk of postsurgical complications. Praising the client for wanting to be drugfree may lead him to believe that addiction is a high risk for him. Unless under legal commitment, clients have the right to refuse medications. Situation 12: A client has been diagnosed with adenocarcinoma of the stomach and is scheduled to undergo a subtotal gastrectomy (Billroth II procedure). 56. During the preoperative teaching, the nurse is reinforcing information about the surgical procedure. Which of the following explanations is most accurate? a. The procedure will result in enlargement of the pyloric sphincter b. The procedure will result in anastomosis of the gastric stump to the jejunum c. The procedure will result in removal of the duodenum d. The procedure will result in repositioning of the vagus nerve Answer: B Rationale: A Billroth II procedure bypasses the duodenum and connects the gastric stump directly to the jejunum. The pyloric sphincter is removed, along with some of the stomach fundus. 57. After a subtotal gastrectomy, care of the clients nasogastric tube and drainage system should include which of the following nursing interventions? a. Irrigate the tube with 30 ml of sterile water every hour, if needed b. Reposition the tube if it is not draining well c. Monitor the client for nausea, vomiting and abdominal distention d. Turn the machine to high suction if the drainage is sluggish on low suction Answer: C Rationale: Nausea, vomiting, or abdominal distention indicates that gas and secretions are accumulating within the gastric pouch due to impaired peristalsis or edema at the operative site and may indicate that the drainage system is not working properly. 58. Which of the following systems would be indicative of the dumping syndrome? Hunger c. Diaphoresis Vomiting d. Heartburn Answer: C Rationale: Symptoms of the dumping syndrome usually begin 15 to 30 minutes after eating and include weakness, dizziness, diaphoresis, palpitations, a sense of fullness, abdominal cramps, and diarrhea. These symptoms result when a large bolus of hypertonic fluid enters the small intestine and

East West Nursing Review Center

causes a sudden decrease in plasma volume as fluid is shifted into the bowel. D. Heartburn is the result of gastric reflex, not the dumping syndrome. 59. Which measure helps prevent dumping syndrome? a. Sitting up after meals b. Drinking fluids between meals rather than with meals c. Eating large amounts of carbohydrates d. Eating four to six small low-protein meals during the day Answer: B Rationale: Dumping syndrome (rapid gastric emptying) causes distention of the duodenum or jejunum. Drinking fluids between meals rather than with meals helps to avoid distention. (Straight As in Pathophysiology) 60. Which of the following would be an expected nutritional outcome for a client who has undergone a subtotal gastrectomy for cancer? a. Regain weight loss within 1 month after surgery b. Resume normal dietary intake of three meals a day c. Control nausea and vomiting through regular use of antiemetics d. Achieve optimal nutritional status through oral or parenteral feedings Answer: D Rationale: An appropriate expected outcome is for the client to achieve optimal nutritional status through the use of oral feedings or total parenteral nutrition (TPN). TPN may be used alone if the client cannot tolerate oral feedings. Situation 13: Mang Rolando was long diagnosed with Chronic Renal Failure. You are his nurse and the following questions will assess your knowledge on the different fluid and electrolyte imbalances associated with chronic renal failure. 61. Mang Rolando is scheduled for hemodialysis. The main indicator for the need for hemodialysis is: a. Ascites c. Hypertension b. Acidosis d. Hyperkalemia Answer: D Rationale: Severe hyperkalemia is considered a medical emergency, and is an absolute indication for hemodialysis. It could precipitate cardiac arrhythmias leading to cardiac arrest. Hemodialysis is started immediately to correct this electrolyte abnormality. 62. Mang Rolando misses two sessions of hemodialysis. Blood was drawn and is sent for analysis. Which electrolyte disturbance is expected in a client with CRF? a. Hyponatremia c. Hypomagnesemia b. Hyperkalemia d. Hypercalcemia Answer: B Rationale: Hyperkalemia occurs as the kidney's ability to excrete potassium is impaired. (Fluids and Electrolytes Made Incredibly Easy)

a. b. c. d.

63. In Mang Rolando's ECG tracing, you would expect to find: Depressed T wave Presence of U wave A peaked T wave Inverted T wave Answer: C Rationale: With hyperkalemia, ECG tracings show tall, peaked T waves; a widened QRS complex; and disappearing P waves. (Fluids and Electrolytes Made Incredibly Easy) 64. When the GFR is 30% to 59% of normal during chronic renal failure, the client is experiencing? Reduced renal reserves Renal insufficiency Renal failure End-stage renal disease Answer: B Rationale: Renal insufficiency phase GFR of 30% to 59% of normal. Reduced renal reserves GFR of 60% to 89% or normal Renal failure GFR of 15% to 29% or normal End-stage renal disease GFR Less than 15% of normal (Straight As in Pathophysiology) 65. As Mang Rolando's condition continues to worsen, the client undergoes renal transplant. Mang Rolando is started on Cyclosporine therapy to prevent graft rejection. Which of the following is a major complication of this drug therapy?

a. b. c. d.

East West Nursing Review Center

a. b.

Depression c. Infection Hemorrhage d. Peptic ulcer disease Answer: C Rationale: Cyclosporine inhibits proliferation and function of T-lymphocytes. It places the patient susceptible to opportunistic infections due to cyclosporine-induced immunosuppression. Situation 14: A 74-year-old man with a 3-day history of worsening Chronic Obstructive Pulmonary Disease (COPD) is hospitalized. His breathing is labored; breath sounds are congested with rhonchi throughout; and his SaO2 (as measured by pulse oximetry) is 89%. 66. The client is placed on a 35% aerosol mask, and blood is drawn for arterial blood gas analysis. The results are pH 7.33; PaO2 68 mmHg; PaCO2 53 mmHg, and bicarbonate 18 mEq/L. Which acid-base imbalance does the patient most likely have? Metabolic acidosis Metabolic alkalosis Respiratory acidosis Respiratory alkalosis Answer: C Rationale: When a patients PaCO2 is elevated, carbonic acid is retained leading to acidosis. Because the acidosis is respiratory in origin, the patient most likely has respiratory acidosis. (Lippincott's Fluids and Electrolytes Made Incredibly Easy)

a. b. c. d.

67. When attempting to improve the client's blood gas values through improved ventilation and oxygen therapy, which is the client's primary stimulus for breathing? a. High PCO2 c. Normal pH b. Low PO2 d. Normal HCO3Answer: B Rationale: A chronically elevated PCO2 level (above 50 mmHg) is associated with inadequate response of the respiratory center to plasma carbon dioxide. The major stimulus to breathing now becomes hypoxia (low PO2). (Lippincott's Review Series: Medical-Surgical Nursing. 4th Edition) 68. Why is it important for supplemental oxygen to be carefully monitored in this patient? a. Increasing PaO2 beyond what is needed will lead to oxygen toxicity b. High oxygen levels will promote microbial growth in the patient's lungs c. Increased PaO2 levels can depress the drive to breathe in patients with COPD d. Increased PaO2 levels can elevate the drive to breathe in patients with COPD Answer: C Rationale: Increased PaO2 levels can depress the drive to breathe, which is largely driven by hypoxemia. (Lippincott's Fluids and Electrolytes Made Incredibly Easy) 69. The client is complaining of increased dyspnea. Upon assessment, the client's respiratory rate is 22 breaths per minute. The appropriate nursing action is to: a. Determine the need to increase the oxygen b. Conduct further assessment of the client's respiratory status c. Call a code d. Reassure the client that there is no need to worry Answer: B Rationale: Obtaining further assessment data is the appropriate nursing action. (Silvestri, Saunders Q&A Review for the NCLEX-RN Examination) 70. Why should the nurse who is caring for a client with COPD encourage the client to quit smoking? a. Smoking decreases the amount of mucus production b. Smoking allows hemoglobin to become highly oxygenated c. Smoking shrinks the alveoli in the lungs d. Smoking damages the ciliary cleaning mechanism Answer: D Rationale: Smoking damages the ciliary action in the respiratory tract, which is a protective mechanism. (Chernecky, NCLEX-RN Review Guide)

Situation 15: Jacob, 45 years old, presents to the emergency department with nausea, and steady epigastric pain centered near the navel that radiates to the back. Blood studies reveal elevated amylase, lipase, and while blood cell count levels. The client is diagnosed with acute pancreatitis. 71. The pancreas functions as both an exocrine and endocrine gland. Which of these is an example of its exocrine function? a. The pancreas produces hydrochloric acid b. Amylase is produced in the acinar cells c. Insulin is produced into islets of Langerhans d. The pancreas secretes its enzymes into the stomach

East West Nursing Review Center

Answer: B Rationale: The production of amylase in the acinar cells is an example of exocrine function. A, C and D are endocrine functions. (Lippincotts Fluids and Electrolytes Made Incredibly Easy) 72. The most common cause of acute pancreatitis is: a. Alcohol b. Eating low-fat foods c. Gallstones d. Pregnancy Answer: C Rationale: Gallstones are the most common cause of acute pancreatitis. A. Alcohol consumption is the second most common cause. (Lippincotts Fluids and Electrolytes Made Incredibly Easy) 73. Which of these imbalances typically occur in acute pancreatitis? a. Hypovolemia b. Hypercalcemia c. Hypernatremia d. Hypermagnesemia Answer: A Rationale: In acute pancreatitis, fluid shifting from the intravascular space into the interstitial spaces and retroperitoneum causes hypovolemia. (Lippincotts Fluids and Electrolytes Made Incredibly Easy) 74. The patient with acute pancreatitis may report that his pain decreases: a. When he lies on his stomach b. After vomiting c. After eating a large meal d. When he lies on his side with his knees drawn toward his chest Answer: D Rationale: Pain caused by acute pancreatitis is commonly relieved when the patient lies on his side with his knees drawn toward his chest. (Lippincotts Fluids and Electrolytes Made Incredibly Easy) 75. Patients recovering from acute pancreatitis should eat foods that are: a. Low in carbohydrates, and high in fats and proteins b. Low in carbohydrates, proteins, and fats c. High in carbohydrates and fats, and low in proteins d. High in carbohydrates, and low in fats and proteins Answer: D Rationale: The patient recovering from acute pancreatitis should eat foods that are high in carbohydrates and low in fats and proteins. (Lippincotts Fluids and Electrolytes Made Incredibly Easy) Situation 16: Aguada, a client with leukemia, is in a clinic for her routine check-up. 76. Which of the following is unlikely when assessing Aguada? Small abdomen Bruises and petechiae Increased WBC count Dyspnea during exercise Answer: A Rationale: A client with leukemia has a distended abdomen due to enlarged liver and spleen. (Brunner and Suddarth's Textbook of Medical-Surgical Nursing, 11th Edition) 77. The most appropriate examination for Aguada is: RBC count c. CBC WBC count d. BMA Answer: D Rationale: In bone marrow aspiration, the soft tissue contained in the medullary canals of long bones and interstices of cancellous bones is removed under local anesthesia. Hematologic analysis of the bone marrow specimen that reveals positive for leukemic blast phase cells is diagnostic of leukemia. (Brunner and Suddarth's Textbook of Medical-Surgical Nursing, 11th Edition) 78. In order to maintain oral hygiene, the client should be encouraged to: Use regular toothbrush Gargle with mouthwash Use cotton pledget only Use soft toothbrush Answer: D Rationale: Soft-bristled toothbrushes should be used to prevent bleeding (Brunner and Suddarth's Textbook of Medical-Surgical Nursing, 11th Edition) 79. Which of the following are the three main consequences of leukemia? a. Bone deformities, infection and anemia b. Anemia, infection and bleeding tendencies c. Leukopenia, thrombocytopenia and anemia d. Leukoctytosis, thrombocytopenia and polycythemia

a. b. c. d.

a. b.

a. b. c. d.

East West Nursing Review Center

Answer: B Rationale: Leukemia is characterized by unregulated proliferation of immature WBCs in the bone marrow (leukocytosis). The proliferation of leukemic cells leaves little room for normal cell production resulting to reduced production of RBCs, hematocrit, hemoglobin, and platelets (thrombocytopenia). These cause anemia, and place the client at risk for infection and bleeding. (Brunner and Suddarth's Textbook of Medical-Surgical Nursing, 11th Edition) 80. Aguada experiences nasal bleeding. The client should be instructed to: a. Lie supine with her neck extended b. Sit upright, leaning slightly forward c. Blow her nose and then put lateral pressure on her nose d. Hold her nose while bending forward at the waist Answer: B Rationale: For epistaxis, instruct the client to sit upright with the head tilted forward to prevent swallowing and aspiration of blood. (Brunner and Suddarth's Textbook of Medical-Surgical nursing, 11th Edition) Situation 18: You are the nurse assigned to care for a client diagnosed with Peptic Ulcer Disease. 81. The client asks the nurse what causes a peptic ulcer. You appropriately respond that recent research indicates that many peptic ulcers are the result of which of the following? Work-related stress Helicobacter pylori infection Diets high in fat A genetic defects in the gastric mucosa Answer: B Rationale: Most peptic ulcers are caused by Helicobacter pylori, which release toxins that destroy the gastric and duodenal mucosa. (Straight As in Pathophysiology) 82. A client with peptic ulcer disease tells you that he has black stools, which he has not reported to his physician. Based on this information which nursing diagnosis would be appropriate for this client? a. Ineffective Coping related to fear of diagnosis of chronic illness b. Deficient Knowledge related to unfamiliarity with significant signs and symptoms c. Constipation related to decreased gastric motility d. Imbalanced Nutrition: Less than Body Requirements related to gastric bleeding Answer: B Rationale: Black, tarry stools are an important warming sign of bleeding in peptic ulcer disease. Digested blood in the stool causes it to be black. The odor of the stool is very offensive. Clients with peptic ulcer disease should be instructed to report to incidence of black stools promptly to their primary healthcare provider. 83. Which of the following would be an expected outcome for a client with peptic ulcer disease? a. The client will demonstrate appropriate use of analgesics to control pain b. The client will explain the rationale for eliminating alcohol from the diet c. The client will verbalize the importance of monitoring hemoglobin and hematocrit every 3 months d. The client will eliminate contact sports from his or her lifestyle Answer: B Rationale: Alcohol is a gastric irritant that should be eliminated from the intake of the client with peptic ulcer disease. 84. You are preparing to teach a client with a peptic ulcer about the diet that should be followed after discharge. You should explain that the diet will most likely consist of which of the following? Bland foods High-protein foods Any foods that are tolerated Large amounts of milk Answer: C Rationale: The client can eat three regular meals a day. Specific dietary restrictions vary from client to client. (Lippincotts Review Series: Medical-Surgical Nursing, 4th Edition) 85. Which instruction would be included in the teaching plan for the client taking antacids? a. Take the antacid with 8 oz of water. b. Avoid taking other medications within 2 hours of this one. c. Continue taking antacids even when pain subsides. d. Weigh yourself daily when taking this medication. Answer: B Rationale: Antacids neutralize gastric acid and decrease the absorption of other medications. The client should be instructed to avoid taking other medications within 2 hours of the antacid. (Lippincotts Review Series Medical-Surgical Nursing, 4th Edition) Situation 19: Cancer of the prostate is the leading cancer in men other than skin cancer. The

East West Nursing Review Center

following questions will assess your knowledge and theoretical foundation in dealing with clients with prostate cancer. 86. Among the following population groups, who has a higher risk in the development of prostate cancer? a. African-American c. Asian b. Caucasian d. Hispanics Answer: A Rationale: The worldwide incidence of prostate cancer is highest in African American men, which may be related to their lower engagement in the health care system, disparities in health care, and cultural and structural constraints. (Brunner and Suddarth's Textbook of Medical-Surgical Nursing, 11th Edition)

a. b. c. d.

87. Which among the following is NOT a risk factor for prostate cancer? A family member with prostate cancer Advancing age Diet high in fat and red meats Smoking Answer: D Rationale: Smoking is not a risk factor for prostate cancer. (Brunner and Suddarth's Textbook of Medical-Surgical Nursing, 11th Edition) 88. At the initial sign and symptoms of prostate cancer, before diagnosis, the physician can perform a screening test to detect a characteristic stony hard prostate and nodules at the prostate area using: a. Cystoscopy c. DRE b. PSA d. MRI Answer: C Rationale: With the use of routine repeated rectal palpation of the gland, early cancer may be detected as a nodule within the substance of the gland or as an extensive hardening in the posterior lobe. The more advanced lesion is stony hard and fixed. (Brunner and Suddarth's Textbook of MedicalSurgical Nursing, 11th Edition) 89. To diagnose the presence of prostate cancer, the physician will perform: a. Transrectal needle biopsy of the prostate b. Test to identify the PSA level c. Transurethral ultrasound d. Radiolabeled monocional antibody capromab pendetide with iridium-111 Answer: A Rationale: The diagnosis of prostate cancer is confirmed by a histologic examination of the tissue removed surgically by transurethral resection, open prostatectomy, or transrectal needle biopsy. (Brunner and Suddarth's Textbook of Medical-Surgical Nursing, 11th Edition) 90. In testing for the PSA, the nurse will collect which specimen? a. Blood c. Feces b. Urine d. Prostatic fluid Answer: A Rationale: PSA is a nuetral serine protease produced by both normal and neoplastic ductal epithelium of the prostate. By measuring the amount of this antigen in the blood, it is possible to detect prostate cancer. (Brunner and Suddarth's Textbook of Medical-Surgical Nursing, 11th Edition) Situation 20: You are the nurse caring for a client with hyperthyroidism, who is scheduled for a subtotal thyroidectomy. 91. The physician has ordered Lugols solution for the client. You understand that the primary reason for giving Lugols solution preoperatively is to: a. Decrease the risk of agranulocytosis postoperatively b. Prevent tetany while the client is under general anesthesia c. Reduce the size and vascularity of the thyroid d. Potentiate the effect of the other pre-op medications so less medicine can be given while the client is under anesthesia Answer: C Rationale: The client may receive iodine solution (Lugols solution) for 10 to 14 days before surgery to decrease vascularity of the thyroid and thus prevent excess bleeding. (NCNA NCLEX-RN Review, 4th Edition) 92. In planning care for the client post thyroidectomy, you know that it is most important to: a. Carry out range of motion exercises to the neck and shoulders every shift b. Maintain bed rest with client in supine position at all times c. Ask client questions every hour or two to assess for hoarseness d. Provide tracheostomy care every shift and suction PRN to maintain paten airway Answer: C Rationale: Damage to the recurrent laryngeal nerve is a major complication of thyroid surgery. Hoarseness immediately following surgery is often related to intubation during surgery. However, persistent or worsening hoarseness must be reported immediately to the physician because it may be

East West Nursing Review Center

the first sign of nerve injury. (NSNA NCLEX-RN Review, 4th Edition) 93. The client begins to complain of circumoral tingling. You assess a positive Chovsteks sign and a positive Trousseaus sign. You understand that the most common cause of these symptoms is which of the following? a. Inadvertent removal of the parathyroid glands b. Overuse of radioactive iodine given preoperatively c. History of insufficient iodine intake d. Overstimulation of parathormone during surgery Answer: A Rationale: The symptoms suggest hypocalcemia. The four pea-seized parathyroid glands, which regulate calcium and phosphorous balance, are imbedded in the thyroid. Inadvertent removal during thyroidectomy is a common cause of postoperative hypocalcemia. (NSNA NCLEX-RN Review, 4th Edition) 94. Following a thyroidectomy, the client experiences hemorrhage. You would prepare for which emergency intervention? IV administration of calcium Insertion of an oral airway Creation of a tracheostomy IV administration of thyroid hormone Answer: C Rationale: Following a thyroidectomy, postoperative hemorrhage may cause compression of the trachea, necessitating an emergency tracheostomy to maintain airway patency. (Lippincotts Medical-Surgical Nursing, 4th Edition) 95. Twelve hours post thyroidectomy, the client develops stridor on exhalation. What is your best first action? a. Reassure the client that the voice change is temporary b. Document the finding as the only action c. Hyperextend the clients neck d. Call for emergency assistance Answer: D Rationale: Stridor on exhalation is the hallmark sign of respiratory distress, usually caused by obstruction resulting from tissue edema. A tracheostomy set is usually kept at the bedside in case of such emergencies, and the physician is summoned at the first indication of respiratory distress. (Chernecky, NCLEX-RN Review Guide) Situation: Mr. Calvo is admitted to your ward. The physician ordered Prepare for thoracentesis this pm to remove excess air from the pleural cavity. 96. Which of the following nursing responsibilities is essential for Mr. Calvo who will undergo thoracentesis? a. Support and reassure client during the procedure b. Ensure that informed consent has been signed c. Determine if client has allergic reaction to local anesthesia d. Ascertain if chest x-rays and other tests have been prescribed and completed Answer: D Rationale: Chest x-ray done before the procedure is essential to serve as a baseline data, in order to have a basis for comparison with the chest x-ray done post-procedure. 97. Mr. Calvo, who is for thoracentesis, is positioned by the nurse to which of the following? a. Trendelenburg position b. Supine position c. Dorsal Recumbent position d. Orthopneic position Answer: D Rationale: Patients for thoracentesis should be placed on a straddling position with the arms placed on the back of the chair, or on sitting position leaning forward (orthopneic), with arms placed over a desk or bedside table. If a client cannot sit up, the client is placed lying on bed on the unaffected side with the head of the bed elevated at 45 degrees. 98. During thoracentesis, which of the following nursing interventions will be most crucial? a. Place patient in a quiet and cool room b. Maintain strict aseptic technique c. Advice patient to sit perfectly still during needle insertion until it has been withdrawn from the chest d. Apply pressure over the puncture site as soon as the needle is withdrawn Answer: C. Rationale: The patient should be informed not to cough, move or breathe deeply during the procedure, to prevent trauma to the lungs. 99. To prevent leakage of fluid in the thoracic cavity, how will you position the client after thoracentesis? Place flat in bed Turn on the unaffected side

East West Nursing Review Center

Turn on the affected side On bed rest Answer: B Rationale: Client should be placed on the opposite side (unaffected side) for 1 hour to promote lung expansion. 100. Chest x-ray was ordered after thoracentesis. When your client asks what is the reason for another chest x-ray, you will explain: To rule out pneumothorax To rule out any possible perforation To decongest To rule out any foreign body Answer: A Rationale: Mr. Calvo is admitted to the hospital due to presence of air in the lungs (pneumothorax). Postthoracentesis, and x-ray is necessary to rule out the presence of air in his lungs.

PSYCHIATRIC NURSING Situation 1. Psychiatric Nursing, as a profession, makes use of a theoretical body of knowledge. A competent psychiatric nurse must be knowledgeable of the different concepts in Psychiatric Nursing. 131. A nurse is aware that extremely depressed patients seem to do best in settings where they have: a. Multiple stimuli b. Varied Activities c. Minimal decision making d. Routine Activities Answer: D Rationale: Depression usually is both emotional & physical. A simple daily routine is the best, least stressful and least anxiety producing. 132. A nurse is caring for a patient with delirium and states Look at the spiders on the wall. What should the nurse respond to the patient? a. Youre having a hallucination, there are no spiders in this room at all b. I can see the spiders on the wall, but they are not going to hurt you c. Would you like me to kill the spiders d. I know you are frightened, but I do not see spiders on the wall Answer: D Rationale: When hallucination is present, the nurse should reinforce reality with the patient. 133. A nurse recognizes that the focus of environmental (milieu) therapy is to: a. Role play life events to meet individual needs b. Allow the clients freedom to determine whether or not they will be involved in activities c. Manipulate the environment to bring about positive changes in behavior d. Use natural remedies rather than drugs to control behavior Answer: C Rationale: Environmental (milieu) therapy aims at having everything in the clients surrounding area toward helping the client.

134. A man is confronted by a situation in which a decision must be made about future behavior. The man is at Kohlberg's conventional stage of moral development when before acting, the man asks himself, If I take this course of action, will I: a. Get into trouble? b. Do what is right? c. Receive a reward? d. Obtain acceptance from others? Answer: D Rationale: Kohlberg's second stage of moral development is called conventional moral development. In conventional morality the person seeks conformity and loyalty. It is based on the personal concordance in that it deals with the reciprocal nature of helping others and receiving approval from others. 135. A patient is using a self-report scale in which she assigns a number to the frequency of dissociative experiences. The nurse correctly identifies this as which type of diagnostic tool? a. Dissociative Disorders Interview Schedule b. Dissociative Experiences Scale c. SCID-D d. Diagnostic Drawing Series Answer: B

East West Nursing Review Center

Rationale: Dissociative Experiences Scale is a brief, self-report scale that measures the frequency of dissociative experiences. The patient quantifies her experience by assigning a number for each item in the scale. Situation 2. Implementing the art of psychiatric-mental health nursing is an important way to convey to patients the caring aspect of nursing. 136. The statement that would best describe the practice of psychiatric nursing would be: a. Ensuring patients legal and ethical rights by acting as a patient advocate b. Helping people with present or potential mental health problems c. Focusing interpersonal skills on people with physical or emotional problems d. Acting in a therapeutic way with people who are diagnosed as having a mental disorder Answer: B Rationale: An important aspect of the role of the psychiatric nurse is primary, secondary, and tertiary interventions to promote emotional equilibrium. (Mosby, 18th Edition) 137. A female patient on the psychiatric unit remains aloof from the other patients. A nurse with whom she has developed a friendly relationship may help her participate in some activity by: a. Finding solitary pursuits that the patient can enjoy b. Speaking to the patient about the importance of entering into activities c. Asking the physician to speak to the patient about participating in activities d. Inviting another patient to take part in a joint activity with the nurse and the patient Answer: D Rationale: Bringing another patient into a set situation would be the most therapeutic, leastthreatening approach. (Mosby, 18th Edition) 138. A nurse is assigned to care for a regressed 19-year-old college student recently admitted to the psychiatric unit with a 1-month history of talking to unseen people and refusing to get out of bed, go to class, or get involved in daily grooming activities. The nurses initial efforts should be directed toward helping the patient by: a. Providing frequent rest periods to avoid exhaustion b. Facilitating the patients social relationships with a peer group c. Reducing environmental stimuli and maintaining dietary intake d. Attempting to establish a meaningful relationship with the patient Answer: D Rationale: The first step in a plan of care should be the establishment of a meaningful relationship because it is through this relationship that the patient can be helped. (Mosby, 18th Edition) 139. A nurse should plan to explain to the adult daughters of a dying patient whose mood changes and apparent anger at them is causing them concern that their mother is: a. Frightened by her impending death b. Working through acceptance of her situation c. Attempting to reduce her familys dependence on her d. Hurt that the family will not take her home to die in her own bed Answer: B Rationale: Understanding the stages leading to the acceptance of death may help the family to understand the patients moods and anger. (Mosby, 18th Edition) 140. A 45-year-old physician is admitted to the psychiatric unit. The patient is restless, loud, aggressive, and resistive during the admission procedure and states, I will take my own blood pressure. The most therapeutic response by the nurse would be: a. Right now, doctor, you are just another patient. b. I am sorry, but I cannot allow that. I must take your BP. c. If you would rather, doctor, Im sure you will do it OK. d. If you do not cooperate, I will get the attendants to hold you down. Answer: B Rationale: This simply states facts without getting involved in role conflict. (Mosby, 18th Edition) Situation 3. Defense mechanisms are mental mechanisms (largely unconscious) that provide initial protection for the personality. They are most helpful in dealing with mild and moderate levels of anxiety. 141. When teaching about child abuse, the nurse includes the fact that the defense mechanism most often used by the physically abusive individual is: a. Manipulation c. Displacement b. Transference d. Reaction formation Answer: C Rationale: Displacement is a defense mechanism in which ones pent-up feelings toward threatening others are discharged on less-threatening others. (Mosby, 18th Edition) 142. A patient being treated in a chemical dependency unit tells a nurse that he only uses drugs when under stress and therefore does not have a substance problem. Which of the following defense mechanism is the patient using? a. Compensation c. Suppression b. Undoing d. Denial Answer: D

East West Nursing Review Center

Rationale: Denial is commonly used by individuals who have substance problems. (Lippincotts Review Series: Mental Health and Psychiatric Nursing, 3rd Edition) 143. One day a male patient with the diagnosis of borderline personality disorder describes a situation that happened at work when his immediate supervisor reprimanded him for not completing an assignment. He explains that it was not his fault and states, people get angry and take it out on me. The nurse recognizes that the patient is using the defense mechanism called: a. Denial c. Displacement b. Projection d. Intellectualization Answer: B Rationale: Attributing unacceptable feelings or attributes to others is the mechanism known as projection; the data demonstrate use of this defense mechanism. (Mosby, 18th Edition) 144. A patient with diabetes is able to discuss in great detail the metabolic process in diabetes while eating a piece of chocolate cake topped with butter frosting. This is an example of the defense mechanism known as: a. Intellectualization c. Displacement b. Dissociation d. Projection Answer: A Rationale: Intellectualization occurs when a painful emotion is avoided by means of a rational explanation that removes the event from any personal significance. (Mosby, 18th Edition) 145. A person released from prison for selling narcotics has been rehabilitated and now works for a youth drug prevention agency. This persons current behavior reflects which of the following defense mechanisms? a. Denial c. Identification b. Displacement d. Sublimation Answer: D Rationale: Sublimation is the defense mechanism whereby an individual substitutes constructive, socially acceptable behavior for strong impulses that are unacceptable. (Lippincotts Review Series: Mental Health and Psychiatric Nursing, 3rd Edition) Situation 4. Concha, a 35-year-old patient with major depression, has been hospitalized for treatment after taking a leave of absence from work. The patients employer expects the patient to return to work following inpatient treatment. 146. During the initial assessment, Nurse Melai would expect the patient to display: a. Elated affect related to reaction formation b. Loose associations related to thought disorder c. Physical exhaustion resulting from decreased physical activity d. Paucity of verbal expression related to slowed thought processes Answer: D Rationale: As depression increases, thought processes become more slowed and verbal expression decreases. (Mosby, 18th Edition) 147. Concha has not verbalized problem areas to staff or peers since admission to the psychiatric unit. Which activity should the nurse recommend to help this patient express herself? a. Art therapy in a small group b. Basketball game with peers on the unit c. Reading a self-help book on depression d. Watching movie with the peer group Answer: A Rationale: Art therapy provides a nonthreatening vehicle for the expression of feelings, and use of a small group will help the patient become comfortable with peers in a group setting. 148. Concha is given prescribed medications and talks with her therapist about her belief that she is worthless and unable to cope with life. Psychiatric care in this treatment plan is based on which framework? a. Cognitive framework b. Behavioral framework c. Interpersonal framework d. Psychodynamic framework Answer: A Rationale: Cognitive thinking therapy focuses on the patients misperceptions about self, others and the world that impact functioning and contribute to symptoms. 149. During the morning rounds, Concha tells the nurse, Im no good. Im a failure. The statement that would be most appropriate for Nurse Melai to use in interviewing the patient whose thoughts focus on feelings of unworthiness and failure would be: a. Tell me how you feel about yourself. b. Tell me what has been bothering you. c. Why do you feel so bad about yourself? d. What can we do to help you during your stay with us? Answer: A

East West Nursing Review Center

Rationale: Since major depression is due to the patients feelings of self-rejection, it is important for the nurse to have the patient identify these feelings before a plan of action can be taken. (Mosby, 18th Edition) 150. Nurse Melai who is caring for Concha knows that the priority nursing intervention is to assess the patient's: a. Response to medication administration b. Current mood and activity level c. Risk of suicide d. Appetite and weight Answer: C Rationale: While it is important for the nurse to assess the patient's areas of functioning, current mood, and fluid/electrolyte balance, assessing the suicide risk of the patient with major depression takes priority. Situation 5. Nurse Agua is working with Mr. Jayson Ivlearn, age 37, who has schizophrenia, paranoid type. 151. Prominent symptoms lasting for at least 1 month that are diagnostic for paranoid schizophrenia are: a. Delusions and hallucinations b. Poverty of speech and apathy c. Disturbed relationships and poor grooming d. Bizarre behaviors associated with drug use Answer: A Rationale: Diagnostic criteria for paranoid schizophrenia include two or more symptoms such as delusions and hallucinations; other less prominent criteria are disorganized behavior and negative symptoms. (Mosby, 18th Edition) 152. As Nurse Agua enters the room and approaches Mr. Ivlearn, the patient states, Get out of here before I hit you! Go away! The nurse recognizes that this patients aggressive behavior was probably related to the fact that he: a. Was hallucinating and the voices were directing his response b. Was afraid that he might harm the nurse if the nurse came nearer c. Was reminded of someone who was frightening and threatening to him d. Felt hemmed in and trapped when the nurse came around the bed toward him Answer: D Rationale: Patients acutely ill with schizophrenia frequently do not trust others; feeling hemmed in would be frightening, causing them to lash out. (Mosby, 18th Edition) 153. Mr. Ivlearn often directs brief, hostile verbal outbursts toward the nursing staff. Which of the following nursing actions is the most therapeutic way to address this problem? a. Administer antipsychotic medications as needed when verbal outburst occur b. Set limits and provide a structured, predictable environment c. Place the client in seclusion when these episodes occur d. Minimize the outbursts by walking away when they occur Answer: B Rationale: Firm, non-punitive limit setting and a structured environment are the best approach to a verbally hostile patient. (Lippincotts Review Series: Mental Health and Psychiatric Nursing, 3rd Edition) 154. Which of the following outcomes related to Mr. Ivlearns delusional perceptions would Nurse Agua establish? a. The patient will demonstrate realistic interpretation of daily events in the unit b. The patient will perform daily hygiene and grooming without assistance c. The patient will take prescribed medications without difficulty d. The patient will participate in unit activities Answer: A Rationale: A patient with schizophrenia, paranoid type, will distort perceptions and view events in the environment as related to people or institutions plotting against him. The outcome related to a realistic interpretation of daily events would establish improvement in the patients perceptual ability. (Lippincotts Review Series: Mental Health and Psychiatric Nursing, 3rd Edition) 155. A week after the admission of Mr. Ivlearn, the patient stands up in the lounge and throws a chair across the room and starts yelling at the other patients. Several of the other patients have frightened expressions, one starts to cry, and another begins to pace. After removing Mr. Ivlearn from the room, Nurse Agua should: a. Arrange a unit meeting to discuss what just happened b. Continue the units activities as if nothing has happened c. Refocus patients negative comments to more positive topics d. Have a private talk with the patients who cried and started to pace Answer: A Rationale: This provides an opportunity for the other patients to voice and share feelings and to identify and separate real from imaginary fears; an open expression of feelings allows the nurse to deal with clients fears and provide reassurance. (Mosby, 18th Edition)

East West Nursing Review Center

Situation 6. Ms. Mytililing Aguila, age 40, is admitted to the psychiatric unit with the DSM-IV diagnosis of bipolar I disorder, manic episode. 156. Which of the following behaviors in Ms. Aguila would be assessed by Nurse Bhoy? a. Apathy, poor insight, poverty of ideas b. Anxiety, somatic complaints, insomnia c. Elation, hyperactivity, impaired judgment d. Social isolation, delusional thinking, clang association Answer: C Rationale: A client with bipolar I disorder, manic episode, would demonstrate symptoms, such as flight of ideas and hyperactivity, as part of the increased psychomotor activity. The mood is one of elation and the feeling is that one is invincible; therefore, judgment may be quite impaired. (Isaacs, Lippincotts Review Series: Mental Health and Psychiatric Nursing, 3rd Edition) 157. When the language of Ms. Aguila becomes vulgar and profane, Nurse Bhoy should: a. State, We do not like that kind of talk around here. b. Ignore it, since the client is using it only to get attention c. Recognize the language as part of the illness, but set limits on it d. State, When you can talk in an acceptable way, we will talk to you. Answer: C Rationale: Recognizing the language as part of the illness makes it easier to tolerate, but limits must be set for the benefit of the staff and other patients. Setting limits also shows the client that the nurse cares enough to stop the behavior. (Mosby, 18th Edition)

158. Ms. Aguila has a superior, authoritative manner and is constantly instructing the other patients on the unit about how to dress, what to eat, and where to sit. These behaviors will eventually make the other patients feel: a. Ambivalent c. Dependent b. Inadequate d. Angry Answer: D Rationale: When people are imposed on by a person with a condescending, bossy attitude they react frequently with feelings of anger in an attempt to decrease their anxiety. (Mosby, 18th Edition) 159. Ms. Aguila is extremely active, talks constantly, and tends to badger the other patients, some of whom are now becoming agitated. The best strategy to use with this patient is: a. Distraction c. Assertiveness b. Sympathy d. Confrontation Answer: A Rationale: During periods of hyperactivity, the patient has a short attention span and can be distracted easily; this is a therapeutic intervention for all the clients. (Mosby, 18th Edition) 160. A nursing diagnosis of Altered thought processes related to difficulty concentrating, secondary to flight of ideas was made. Which of the following outcome criteria would indicate improvement in Ms. Aguila? a. The patient verbalizes feelings directly during treatment b. The patient speaks in coherent sentences c. The patient verbalizes positive self-statements d. The patient reports increased feelings of clam Answer: B Rationale: Flight of ideas occurs when the clients speech flow is continuous and the client jumps from one topic to another. The client who can speak in coherent sentences shows that concentration has improved and thoughts are no longer racing. (Isaacs, Lippincotts Review Series: Mental Health and Psychiatric Nursing, 3rd Edition) Situation 7. Kevin Kosme, age 81, is admitted to a psychiatric hospital with the diagnosis of dementia, Alzheimers type. 161. Nurse Azon recognizes that dementia of the Alzheimers type is characterized by: a. Hypoxia of selected areas of brain tissue b. Areas of brain destruction called senile plaques c. Aggressive acting-out behavior d. Periodic remissions and exacerbation Answer: B Rationale: When an older persons brain atrophies, some unusual deposits of iron are scattered on nerve cells. Throughout the brain, areas of deeply staining amyloid, called senile plaques, can be found; these plaques are end stages in the destruction of brain tissue. (Mosby, 18th Edition) 162. When answering questions from the family of Mr. Kosme with Alzheimers disease, Nurse Azon explains that this disease is: a. A slow, relentless deterioration of the mind b. A functional disorder that occurs in the later years c. A disease that first emerges in the fourth decade of life d. Easily diagnosed through laboratory and psychologic tests Answer: A Rationale: This is a true statement; patients become progressively worse over time. (Mosby, 18th

East West Nursing Review Center

Edition) 163. Mr. Kosme confabulates. Nurse Azon understands that the patient: a. Denies confusion by being jovial b. Pretends to be someone else c. Rationalizes various behaviors d. Fills in memory gaps with fantasy Answer: D Rationale: Confabulation is a communication device used by clients with dementia to compensate for memory gaps. (Lippincotts Review Series: Mental Health and Psychiatric Nursing, 3rd Edition)

164. Mr. Kosme becomes agitated and combative when Nurse Azon approaches to help with morning care. The most appropriate nursing intervention would be: a. To tell the patient firmly that it is time to get dressed b. To obtain assistance to restrain the patient for safety c. To remain calm and talk quietly to the patient d. To call the physician and request order for sedation Answer: C Rationale: It is important to maintain a calm approach when intervening with an agitated client. (Lippincotts Review Series: Mental Health and Psychiatric Nursing, 3rd Edition) 165. Which of the following would Nurse Azon implement for Mr. Kosme who has frequent episodes of emotional lability? a. Attempt humor to alter mood b. Explore reasons for altered mood c. Reduce environmental stimuli to redirect attention d. Use logic in order to point out reality aspects Answer: C Rationale: The client with Alzheimers disease can have frequent episodes of labile mood, which can best be handled by decreasing a stimulating environment and redirecting the clients attention. An over-stimulating environment may cause the labile mood, which will be difficult for the client to understand. (Lippincotts Review Series: Mental Health and Psychiatric Nursing, 3rd Edition) Situation 8. Charice Pipino, a 10-year-old girl, who was diagnosed with autism at the age of 3, attends a school for developmentally disabled children and lives with her parents. 166. When assessing Charice, Nurse Raph should expect which of the following? 1. Imitates others 2. Seeks physical contact 3. Avoids eye-to-eye contact 4. Engages in cooperative play 5. Performs repetitive activities 6. Displays interest in children rather than adults a. 3 and 5 only c. 2 and 6 only b. 1 and 4 only d. 3 only Answer: A Rationale: Qualitative impairments in social interaction are manifested by a lack of eye contact (3), a lack of facial responses, and a lack of responsiveness to and interest in others. In addition, children with autism display obsessive ritualistic behaviors (5), such as rocking, spinning, dipping, swaying, walking on toes, head banging, or hand biting because of their self-absorption and need to stimulate themselves. (Mosby, 18th Edition) 167. When planning activities for Charice, Nurse Raph must remember that autistic children respond best to: a. Large-group activity b. Loud, cheerful music c. Individuals in small groups d. Their own self-stimulating acts Answer: D Rationale: Autistic behavior turns inward. These children do not respond to the environment but attempt to maintain emotional equilibrium by rubbing and manipulating themselves and displaying a compulsive need for behavioral repetition. (Mosby, 18th Edition) 168. Charice is nonverbal and has limited eye contact. To promote social interaction, Nurse Raph initially should: a. Engage in parallel play while sitting next to the child b. Encourage the child to vocalize through sound games and songs c. Provide play opportunities for the child to play with other children d. Use therapeutic holding when the child does not respond to verbal interactions Answer: A Rationale: Entering the childs world in a nonthreatening way helps to promote trust and eventual interaction with the nurse. (Mosby, 18th Edition) 169. Charice has frequent episodes of biting her arms and banging her head and needs help with feeding and toileting. The priority goal for this child would be, The child will:

East West Nursing Review Center

Be able to feed herself. Control repetitive behaviors. Remain safe from self-inflicted injury. Develop control of fecal and urinary elimination. Answer: C Rationale: The priority is safety; the child must be protected from self-harm. (Mosby, 18th Edition) 170. Nurse Raph visits the home of Charice. The parents express feelings of shame and guilt about having somehow caused this problem. Which of the following statements by the nurse would be best to help alleviate parental guilt? a. Autism is a rare disorder. Your other children shouldnt be affected. b. The specific cause of autism is unknown. However, it is known to be associated with problems in the structure of and chemicals in the brain. c. Sometimes a lack of prenatal care can be the cause of autism. d. Although autism is inherited from the genes, if you didnt have testing you could not have known this. Answer: B Rationale: This statement is factual and does not cast blame on anything the parents did or did not do. (Lippincotts Review Series: Mental Health and Psychiatric Nursing, 3rd Edition) Situation 9. Mental Retardation is an increasingly common childhood disorder that impairs learning. 171. Mental retardation is: a. A delay in normal growth and development caused by an inadequate environment b. A lack of development of sensory abilities c. A severe lag in neuromuscular development and motor abilities d. A condition of sub-average intellectual functioning that originates during the developmental period and is associated with impairment in adaptive behavior Answer: D Rationale: Mental retardation refers to below-average general intellectual functioning with associated deficits in communication, social skills, self-care, and adaptive behavior. (Straight As in Psychiatric and Mental Health Nursing) 172. Which of the following is true with regards to mental retardation? a. Mental retardation is always accompanied by physical features b. Hereditary and prenatal factors do not result to mental retardation c. Mental retardation is a mental illness d. Hereditary and prenatal factors are known to result to impaired intellectual functioning Answer: D Rationale: In mental retardation, hereditary and prenatal factors are known to result to impaired intellectual functioning, which includes: 1. Chromosomal disorders: trisomy 21 (Down syndrome), fragile X syndrome, or Klinefelters syndrome. 2. Single dominant gene problems: neurofibromatosis or tuberous sclerosis. 3. Inborn errors of metabolism: phenylketonuria (PKU) or hyperglycinemia. 4. Problems during embryonic development: mental illness (such as diabetes or toxemia) or maternal infection (such as rubella, herpes simplex, or human immunodeficiency virus). 5. Pregnancy and perinatal factors: prematurity, maternal-neonate blood group incompatibility, brain trauma or oxygen deprivation. (Mosby, 18th Edition) and (Straight As in Psychiatric and Mental Health Nursing) An important principle for the nurse to follow in interacting with retarded children is: a. Seen that if the child appears contented, his needs are being met b. Provide an environment appropriate to their developmental task as scheduled c. Treat the child according to his developmental age d. Treat the child according to his chronological age Answer: C Rationale: The nurse should monitor the childs developmental levels and initiate supportive interventions, such as speech, language, or occupational skills, as needed. (Mosby, 18th Edition) and (Straight As in Psychiatric and Mental Health Nursing)

a. b. c. d.

173.

174. The onset of mental retardation is before the child reaches what particular age? a. 18 years old c. 16 years old b. 17 years old d. 15 years old Answer: A Rationale: Mental retardation affects roughly 1% to 3% of the population that occurs before age 18. If mild, it may not be recognized until school age or later. (Straight As in Psychiatric and Mental Health Nursing) 175. Which of the following is true with regards to Mild Mental Retardation? a. Trainable; can reach up to grade 2 and can reach the maturity of a 7-year-old b. Custodial and barely trainable

East West Nursing Review Center

Requires total care throughout life, mental age of a young infant Educable; can reach up to grade 6 and has a maturity of a 12-year-old Answer: D Rationale: In mild mental retardation, the child may be able to live somewhat independently with monitoring or assistance with life changes, challenges or stressors (such as personal illness or the death of a loved one). They are educable; the child can achieve fourth- to sixth-grade reading skills and may master vocational training. They can achieve a mental age (maturity) of 8 to 12 years old. (Mosby, 18th Edition) and (Straight As in Psychiatric and Mental Health Nursing) Situation 10. A personality disorder exists when personality traits become rigid, maladaptive and fixed. The disorder affects the person's cognition, behavior, and style of interacting with others. 176. A male patient is diagnosed with schizotypal personality disorder. Which signs would this patient exhibit during social situation? a. Emotional affect b. Paranoid thoughts c. Aggressive behavior d. Independence need Answer: B Rationale: Patients with schizotypal personality disorder are paranoid about other's motivations, which cause them to experience excessive social anxiety.

c. d.

177.

Which of the following approaches would be most appropriate to use with a patient suffering from narcissistic personality disorder when discrepancies exist between what the patient states and what actually exist? a. Consistency b. Limit setting c. Rationalization d. Supportive confrontation Answer: D Rationale: The nurse would specifically use supportive confrontation with the patient to point out discrepancies between what the patient states and what actually exists to increase responsibility for self.

178. A female patient is admitted to the psychiatric unit with a diagnosis of borderline personality disorder. The nurse should expect the assessment to reveal: a. Coldness, detachment and lack of tender feelings b. Somatic symptoms c. Inability to function as responsible parent d. Unpredictable behavior and intense interpersonal relationships Answer: D Rationale: A patient with borderline personality displays a pervasive pattern of unpredictable behavior, mood and self image. Interpersonal relationships may be intense and unstable and behavior may be inappropriate and impulsive.

179.

The patient with borderline personality disorder who is to be discharged soon threatens to do something to herself if discharged. Which of the following actions by the nurse would be most important? a. Ask a family member to stay with the patient at home temporarily b. Discuss the meaning of the patients statement with her c. Request an immediate extension for the patient d. Ignore the patients statement because its a sign of manipulation Answer: B Rationale: Any suicidal statement must be assessed by the nurse. The nurse should discuss the patients statement with her to determine its meaning in terms of suicide. A 20-year-old client was diagnosed with dependent personality disorder. Which behavior is most likely to be evidence of ineffective individual coping? a. Recurrent self-destructive behavior b. Avoiding relationship c. Showing interest in solitary activities d. Inability to make choices and decision without advice Answer: D Rationale: Individual with dependent personality disorder typically shows indecisiveness submissiveness and clinging behavior so that others will make decisions with them.

180.

Situation 11. Somatoform Disorders are groups of psychiatric disorders in which the patient has persistent physical complaints that cannot be explained by a physical disorder, substance use, or another mental disorder. 181. A somatoform disorder is:

East West Nursing Review Center

A physiologic response to stress An unconscious means to reduce stress A conscious defense against anxiety An intentional attempt to gain attention Answer: B Rationale: When emotional stress overwhelms an individuals ability to cope, the unconscious seeks to reduce stress. A conversion reaction removes the client from the stressful situation and the conversion reactions physical/sensory manifestation causes little or no anxiety in the individual. This lack of concern is called la belle indifference. (Mosby, 18th Edition) 182. A patients family brought the patient into the hospital because of his many somatic complaints. He has been seen by many medical specialists in the past without discovery of organic pathology. The nurse assesses that the patient is experiencing which of the following problems? a. Conversion disorder b. Body dysmorphic disorder c. Hypochondriasis d. Malingering Answer: C Rationale: Hypochondriasis is excessive preoccupation with ones physical health, without organic pathology. (NSNA NCLEX-RN Review, 4th Edition) 183. The nurse finds, during the initial assessment of the star player on the basketball team, that he is not concerned about the sudden paralysis of his shooting arm. This behavior is known as: a. Secondary gain c. Malingering b. La belle indifference d. Hypochondriasis Answer: B Rationale: This lack of concern is identified as la belle indifference and is often a clue that the problem may be psychological rather than physical. (NSNA NCLEX-RN Review, 4th Edition) 184. A patient who works as a receptionist in a physicians office has been an inpatient on a medical unit for over 6 days. The patient continues to complain of severe abdominal symptoms, is febrile, and has the primary care providers deeply concerned because there has been no response to treatment. All tests are negative. The patient finally is diagnosed with Munchausen syndrome. The primary care providers would probably experience feelings of: a. Anger c. Annoyance b. Pity d. Indifference Answer: A Rationale: Anger is the expected response of staff at having been duped by a client with a fictitious disorder; they feel both used and abused. (Mosby, 18th Edition) 185. A patient with a history of stabbing pain in the eyes and blurring and gradual loss of vision is examined by an ophthalmologist, neurologist, and an internist, all of whom have found no organic cause. The patient is admitted to the psychiatric unit when eye complaints increase. Nursing intervention should include: a. Requesting a description of the eye discomfort b. Encouraging becoming involved with unit activities c. Exploring feelings about a possible impending blindness d. Focusing on activities while avoiding discussion of the eye discomfort Answer: D Rationale: The patients eye problems are a conversion reaction. Avoiding discussion of the physical problems prevents the patient from using this topic to avoid an exploration of feelings. Focusing on the safe topic of activities may eventually progress to the discussion of emotion-laden topics such as feelings. (Mosby, 18th Edition) Situation 12. Dissociative disorders are a group of disorders characterized by disturbances in normal waking state. It affects fundamental aspects of consciousness, memory, identity, selfperception and perception of the environment. 186. When would the nurse expect the signs and symptoms of dissociative fugue to be most pronounced? a. After the fugue episode b. During the fugue episode c. Hours before the fugue episode d. Weeks before the fugue episode Answer: A Rationale: After the fugue, the person may experiences depression, grief, shame, intense conflict, confusion, terror, or suicidal or aggressive impulses. In contrast a fugue in progress is rarely recognized. There are no warning signs of an impending fugue episode. 187. Assessment reveals that a patient with dissociative amnesia has had amnesia from a specific time through the present. Then nurse documents that the patient is experiencing: a. Selective amnesia b. Generalized amnesia c. Continuous amnesia

a. b. c. d.

East West Nursing Review Center

d. Systematized amnesia Answer: C Rationale: In continuous amnesia, the patient forgets all events from a given time forward to the present.
188. After teaching a group of students about dissociative disorders, the instructor determines that teaching has been successful when the students correctly identify multiple personality disorder as: a. Dissociative fugue b. Depersonalization disorder c. Dissociative amnesia d. Dissociative identity disorder Answer: D Rationale: Dissociative Identity Disorder (DID), formerly known as multiple personality disorder, is characterized by emergence of two or more distinct identities or subpersonalities that recurrently take control of the patient's consciousness and behavior on a recurring basis. 189. The nurse would expect a patient to report feelings of a dreamlike state or being a detached observer when assessing for which disorder? a. Dissociative fugue b. Dissociative amnesia c. Depersonalization disorder d. Dissociative identity disorder Answer: C Rationale: Depersonalization disorder is characterized by a sense of being in a dreamlike state or being a detached observer. 190. When assessing a patient with Dissociative Identity Disorder, which factor would the nurse identify as least likely to contribute to its development? a. History of seizures b. Emotional, physical or sexual abuse c. Genetic predisposition d. Extreme stress and trauma Answer: A Rationale: A history of seizures has not been linked to the development of DID. Contributing factors may include severe trauma; emotional, physical or sexual abuse; genetic predisposition; lack of nurturing experiences to recover from abuse; and low self-esteem. Situation 13. Flo Angeles, a 16-year-old female patient, is admitted to the psychiatric unit with the diagnosis of anorexia nervosa. 191. Ms. Angeles has lost 20 pounds in 6 weeks. She is very thin but excessively concerned about being overweight. Her daily intake is 10 cups of coffee. The most important initial nursing intervention would be to: a. Compliment her on her lovely figure b. Try to establish a relationship of trust c. Explain the value of adequate nutrition d. Explore the reasons why she does not eat Answer: B Rationale: The problem is psychologic. Therefore the initial approach by the nurse should be directed toward establishing trust. (Mosby, 18th Edition) 192. Nurse Adel is assessing Ms. Angeles. In addition to weight loss, which of the following would the nurse expect to find? a. Irregular menses, diarrhea, dental carries b. Fluid retention, tachycardia, hypertension c. Bradycardia, lanugo, amenorrhea d. Tachycardia, intolerance to heat, skin rashes Answer: C Rationale: The client with anorexia nervosa will demonstrate decreased heart rate due to decreased metabolic rate, as well as possible loss of heart muscle due to starvation. Lanugo occurs as a result of loss of subcutaneous tissue. Amenorrhea occurs as a result of losing fatty tissue in which estrogen is stored. Weight less than 85% of the norm for age and height is also important to establish diagnosis. (Lippincotts Review Series: Mental Health and Psychiatric Nursing, 3rd Edition) 193. The psychoanalytic theory regarding etiology of anorexia nervosa includes which of the following concepts? a. Achievement of secondary gain through control of eating b. Conflict between mother and child over separation and individualization c. Family dynamics that lead to enmeshment of members d. Incorporation of body image ideal of thinness Answer: B

East West Nursing Review Center

Rationale: According to psychoanalytic theory, early mother-child dynamics lead to difficulty with a child establishing a sense of separateness from the mother. Control of eating becomes one area in which the child establishes a sense of independence. (Lippincotts Review Series: Mental Health and Psychiatric Nursing, 3rd Edition) 194. An important behavior modification goal for Ms. Angeles would be: a. The patient will eat every meal for a week b. The patient will gain a pound of weight a week c. The patient will attend group therapy every day d. The patient will talk about food for 1 hour a day Answer: B Rationale: A goal focuses on where the patient should be after certain actions are taken; these patients need to gain weight. (Mosby, 18th Edition)

195. Evaluation of Ms. Angeles requires reassessment of behaviors after admission. The assessment that indicates that the therapy is beginning to become effective is when the patient: a. Is hiding food in pockets of clothing b. States that the admission has been helpful c. Has gained 6 pounds since admission 3 weeks ago d. Is the first to sit down and the last to leave the dining room table Answer: C Rationale: This is objective proof that eating behaviors have improved. (Mosby, 18th Edition) Situation 14. Nurse Jason is assigned to take care of Mr. Paring Roy, 28 years old, who is diagnosed with a severe anxiety disorder. 196. Which of the following statements about anxiety is true? a. Anxiety is usually pathological b. Anxiety is a response to a threat c. Anxiety is usually harmful d. Anxiety is directly observable Answer: B Rationale: Anxiety is a response to a threat arising from internal or external stimuli. 197. Mr. Roy is experiencing an anxiety attack. The most appropriate nursing intervention should include? a. Turning on the television b. Leaving the patient alone c. Staying with the patient and speaking in short sentences d. Ask the patient to play with other patients Answer: C Rationale: Appropriate nursing interventions for an anxiety attack include using short sentences, staying with the patient, decreasing stimuli, remaining calm and medicating as needed.

198.

To establish open and trusting relationship with Mr. Roy, Nurse Jason should? a. Respect patients need for personal space b. Share an activity with the patient c. Give patient feedback about behavior d. Encourage the staff to have frequent interaction with the patient Answer: A Rationale: Moving to a patients personal space increases the feeling of threat, which increases anxiety.

199. Mr. Roy is pacing the floor and appears anxious. Nurse Jason approaches in an attempt to alleviate the patients anxiety. The most therapeutic question by the nurse would be? a. Would you like to watch TV? b. Would you like me to talk with you? c. Are you feeling upset now? d. Ignore the patient Answer: B Rationale: The nurses presence may provide the patient with support & feeling of control. 200. When planning the discharge of Mr. Roy, Nurse Jason evaluates achievement of the discharge maintenance goals. Which goal should be most appropriately included in the plan of care requiring evaluation? a. The patient eliminates all anxiety from daily situations b. The patient ignores feelings of anxiety c. The patient maintains contact with a crisis counselor d. The patient identifies anxiety producing situations Answer: D Rationale: Recognizing situations that produce anxiety allows the patient to prepare to cope with anxiety or avoid specific stimulus.

East West Nursing Review Center

Situation 15. Nurse Kokey is working in a psychiatric facility where he encounters multiple patients admitted due to substance abuse. 201. With a tentative diagnosis of opiate addiction, Nurse Kokey should assess a recently hospitalized patient for signs of opiate withdrawal. These signs would include: a. Rhinorrhea, convulsions and subnormal temperature b. Nausea, constricted pupils and constipation c. Drowsiness and decreased level of consciousness d. Muscle aches, pupillary dilation and yawning Answer: D Rationale: Muscle aches, pupillary dilation, and yawning are adaptations associated with opiate withdrawal which occurs after cessation or reduction of prolonged moderate or heavy use of opiates. 202. Which of the following would Nurse Kokey eliminate from the diet of a patient in alcohol withdrawal? a. Milk c. Soda b. Orange Juice d. Regular Coffee Answer: D Rationale: Regular coffee contains caffeine which acts as psychomotor stimulants, which aggravates the signs and symptoms of alcohol withdrawal. It leads to feelings of anxiety and agitation, and may add to tremors or wakefulness. 203. Nurse Kokey is caring for a patient with a diagnosis of cocaine addiction. He is aware that a serious effect of inhaling cocaine is? a. Deterioration of nasal septum b. Acute fluid and electrolyte imbalances c. Extra pyramidal tract symptoms d. Esophageal varices Answer: A Rationale: Cocaine is a chemical that when inhaled, causes destruction of the mucous membranes of the nose. Regular cocaine snorting can lead to the loss of sense of smell and to nosebleed, swallowing difficulty, hoarseness, and nasal septum irritation. 204. A patient, who is experiencing alcohol withdrawal, exhibits tremors, diaphoresis and hyperactivity. Blood pressure is 190/87 mm Hg and pulse rate of 92 beats per minute. Which of these medications would Nurse Kokey expect to administer? a. Naloxone (Narcan) b. Benztropine (Cogentin) c. Lorazepam (Ativan) d. Haloperidol (Haldol) Answer: C Rationale: The nurse would most likely administer a benzodiazepine, such as lorazepam (Ativan) to the client who experience symptoms of alcohol withdrawal. The symptoms of withdrawal are caused by rebound phenomenon when the sedative effect of alcohol to the CNS begins to decrease. 205. Which of the following would Nurse Kokey expect to assess to a patient who is exhibiting late signs of heroin withdrawal? a. Yawning and diaphoresis b. Restlessness and irritability c. Constipation and steatorrhea d. Vomiting and diarrhea Answer: D Rationale: Vomiting and diarrhea are usually the late signs of heroin withdrawal, along with muscle spasm, fever, nausea, repetitive, abdominal cramps and backache. Situation 16. Alcohol abuse is a psychiatric diagnosis describing the use of alcoholic beverages despite negative consequences. Appropriate management and nursing interventions must be considered. 206. To determine the potential an individual has for a drinking problem, the nurse uses the CAGE Screening Test for Alcoholism. One of the four questions included in this test is: a. Do you feel you are a normal drinker? b. Are you always able to stop drinking when you want to? c. Have you ever felt bad or guilty about your drinking? d. How often did you have a drink containing alcohol in the past year? Answer: C Rationale: The CAGE screening test for alcoholism contains four questions, corresponding to the letters CAGE; C Have you ever felt you ought to Cut down on your drinking?; A Have people Annoyed you by criticizing your drinking?; G Have you ever felt bad or Guilty about your drinking?; and E Have you ever had a drink first thing in the morning (as an Eye-opener) to steady your nerves or get rid of a hangover? Options A and B are incorrect because these questions are 2 of the 26 questions that are included on the Michigan Alcohol Screening Test (MAST). Option D is incorrect because this question is 1 of the 10 questions that is included on the Alcohol Use Disorders Identification Test (AUDIT). (Mosby, 18th Edition)

East West Nursing Review Center

207.

The most important factor in rehabilitation of a patient addicted to alcohol is: a. The availability of community resources b. The accepting attitude of the clients family c. The qualitative level of the clients physical state d. The clients emotional or motivational readiness Answer: D Rationale: Intrinsic motivation, stimulated from within the learner, is essential if rehabilitation is to be successful. Often clients are most emotionally ready for help when they have hit bottom. Only then are clients motivationally ready to face reality and put forth the necessary energy and effort to change behavior. (Mosby, 18th Edition)

208. A patient who has just begun attending Alcoholics Anonymous asks the nurse whether it is really necessary to go to meetings. The nurses best response would be: a. Yes, if you really want to get well. b. Its your decision about whether or not you want to attend. c. You think that attending these meetings may not be helpful. d. It sounds like you think attending meetings is too much effort. Answer: C Rationale: This statement reflects the underlying theme in the patients statement and nonjudgmentally encourages the patient to verbalize further. (Mosby, 18th Edition) 209. A patient is attending Alcoholics Anonymous after withdrawing from alcohol. The nurse recognizes that the ultimate purpose of self-help group such as AA is to help members: a. Develop functional relationships b. Change destructive behavior c. Identify how they present themselves to others d. Understand their patterns of interacting within the group Answer: B Rationale: The purpose of a self-help group is for individuals to develop their strengths and new individual patterns of coping. (Mosby, 18th Edition) 210. Patients addicted to alcohol use denial as one of their prime defense mechanisms. The nurse further understands that these patients use denial to: a. Reduce their feelings of guilt b. Live up to others expectation c. Make them seem more independent d. Make them look better in the eyes of others Answer: A Rationale: The patient is using denial as a defense against feelings of guilt, which will reduce anxiety and protect the self. (Mosby, 18th Edition) Situation 17. Nurse Pharma is assigned as the medicating nurse for the morning shift. It is essential that she knows the modes of action, side effects, adverse reactions, and nursing responsibilities of the different drugs for psychiatric patients in order to ensure patient safety. 211. A patient receiving lithium has a serum drug level of 2.3 mEq/L. What would Nurse Pharma most likely assess given this serum drug level? a. Urinary incontinence b. Seizures c. Muscle weakness d. Hyperactive deep tendon reflexes Answer: D Rationale: With a serum level between 2 and 2.5 mEq/L, assessment findings include hyperactive deep tendon reflexes, persistent nausea and vomiting, blurred vision, and muscle twitching. Option A is incorrect because urinary incontinence is associated with drug levels of 2.5 to 3 mEq/L. Option B is incorrect because seizures are associated with drug levels above 3 mEq/L. Option C is incorrect because muscle weakness is seen with lithium levels ranging from 1.5 to 2 mEq/L. (Straight As in Psychiatric & Mental Health Nursing, A Review Series) 212. Which information would Nurse Pharma include when teaching the parents of a child who is receiving methylphenidate (Ritalin)? a. Monitor the childs blood glucose level because the drug increases the risk of diabetes b. Have the child undergo IQ testing because the drug may decrease intelligence c. Monitor the childs growth closely because the drug may interfere with growth and development d. Have the child's hearing tested because the drug can cause hearing loss Answer: C Rationale: The childs physical growth should be monitored because methylphenidate (Ritalin) may cause weight loss and temporary interference with growth and development. (Straight As in Psychiatric & Mental Health Nursing, A Review Series) 213. Which medication would Nurse Pharma expect the physician to prescribe for a patient with depersonalization disorder?

East West Nursing Review Center

Clomipramine (Anafranil) Lithium (Eskalith) Chlorpromazine (Thorazine) Fluphenazine (Prolixin) Answer: A Rationale: Clomipramine (Anafranil), a tricyclic antidepressant (TCA), has been moderately successful in treating clients with depersonalization disorders. (Straight As in Psychiatric & Mental Health Nursing, A Review Series) 214. Which of the following behaviors indicates to Nurse Pharma that the patients antipsychotic medication is having a desired effect? a. The patient states that her voices are not as threatening b. The patient reports having inner feelings of restlessness c. The patient sleeps all day d. The patient reports muscular stiffening in her face and arms Answer: A Rationale: A desired effect of the antipsychotics is to reduce the disturbing quality of hallucinations and delusions. (NSNA, NCLEX-RN Review, 4th Edition) 215. A patient is to receive conventional antipsychotic drug therapy. Which drug would Nurse Pharma expect to administer? a. Prolixin c. Seroquel b. Olanzapine d. Risperidone Answer: A Rationale: Fluphenazine (Prolixin) is a conventional (typical) antipsychotic. (Straight As in Psychiatric & Mental Health Nursing, A Review Series) Situation 18. Electroconvulsive Therapy is used as a treatment modality for severe depression when psychotherapy and pharmacotherapy are not effective. 216. Which patient would the nurse expect to prepare for ECT? a. A female patient with dysthymic disorder b. An elderly male with a major depressive disorder with a history of stroke c. A female patient with depression and hypomania due to a cyclothymic disorder d. A middle-age, female patient with major depression and an immediate risk of suicide Answer: D Rationale: Electroconvulsive Therapy (ECT) may be used to treat major depression as well as certain psychotic disorders particularly in situations of severe depression when psychotherapy and medications have been ineffective, when ECT poses a lower risk than other treatments. 217. A patient is scheduled for ECT at 10 am. Which action of the nurse would be most appropriate? a. Giving the patient a clear liquid breakfast b. Catheterizing the patient for a morning urine sample c. Administering prescribed medications to reduce secretions d. Allowing the patient to keep his dentures in his mouth Answer: C Rationale: In this situation, the nurse should administer the prescribed medication (such as atropine or glycopyrrolate) to reduce secretions, prevent aspiration, and reduce the risk of bradycardia.

a. b. c. d.

218.
A neuromuscular blocking agent is administered to a patient before ECT therapy. The nurse should carefully observe the patient for? a. Respiratory difficulties b. Nausea and vomiting c. Dizziness d. Seizures Answer: A Rationale: A neuromuscular blocker, such as succinylcholine (Anectine) produces respiratory depression because it inhibits contractions of respiratory muscles. 219. During electroconvulsive therapy (ECT) the patient receives oxygen by mask via positive pressure ventilation. The nurse assisting with this procedure knows that positive pressure ventilation is necessary because? a. Muscle relaxations given to prevent injury during seizure activity depress respirations b. Decrease oxygen to the brain increases confusion and disorientation c. Grand mal seizure activity depresses respirations d. Anesthesia is administered during the procedure Answer: A Rationale: A short acting skeletal muscle relaxant such as succinylcholine (Anectine) is administered during this procedure to prevent injuries during seizure.

East West Nursing Review Center

220. When assessing the patient immediately after ECT, the nurse expects to find: a. Permanent short-term memory loss and hypertension b. Transitory short and long term memory loss and confusion c. Transitory short-term memory loss and permanent long-term memory loss d. Permanent long-term memory loss and hypomania Answer: B Rationale: ECT commonly causes transitory short and long term memory loss and confusion, especially in geriatric clients. It rarely results in permanent short and long term memory loss. Situation 19. Sexuality is expressed not just in a person's appearance but also in his attitude, behaviors and relationships. 221. The multidisciplinary care team would suspect gender identity disorder if a patient: a. Has a strong desire to be of the same sex b. Insists that he or she is of the opposite sex c. Prefers the opposite sex d. Engages in sexual activities with the same sex Answer: B Rationale: Gender identity disorder is marked by a repeatedly stated desire to be of the opposite sex or insistence that one is of the opposite sex. Option C is incorrect because Heterosexual individuals prefer members of the opposite sex. Option D is incorrect because Homosexual individual engages in sexual activities with members of the same sex. 222. A student is reviewing content for a test on the phases of the sexual response cycle. The student demonstrates understanding of the material by identifying which phase as the one involving fantasy and expectation? a. Desire phase c. Orgasm phase b. Excitement phase d. Resolution phase Answer: A Rationale: Desire phase involves fantasy and expectation. Option B is incorrect because Excitement phase is the arousal phase that prepares both partners for intercourse. Option C is incorrect because Orgasm phase is the peak of sexual excitement. Option D is incorrect because in the Resolution phase, the body returns to its normal unexcited state. 223. When developing the care plan for a patient diagnosed with premature ejaculation, which intervention would the nurse expect to include? a. Administration of sildenafil (Viagra) b. Preparation for insertion of a penile prosthesis c. Instructions in the squeeze technique d. Sex therapy to reduce performance anxiety Answer: C Rationale: For the patient with premature ejaculation, treatment may include squeeze technique to help the patient control of ejaculatory tension. How it is done: When the patient feels the urge to ejaculate, place the thumb on the frenulum of the penis; and place the index and middle fingers above and below the coronal ridge. Squeeze penis from front to back. Apply and release pressure during a touching exercise. 224. Which behavior or disorder would the nurse identify as a possible cause of or contributing factor to sexual dysfunction? a. Exercise b. Drug use c. Supplemental vitamin use d. Dissociative disorders Answer: B Rationale: Sexual dysfunctions sometimes stem from transient conditions such as drug or alcohol use. 225. Which treatment would the nurse expect to include in the care plan for a woman with orgasmic dysfunction? a. Use of soothing bubble baths b. Exercises involving touching her partner c. More frequent sexual intercourse d. Increased in the degree of sexual arousal Answer: B Rationale: Sensate focus exercises are recommended for female orgasmic disorder. These exercises emphasize touching and awareness of sensual feelings throughout the entire body while minimizing the importance of intercourse and orgasm. The couple takes turns giving and receiving touch. Situation 20. Crisis is temporary state of severe emotional disorganization resulting from failure of coping mechanisms and/or lack of support. Treatment must be immediate,

East West Nursing Review Center

supportive and directly responsive to the immediate crisis. 226. The most critical factor for a nurse to determine during crisis intervention would be the patients: a. Developmental theory b. Willingness to restructure the personality c. Available situational supports d. Underlying unconscious conflict Answer: C Rationale: Personal internal strength and supportive individuals are critical factors that can be employed to assist the individual to cope with a crisis. 227. A nurse is assisting in panning care for a patient being admitted to the nursing unit who attempted suicide. Which of the following priority nursing interventions will the nurse include in the plan of care? a. Check the whereabouts of the patient every 15 minutes b. Suicide precautions with 30 minute checks c. One-to-one suicide precautions d. Ask that the patient report suicidal thoughts immediately Answer: C Rationale: One-to-one suicide precautions are required for the client who has attempted suicide. 228. A nurse suggests a crisis intervention group to a patient experiencing a developmental crisis. These groups are successful because the: a. Crisis intervention worker is a psychologist and understands behavior patterns b. Crisis group supplies a workable solution to the patients problem c. Patient is encouraged to talk about personal problems d. Patient is assisted to investigate alternative approaches to solving the identified problem Answer: D Rationale: Crisis intervention group helps client reestablish psychologic equilibrium by assisting them to explore new alternatives for coping. It considers realistic situations using rational and flexible problem solving methods. 229. A patient who is unable to cope with the sudden loss of a job and who is feeling confused and unable to make decisions is said to be experiencing which of the following? a. Adventitious crisis b. Maturational crisis c. Situational crisis d. Cultural crisis Answer: C Rationale: A situational crisis is one that is often sudden and unavoidable, such as losses and death of a loved one. 100. A nurse is caring for a young woman who was sexually assaulted. Which of the following is indicative of successful adjustment to the trauma? a. She moves to another city b. She resumes her work and activities c. She takes classes in the martial arts d. She remains silent about the assault Answer: B Rationale: The goal of adjustment is to have the women return to her precrisis level of functioning.

East West Nursing Review Center

Potrebbero piacerti anche